SlideShare a Scribd company logo
1 of 73
Tải tài liệu tại sividoc.com
Viết đề tài giá sinh viên – ZALO:0973.287.149-TEAMLUANVAN.COM
ĐẠI HOC THÁI NGUYÊN
TRƯ NG ĐẠI HOC KHOA HOC
NGUYEN NGOC HÀ
BAT ĐANG THỨC VÀ BÀI TOÁN CỰC
TR± TRONG L P CÁC ĐA THỨC VÀ
PHÂN THỨC H SO NGUYÊN
LU N VĂN THẠC SỸ TOÁN HOC
THÁI NGUYÊN - NĂM 2016
Tải tài liệu tại sividoc.com
Viết đề tài giá sinh viên – ZALO:0973.287.149-TEAMLUANVAN.COM
ĐẠI HOC THÁI NGUYÊN
TRƯ NG ĐẠI HOC KHOA HOC
NGUYEN NGOC HÀ
BAT ĐANG THỨC VÀ BÀI TOÁN CỰC
TR± TRONG L P CÁC ĐA THỨC VÀ
PHÂN THỨC H SO NGUYÊN
LU N VĂN THẠC SỸ TOÁN HOC
Chuyên ngành: PHƯƠNG PHÁP TOÁN SƠ CAP
Mã so: 60 46 01 13
Ngư i hư ng dȁn khoa hoc:
GS. TSKH. NGUYEN VĂN M U
THÁI NGUYÊN - NĂM 2016
i
Viết đề tài giá sinh viên – ZALO:0973.287.149-TEAMLUANVAN.COM
Mnc lnc
M đau 1
1 M t so kien thfíc chuan bị 3
1.1 M®t so tính chat cơ bản của đa thác với h so nguyên . . . . 3
1.2 M®t so tính chat của phân thác hǎu tỉ với h so nguyên . . . 8
1.3 Định lý Viète............................................................................... 12
1.4 M®t so bat đȁng thác cơ bản ............................................................ 13
2 Các dạng toán ve đa thfíc và phân thfíc v i h so nguyên 19
2.1 Các dạng toán ve đa thác m®t bien với h so nguyên và h
thác Viète.......................................................................................19
2.2 Đa thác với các h so nguyên và đong dư thác .............................30
2.3 Bat đȁng thác phân thác sinh bởi tam thác b c hai trên m®t
khoảng............................................................................................35
2.4 Bat đȁng thác sinh bởi hàm phân tuyen tính trên m®t khoảng 40
2.5 Phân thác chính quy và m®t so tính chat .....................................45
3 M t so bài toán liên quan đen bat đang thfíc và cfic trị trên
t p so nguyên 49
3.1 Bat đȁng thác trên t p so nguyên .................................................49
3.2 Cực trị trên t p so nguyên .......................................................... 58
Ket lu n 69
Tài li u tham khảo 70
1
Viết đề tài giá sinh viên – ZALO:0973.287.149-TEAMLUANVAN.COM
M đau
Chuyên đe đa thác là m®t chuyên đe rat quan trong ở b c trung hoc phő
thông. Đa thác không chỉ là đoi tượng nghiên cáu trong tâm của Đại so mà
còn là công cụ đac lực trong nhieu lĩnh vực khác của toán hoc.
Trong các kì thi hoc sinh giỏi toán các cap, Olympic Toán sinh viên, các
bài toán liên quan tới đa thác nói chung và đ c bi t là các bài toán ve bat
đȁng thác, cực trị của đa thác, phân thác có h so nguyên thường xuyên
được đe c p. Nhǎng dạng toán này thường được xem là thu®c loại khó, hơn
nǎa phan kien thác ve đa thác, phân thác h so nguyên lại không nam trong
chương trình chính thác của So hoc và Đại so b c trung hoc phő thông.
Đe đáp áng nhu cau boi dương giáo viên và boi dương hoc sinh giỏi ve
chuyên đe đa thác, tôi đã làm lu n văn: Bat đȁng thác và bài toán cực trị
trong lớp các đa thác và phân thác h so nguyên. Lu n văn gom phan mở
đau, ba chương, phan ket lu n và danh mục tài li u tham khảo.
Chương I trình bày các kien thác cơ bản ve đa thác và phân thác h so
nguyên, định lý Viète, m®t so bat đȁng thác cơ bản.
Chương II trình bày m®t so dạng toán ve bat đȁng thác và cực trị trong
lớp các đa thác và phân thác h so nguyên, phân thác chính quy và áp dụng.
Chương III trình bày m®t so bat đȁng thác và bài toán cực trị trên t p
so nguyên.
Lu n văn có the được xem như m®t tài li u boi dương giáo viên và boi
dương hoc sinh giỏi ve chuyên đe đa thác. Có the sả dụng lu n văn trong
vi c giảng dạy hoc sinh thi hoc sinh giỏi các cap, Olympic sinh viên.
2
Viết đề tài giá sinh viên – ZALO:0973.287.149-TEAMLUANVAN.COM
Lu n văn được hoàn thành dưới sự hướng dan trực tiep của GS.TSKH
Nguyen Văn M u. Tác giả xin bày tỏ lòng biet ơn sâu sac ve sự chỉ bảo t n
tình của Thay trong suot quá trình xây dựng đe cương cũng như hoàn thành
lu n văn.
Tác giả xin gải lời cảm ơn chân thành tới TS Nguyen Thị Thu Thủy và
các quí thay cô đã đoc, kiem tra, đánh giá và đưa ra nhǎng ý kien quý báu
đe lu n văn được hoàn thi n hơn.
Tác giả xin chân thành cảm ơn quý Thay Cô trong Ban Giám hi u, phòng
sau Đại hoc, khoa Toán Tin trường Đại hoc Khoa hoc, Đại hoc Thái Nguyên
tạo đieu ki n thu n lợi trong suot quá trình hoc t p và hoàn thành lu n văn.
Trong khuôn khő m®t lu n văn, tác giả chưa the trình bày được het các
van đe ve đa thác và phân thác h so nguyên. Tuy bản thân đã có nhieu
co gang, no lực nghiên cáu, song do đieu ki n và trình đ® còn hạn che nên
nhǎng ket quả đạt được trong lu n văn còn rat khiêm ton. Tác giả kính mong
nh n được ý kien đóng góp quí báu của các thay cô đe bản lu n văn được
hoàn thi n hơn.
Tác giả xin chân thành cảm ơn!
Thái Nguyên, ngày 20 tháng 5 năm 2016.
Hoc viên
Nguyen Ngoc Hà
3
Viết đề tài giá sinh viên – ZALO:0973.287.149-TEAMLUANVAN.COM
Chương 1
M t so kien thfíc chuan bị
Trong chương này sě trình bày m®t so kien thác cơ bản: định nghĩa, m®t
so tính chat ve sự chia het, ve nghi m nguyên, ve h so . . . của đa thác và
phân thác h so nguyên. đây ta sě sả dụng m®t so ký hi u: Cho đa thác
f(x) = anxn
+ an−1xn−1
+ · · · + a1x + a0, neu f(x) có các h so là so nguyên
thì ta ký hi u f(x) ∈ Z[x], neu f(x) có các h so là so hǎu tỉ thì ta ký hi u
f(x) ∈ Q[x].
1.1 M t so tính chat cơ bản của đa thfíc v i h so
nguyên
Định lj 1.1 (xem [4]). Cho đa thác f(x) = anxn
+an−1xn−1
+· · ·+a1x+a0 ∈
Z[x], an 0, a là so nguyên. Khi đó [f(x) − f(a)].(x − a).
ChGng minh.
Ta có
f(a) = anan
+ an−1an−1
+ · · · + a1a + a0,
f(x) − f(a) = an (xn
− an
) + an−1 xn−1
− an−1
+ · · · + a1(x − a).(x − a).
V y ta có đieu can cháng minh.
4
Viết đề tài giá sinh viên – ZALO:0973.287.149-TEAMLUANVAN.COM
q
−
q q q
Bài toán 1.1 (xem [4]). Cháng minh rang neu phân so toi giản
1) là nghi m của đa thác với h so nguyên
f(x) = anxn
+ an−1xn−1
+ · · · + a1x + a0
thì p là ước của a0 và q là ước của an.
Lài giai.
p
, ((p, q) =
q
p
Giả sả phân thác toi giản
q
là nghi m của đa thác f(x). Khi đó, ta có
f
p
= an
p
n
+ an−1
p n−1
+ · · · + a1
p
+ a0 = 0.
Tà đó, ta có
và
anpn
= −q(an−1pn−1
+ · · · + a1qn−2
p + a0qn−1
) (1.1)
a0qn
= −p(anpn−1
+ an−1pn−2
q + · · · + a1qn−1
). (1.2)
Tà (1.1) suy ra anpn
chia het cho q mà (p, q) = 1 nên an chia het cho q.
Tà (1.2)suy ra a0qn
chia het cho p mà (p, q) = 1 nên a0 chia het cho p.
p
Bài toán 1.2 (xem [4]). Cháng minh rang neu phân thác toi giản
1) là nghi m của đa thác với h so nguyên
f(x) = anxn
+ an−1xn−1
+ · · · + a1x + a0
thì p − mq là ước của f(m) với m là so nguyên.
Lài giai.
Phân tích f(x) theo các lũy thàa của (x − m) ta được
, ((p, q) =
q
f(x) = an(x − m)n
+ bn−1(x − m)n−1
+ · · · + b1(x − m) + b0 = g(x − m).
Nh n xét rang các h so b0, bn 1 là các so nguyên vì m là m®t so nguyên.
p
Ta có f(m) = b0. Thay x bởi ta thu được đȁng thác
q
f
p
= g
p
− m = g
p − mq
= 0.
q
q
q
5
Viết đề tài giá sinh viên – ZALO:0973.287.149-TEAMLUANVAN.COM
Do đó
p − mq
q
b0 = f(m).
là nghi m của g(x). theo Bài toán 2.6 thì p − mq là ước của
Bài toán 1.3. Cho đa thác f(x) có h so nguyên thỏa mãn đieu ki n
f (0), f (1), . . . , f (m − 1) đeu không chia het cho m (m là so nguyên dương
cho trước, m > 1). Cháng minh rang f(x) = 0 không có nghi m nguyên.
Lài giai.
Giả sả f(x) = 0 có nghi m nguyên là x = c, khi đó
f(x) = (x − c)g(x), g(x) ∈ Z[x].
Ta có
f(0) = (0 − c)g(0),
f(1) = (1 − c)g(1),
. . .
f(m − 1) = (m − 1 − c)g(m − 1).
Vì 0 − c, 1 − c, . . . , m − 1 − c là m so nguyên liên tiep nên phải có m®t so
chia het cho m.
Vì v y trong m so f(0), f(1), . . . , f(m − 1) phải có ít nhat m®t so chia
het cho m. Đieu này trái với giả thiet.
V y f(x) = 0 không có nghi m nguyên.
Bài toán 1.4. Cho đa thác P(x) với các h so nguyên, chia het cho 3 khi x
lay các giá trị nguyên k, k + 1, k + 2. Cháng minh rang P(m) .3 với moi so
nguyên m.
ChGng minh.
Với hai so nguyên m và n phân bi t, ta có P(m) − P(n).(m − n).
Ta có các so P (m) − P (k), P (m) − P (k + 1) và P (m) − P (k + 2) theo
thá tự đó lan lượt chia het cho m − k, m − (k + 1), m − (k + 2) với moi
m ∈
/ {k, k + 1, k + 2}.
6
Viết đề tài giá sinh viên – ZALO:0973.287.149-TEAMLUANVAN.COM
Vì m − k, m − (k + 1), m − (k + 2) là ba so nguyên liên tiep nên trong đó
có m®t so chia het cho 3. Do đó trong các so P (m) − P (k), P (m) − P (k + 1)
và P(m) − P(k + 2) có m®t so chia het cho 3.
M t khác, theo giả thiet, các so P (k), P (k + 1), P (k + 2) đeu chia het cho
3. V y P(m).3 với moi so nguyên m.
Bài toán 1.5 (xem [4]). Cho đa thác f (x) ∈ Z[x]. Cháng minh rang neu
phương trình f (x) = 1 có nhieu hơn 3 nghi m nguyên phân bi t thì phương
trình f(x) = −1 không có nghi m nguyên.
Lài giai.
Giả sả phương trình f (x) = −1 có nghi m nguyên là a thì f (a) = −1.
Goi x1, x2, x3, x4 là 4 nghi m nguyên phân bi t của phương trình f(x) = 1,
thì
Suy ra
f(x) − 1 = (x − x1)(x − x2)(x − x3)(x − x4)g(x).
f(a) − 1 = −2 = (a − x1)(a − x2)(a − x3)(a − x4)g(a),
trong đó (a − x1), (a − x2), (a − x3), (a − x4) là 4 so nguyên phân bi t.
Nhưng −2 không the phân tích được thành tích của 4 so nguyên khác
nhau nên đieu giả sả ở trên là sai. V y phương trình f(x) = −1 không có
nghi m nguyên.
Bài toán 1.6. Giả sả P (x) là đa thác b c 1991 với h so nguyên. Xét đa
thác Q(x) = P 2
(x) − 9. Cháng minh rang so nghi m nguyên của đa thác
Q(x) nhỏ hơn 1996.
Lài giai.
Giả sả so nghi m của đa thác Q(x) không nhỏ hơn 1996.
Q(x) = 0 ⇔ P2
(x) − 9 = 0 ⇔ [P(x) − 3][P(x) + 3] = 0.
7
Viết đề tài giá sinh viên – ZALO:0973.287.149-TEAMLUANVAN.COM
Goi x1, x2, . . . , xk là các nghi m nguyên của
P(x) = 3, (x1 < x2 < · · · < xk)
và y1, y2, . . . , yl là các nghi m nguyên của
P(x) = −3, (y1 < y2 < · · · < yl).
Rõ ràng xi /= yj, ∀i, j. Vì deg P(x) = 1991 nên k ≤ 1991; l ≤ 1991.
M t khác k + l chính là so nghi m của đa thác Q(x) nên theo giả thiet
phản cháng thì k + l ≥ 1996. Tà đó ta có k ≥ 5, l ≥ 5, suy ra ton tại
i0, j0 (1 ≤ i0 ≤ k; 1 ≤ j0 ≤ l) sao cho
|xi0 − yj0| ≥ 7. (1.3)
Giả sả
P(x) = a1991x1991
+ a1990x1990
+ · · · + a1x + a0
với ai ∈ Z, i = 0, 1991.
The thì tà ai ∈ Z, i = 0, 1991, P(xi0) = 3 và P (yi0) = −3, suy ra
P(xi0 ) − P (yi0 ) = 6.
Vì P(x) là đa thác với h so nguyên và xi0, yi0 là các so nguyên nên ta có
P(yi ) − P(xi ). i — xi .
Như v y 6.(yi0
0
— xi0), suy ra
0
.y 0 0
yi0 − xi0 ≤ 6. (1.4)
Tà (1.3) và (1.4) suy ra mâu thuan. V y giả thiet phản cháng là sai, tác là
đa thác Q(x) = P2
(x) − 9 không the có quá 1995 nghi m nguyên.
Nh n xét 1.1. Ta có the cháng minh (1.3) như sau: Vì k ≥ 5, l ≥ 5, xi /
=
yj,
∀i, j do đó có ít nhat ba nghi m trong so các nghi m yj (j = 1, l) nhỏ
hơn (ho c lớn hơn) các nghi m xi (i = 1, k). Giả sả
x1 < x2 < · · · < xk < yp < yp+1 < yp+2
8
Viết đề tài giá sinh viên – ZALO:0973.287.149-TEAMLUANVAN.COM
(ở đây yp, yp+1, yp+2 là 3 trong so các nghi m y1, y2, y3, . . . , yl).
Chú ý là ở đây các xi (i = 1, k), yp, yp+1, yp+2 đeu là so nguyên nên
|yp+2 − x1| = yp+2 − x1 ≥ k − 1 + 3 = k + 2.
Do k ≥ 5 nên suy ra |yp+2 − xi| ≥ 7.
1.2 M t so tính chat của phân thfíc hfiu tỉ v i h so
nguyên
Định nghĩa 1.1 (xem [4]). Hàm so f : R → R có dạng
P (x)
f(x) =
Q(x)
được goi là phân thác hǎu tỉ, trong đó P(x), Q(x) là các đa thác.
Neu đa thác P(x) và Q(x) là các đa thác có h so hǎu tỉ thì bang vi c
quy đong mau so ta sě đưa f(x) ve dạng
f(x) =
P1(x)
Q1(x)
trong đó P1(x) và Q1(x) là các đa thác có h so nguyên.
P (x)
Do v y phân thác hǎu tỉ f(x) =
Q(x)
được goi là phân thác hǎu tỉ có
h so nguyên neu như P(x), Q(x) ∈ Q[x].
Bài toán 1.7 (xem [4]). Cho phân thác hǎu tỉ
1
f(x) =
ax + b
∈ Q
với moi x ∈ Z. Cháng minh rang a, b ∈ Q.
ChGng minh.
1 1
f(x) = ∈ Q với moi x ∈ Z nên ax + b = ∈ Q với moi x ∈ Z.
ax + b f(x)
V y ax + b ∈ Q[x] hay a, b ∈ Q.
9
Viết đề tài giá sinh viên – ZALO:0973.287.149-TEAMLUANVAN.COM
Bài toán 1.8 (xem [4]). Cháng minh rang neu f(x) =
C
1
ax + b
∈ Q với moi
x ∈ Z thì f(x) có dạng f(x) =
ChGng minh.
1
Ax + B
với A, B, C thu®c Z.
f(x) = ∈ Q với moi x ∈ Z nên theo Bài toán 1.7 ta có a, b ∈ Q.
ax + b
Đ t a =
m
,b =
n
e
f
(m, n, e, f ∈ Z). Khi đó
1
f(x) = m e
x +
nf
= =
mfx + ne
C
Ax + B
n f
với A, B, C thu®c Z.
Bài toán 1.9 (xem [4]). Cho phân thác hǎu tỉ f(x) =
x ∈ Z.
Cháng minh rang f(x) có the bieu dien dưới dạng
Ax + B
ax + b
cx + d
∈ Q với moi
ChGng minh.
f(x) =
Cx + D
(A, B, C, D ∈ Z). (1.5)
Neu ad − bc = 0 thì f(x) = const nên bieu dien (1.5) là hien nhiên.
Xét trường hợp ad − bc /= 0.
Neu c = 0 thì bieu dien (1.5) là hien nhiên.
Neu c = 0 thì sả dụng phân tích
f(x) − f(0)
x
1
=
αx + β
∈ Q với moi
x ∈ Z. Áp dụng Bài toán 1.8 ta sě được dạng bieu dien (1.5).
Nh n xét rang ket quả của Bài toán 1.9 cũng đúng trong trường hợp tőng
quát.
Bài toán 1.10 (xem [4]). Cho phân thác hǎu tỉ
P (x)
f(x) = ∈ Q ∀x ∈ Z, (P(x), Q(x)) = 1.
Q(x)
Cháng minh rang f(x) có the bieu dien được dưới dạng phân thác của hai
đa thác với h so nguyên.
10
Viết đề tài giá sinh viên – ZALO:0973.287.149-TEAMLUANVAN.COM
p
x + qk
k
k=1
ChGng minh.
Giả sả
m
P(x) = a0 + a1x + · · · + amx ,
n
Q(x) = b0 + b1x + · · · + bnx .
Tại x = j (j = 0, 1, . . . , m + n) hàm f (x) nh n các giá trị hǎu tỉ tương
áng là cj. Khi đó ta có h phương trình tuyen tính với m + n + 2 ȁn:
a0, a1, . . . , am, b0, b1, . . . , bn dạng
a0 + a1k + · · · + amkm
− b0ck − b1ckk − · · · − bnckkn
= 0,
trong đó k = 0, 1, . . . , m + n.
Hai nghi m của h này cho ta hai c p đa thác P (x), Q(x) và P1(x), Q1(x)
có tính chat
P(k) − ckQ(k) = 0, P1(k) − ckQ1(k) = 0, ∀k = 0, m + n.
Hai c p nghi m này cho ta đa thác
g(x) = P (x)Q1(x) − P1(x)Q(x), deg g(x) ≤ m + n
nh n giá trị 0 tại m + n + 1 điem nên g(x) ≡ 0.
Do P (x) và Q(x) nguyên to cùng nhau nên P (x) = cP1(x); Q(x) =
cQ1(x).
V y h đã cho chỉ có m®t nghi m với sự sai khác m®t thàa tỉ l và như
v y ton tại ma tr n cap m + n + 1 trong ma tr n h so của h phương trình
đe định thác của nó khác 0 và nghi m đã nh n được là các so hǎu tỉ. Đây là
đieu can cháng minh.
Bài toán 1.11 (xem [4]). Cho p là m®t so nguyên dương, q ∈ [0, 1]. Giả sả
x ∈ [qp+1
, 1] và
f(x) =
Y x − q
.
11
Viết đề tài giá sinh viên – ZALO:0973.287.149-TEAMLUANVAN.COM
p
=
. . −
.
.
−
−
. .
.
.
x+ q
j
p
.
.
−
.
.
−
j
p
−
p
. .
ta có x ≥ qi
. V y nên . .
k
−
Cháng minh rang
|f(x)| ≤
Y 1 − q
.
ChGng minh.
k=1
1 + qk
Ta có 0 < qp+1
< qp
< · · · < q < 1. Với qj+1
≤ x ≤ qj
thì khi i ≥ j + 1
Xét hi u
x − qi
x − qi
.x + qi .
1 − qi
x qi
x + qi
.
2qi
(x − 1)
x + qi
−
1 + qi
=
Do đó p
(x + qi) (1 + qi)
≤ 0.
p
k=
Y
j+1
Với k = 1, . . . , j, ta có
x qk
.x + qk . ≤
k=
Y
j+1
1 qk
1 + qk
.
Ta lại có
x − qj−(k−1)
j−(k−1)
qj−(k−1) x
= .
qj−(k−1) + x
qj−(k−1) − x 1 − qk
2 qj+1
− x
V y nên
qj−(k−1) − x
−
1 + qk
=
(1 + qk) x + qj−(k−1)
≤ 0.
Tà đó ta được
hay
k
Y
=1
k
Y
=1
x qk
.x + qk . ≤
x qk
.x + qk . ≤
k
Y
=1
k
Y
=1
1 qk
1 + qk
.
1 − qk
1 + qk
|f(x)| ≤
k
Y
=1
1 − qk
.1 + qk . .
12
Viết đề tài giá sinh viên – ZALO:0973.287.149-TEAMLUANVAN.COM
−
−
−
a3 4
≥ a3
.
1.3 Định lj Viète
Định lj 1.2 (Định lý Viète (xem [6])). Giả sả f(x) ∈ R[x] có dạng
f(x) = anxn
+ an−1xn−1
+ · · · + a1x + a0, deg f = n.
Khi đó, neu f có n nghi m x1, x2, . . . , xn (có the không phân bi t) thì
x1 + x2 + · · · + xn =
an−1
,
an
x1x2 + x1x3 + · · · + xn−1xn =
an−2
,
an
x1x2x3 + x1x2x4 + · · · + xn−2xn−1xn =
an−3
,
an
. . .
x1x2
ChGng minh.
. . . xn
= ( 1)n a0
.
an
Do f(x) là đa thác b c n và có n nghi m x1, x2, . . . , xn nên
f(x) = an(x − x1)(x − x2)........(x − xn)
= anxn
− an(x1 + x2 + · · · + xn)xn−1
+ · · · + (−1)n
anx1x2 ....... xn.
Dựa vào so sánh h so của f(x) theo cách khai trien trên và
f(x) = anxn
+ an−1xn−1
+ ····+ a1x + a0,
ta được đieu can cháng minh.
Bài toán 1.12. Cho đa thác f(x) = x4
+a1x3
+a2x2
+a3x+a4 có 4 nghi m
không âm (có the không phân bi t). Cháng minh
4 4
Lài giai.
13
Viết đề tài giá sinh viên – ZALO:0973.287.149-TEAMLUANVAN.COM
1 2 3 4
1 2 3 4
n n
Goi 4 nghi m không âm của đa thác là x1, x2, x3, x4. Theo định lý Vi ét
ta có
x1x2x3x4 = a4 ≥ 0,
x1x2x3 + x1x2x4 + x1x3x4 + x2x3x4 = −a3 ≥ 0.
Áp dụng bat đȁng thác AM - GM cho 4 so không âm ta có
−a3 = x1x2x3 + x1x2x4 + x1x3x4 + x2x3x4 ≥ 4
q
4 x3
x3
x3
x3
.
Suy ra
−a3
≥
q
4 x3
x3
x3
x3
,
−a3
≥
q
4 a3
⇔
a3 4
≥ a3
.
4 4
4 4
Đȁng thác xảy ra khi và chỉ khi
x1x2x3 = x1x2x4 = x1x3x4 = x2x3x4
⇔ x1 = x2 = x3 = x4.
Có the mở r®ng Bài toán 1.12 cho đa thác b c n như sau
Bài toán 1.13. Cho đa thác b c n
f(x) = a0xn
+ a1xn−1
+ · · · + an−1x + an(a0 =
/ 0)
có n nghi m không âm (có the không phân bi t). Cháng minh
an−1 n
≥ an−1
.
Cách giải bài toán này cũng tương tự như trên.
1.4 M t so bat đang thfíc cơ bản
Trong mục này sě trình bày bat đȁng thác AM - GM, đây là m®t bat đȁng
thác quen thu®c. Tà bat đȁng thác AM - MG ta cháng minh được m®t so
bat đȁng thác dưới đây. Các bat đȁng thác này được áp dụng trong phan
sau của lu n văn.
4
14
Viết đề tài giá sinh viên – ZALO:0973.287.149-TEAMLUANVAN.COM
Σ Y !
Y Y !
n
n
Y 1
n
Y x
n
Y 1
n
Y x
n
1 Σ 1 1 Σ x
i=1
Bat đang thfíc AM - GM (xem[3])
n
1
n
i=1
xi ≥
1
n n
xi
i=1
(1.6)
với n là so nguyên dương, xi là so thực không âm, ∀i = 1, n.
Bat đang thfíc 1.4.1 (xem[3])
n
i=1
(1 + xi)
1
n
≥ 1 +
1
n n
xi
i=1
(1.7)
với n là sô nguyên dương, xi là so thực dương, ∀i = 1, n.
ChGng minh.
Bat đȁng thác (1.7) tương đương với
n
!1
n
!1
i=1
1 + xi i=1
1 + xi
Áp dụng bat đȁng thác AM - GM ta có
n !1 n !1
n n
i=1
1 + xi
i=1
1 + xi n i=1 1 + xi n i=1 1 + xi
=
1 Σ 1
+
xi
= 1.
Đây là đieu can cháng minh.
Bat đang thfíc 1.4.2 (xem[3])
n
i=1
1 + xi 1 + xi
Σ q
‚
. Σ
n !2
Σ
n
!2
x2
+ y2
≥ , xi + yi
(1.8)
với n là sô nguyên dương, xi, yi là so thực dương, ∀i = 1, n.
i=1
i
i
i
+
≤
)
i
+
.
)
i
+
1 ≥
!
i=1
15
Viết đề tài giá sinh viên – ZALO:0973.287.149-TEAMLUANVAN.COM
2
k
k
‚
.
, Σ
1 1 2 2
1 1 2 2
i=1
! !
Σ
2 2 2 2 2
i=1
ChGng minh.
Ta cháng minh bat đȁng thác (1.8) đúng với n = 2, tác là
q
x2
+ y2
+
q
x2
+ y2
≥
q
(x1 + x2)2 + (y1 + y2)2. (1.9)
Th t v y, bình phương hai ve của (1.9), ta có
q
(x2
+ y2
)(x2
+ y2
) ≥ x1x2 + y1y2
⇔(x1 + y1)(x2 + y2) ≥ (x1x2 + y1y2)
⇔(x1y2 − x2y1) ≥ 0.
V y (1.9) đúng.
Giả sả bat đȁng thác (1.8) đúng với n = k, tác là
Σ q
‚
. Σ
k !2
Σ
k
!2
x2
+ y2
≥ , xi + yi .
Ta can cháng minh bat đȁng thác đúng với n = k + 1.
Xét
Σ
k+1 q Σ q q
Suy ra
i=1
i i i i
i=1
k+1 k+1
Σ
k+1 q ‚
. Σ
k
!2
Σ
k
!2
q
x2
+ y2
≥ , xi + yi + x2
+ y2
i=1
i i
i=1
k+1 2
≥ xi +
i=1
i=1
k+1
i=1
2
yi .
k+1 k+1
V y bat đȁng thác được cháng minh.
Bat đang thfíc 1.4.3 (xem[3])
Σ
n Σ
n
xiyi
xi + yi
≤
xi
i=1
Σ
n
yi
i=1
Σ
n
(1.10)
i=1
i=1
n
.
i=1
i
i
Σ
i=1
x2
+ y2
= x2
+ y2
+ x2
+ y2
xi + yi
16
Viết đề tài giá sinh viên – ZALO:0973.287.149-TEAMLUANVAN.COM
n
y
Σ
n 2
Σ Σ
!2
√ y
Σ
!
Σ n
n
2
1
2
≥
Σ
yi
Σ i
⇔ ≥
!
Σ
n i
với n là sô nguyên dương, xi, yi là so thực dương,∀i = 1, n.
ChGng minh.
Bat đȁng thác (1.10) tương đương với
n
n xi
Σ
n
Σ xiyi
− y ≤ i=1 i=1
−
Σ
y
i=1 xi + yi
Σ
xi +
Σ
yi
i=1
i=1
2 yi
i=1
.
i=1
Ta có
i=1
xi + yi
n
i=1
xi +
n
yi
i=1
Σ
n
!2
Σ
n
yi =
i=1 i=1
√
xi + yi
xi + yi
n
≤
i=1
2
i
xi + yi
n
i=1
xi +
Σ
i=1
yi
!
.
Ta có đieu can cháng minh.
Bat đang thfíc 1.4.4 (xem[3])
1 Σ
xm
≥ 1
n m
xi (1.11)
n i
i=1
n
i=1
với m, n là so nguyên dương, xi là so thực không âm, ∀i = 1, n.
ChGng minh.
Ta cháng minh bat đȁng thác (1.11) bang phương pháp qui nạp theo n.
Với n = 2 ta có (1.11) là
xm
+ xm
x1 + x2
m
2
n
n
i
y
.
i
17
Viết đề tài giá sinh viên – ZALO:0973.287.149-TEAMLUANVAN.COM
2
1 2 2
2
Σ
Σ
Σ
1
i
Σ
Σ
x
Σ Σ
1
2
≥ 1 2
1 2
1
2
≥ 1 2
1 2
1
i
!
2k i=1
Ta có
xm
+ xm
x1 + x2 xm−1
+ xm−1
xm
+ xm
− x1xm−1
− x2xm
− 21
4
⇔ xm−1
− xm−1
(x1 − x2) ≥ 0.
Đieu này đúng với moi so thực dương x1, x2 và m nguyên dương. Tà đó, ta
suy ra
xm
+ xm
x1 + x2 xm−1
+ xm−1
x1 + x2
2
xm−2
+ xm−2
≥ · · · ≥
x1 + x2
m
Giả sả (1.11) đúng với n = k, ta sě cháng minh (1.11) đúng với n = 2k.
Th t v y
1 Σ
xm
= 1 1 Σ
k 2k
xm
+ xm
2k i
i=1
2 k i
i=1
k i
i=k+1
1
"
1 Σ
k
!m
1
2k
!m#
1 Σ
2k
!m
Bat đȁng thác sě được cháng minh khi ta cháng minh được (1.11) đúng với
n = k + 1 thì sě đúng với n = k.
Th t v y, ta can cháng minh
hay
k
k
i=1
xm
≥
1
k
xi
k
i=1
k
P = m
i=1
1
k
xi
k
i=1
≥ (k + 1)
1
k
k
i=1
m
xi .
xi
k
k
2
2
2
2
2
2
2
2
≥ 0
2k
≥
≥
i=1
xi +
i=k+1
≥ xi .
⇔
.
!
!m
!
+
!m
18
Viết đề tài giá sinh viên – ZALO:0973.287.149-TEAMLUANVAN.COM
xi +
k
xi
Áp dụng giả thiet quy nạp, ta có
"
1 Σ
k
1 Σ !#m
1 Σ
k
!m
V y bat đȁng thác (1.11) đã được cháng minh.
i=1
k
xi
i=1
i=1
k + 1
k
P ≥ (k + 1) ≥ (k + 1) .
19
Viết đề tài giá sinh viên – ZALO:0973.287.149-TEAMLUANVAN.COM
√
Chương 2
Các dạng toán ve đa thfíc và phân
thfíc v i h so nguyên
2.1 Các dạng toán ve đa thfíc m t bien v i h so
nguyên và h thfíc Viète
Trong mục này, sě trình bày các bài toán ước lượng m®t so đại lượng liên
quan đen đa thác h so nguyên như: ước lượng h so, ước lượng nghi m, ước
lượng mien giá trị của đa thác,. . .
Bài toán 2.1. Cho tam thác b c hai f(x) = ax2
+ 1998x + c với a, c ∈ Z,
|a| < 2000, |c| < 2000 và f(x) có hai nghi m phân bi t x1, x2. Cháng minh
1
Lài giai.
|x1 − x2| ≥ .
998
f(x) có bi t thác ∆
′
= 9992
− ac, do f(x) có hai nghi m phân bi t x1, x2
nên ∆
′
> 0, ∆
′
∈ Z và
|x1 − x2| =
2 ∆′
.
|a|
Neu ∆
′
≥ 2 thì
|x1 − x2| =
2
√
∆′
>
|a|
2
√
∆′
2000
≥ 2
√
2
>
2000
1
998
20
Viết đề tài giá sinh viên – ZALO:0973.287.149-TEAMLUANVAN.COM
Σ
1
n
xi
xi a0
a0
i=1
(do |a| < 2000).
Neu ∆
′
= 1 thì
ac = 9992
− 1 = 998.1000 = 24
.53
.449.
Do đó a là m®t ước so của 24
.53
.449, lại có 22
.449 = 1996 là m®t ước so của
24
.53
.449 và |a| < 2000 nên a ≤ 1996.
Tà đây ta suy ra
|x1 − x2| =
2
√
∆′
=
|a|
2
|a|
≥
2
=
1996
1
.
998
Bài toán 2.2. Cho đa thác P (x) = xn
+ an−1xn−1
+ · · · + a1x + a0 và
|ai| = 1(i = 0, 1, . . . , n − 1). Cháng minh rang neu P(x) có n nghi m thực
thì n ≤ 3.
Lài giai.
Giả sả P(x) có n nghi m là x1, x2, . . . , xn. Áp dụng định lý Viète, ta có
n
xi = −an−1,
i=1n
Suy ra
Σ Σ
n
!2
i,j=
Σ
1,i<j
Σ
xixj = an−2.
Do x1x2 . . . xn = (−1)n
a0 và |a0| = 1 nên xi =
/ 0 ∀i.
De thay
xi
là nghi m của đa thác
Q(x) = a0xn
+ a1xn−1
+ · · · + an−1x + 1.
Áp dụng Định lý Viète, ta có
n
Σ 1
=
−a1
,
Σ 1
=
a2
.
i,j=1,i<j
xixj
i,j=1,i<j
i=1
i=1
n n
xi
2
= — 2 xixj = 3 (do|an−1| = 1, |an−2| = 1). (2.1)
21
Viết đề tài giá sinh viên – ZALO:0973.287.149-TEAMLUANVAN.COM
!
2
Σ
Σ Σ
Suy ra
1
x 2
=
Σ
n 1
2
x
− 2
1
x x
= 3, ( do |a1| = 1, |a2| = 1). (2.2)
i=1 i i=1 i i,j=1,i<j i j
Tà (2.1) và (2.2), ket hợp với bat đȁng thác Cauchy, ta được
n n
9 =
Σ
x 2
Σ 1
≥ n .
Suy ra n ≤ 3.
i=1
i
i=1
xi
2
Bài toán 2.3 (xem[5]). Cho đa thác P (x) có b c m > 0 và có các h
so nguyên. Goi n là so các nghi m nguyên phân bi t của hai phương trình
P(x) = 1 và P(x) = −1. Cháng minh rang n ≤ m + 2.
Lài giai.
Xét hai đa thác A(x) và B(x) với các h so nguyên
A(x) = anxn
+ an−1xn−1
+ · · · + a1x + a0,
B(x) = anxn
+ an−1xn−1
+ · · · + a1x + a0 + 2.
Goi r, s là các nghi m nguyên tương áng của hai đa thác trên, tác là A(r) =
0, B(s) = 0. Ta có
n
A(r) − B(s) = ai(ri
− si
) − 2 = 0.
i=1
Tà đây suy ra neu r − s /= 0 thì 2 chia het cho r − s, tác là chỉ xảy ra
m®t trong ba trường hợp r và s hơn kém nhau 0, 1 ho c 2 đơn vị. Ta sě áp
dụng nh n xét trên đe giải bài toán.
Giả sả r là nghi m nguyên nhỏ nhat trong tat cả các nghi m nguyên của
hai phương trình P (x) = 1 và P (x) = −1. Do đa thác P (x) có b c m nên
moi phương trình trên không có quá m nghi m.
Theo nh n xét trên neu r là nghi m của phương trình này, s là nghi m của
phương trình kia và r là nghi m nguyên nhỏ nhat trong tat cả các nghi m
nguyên của hai phương trình thì s = r, s = r + 1 ho c s = r + 2.
n n
22
Viết đề tài giá sinh viên – ZALO:0973.287.149-TEAMLUANVAN.COM
2
Do v y neu k là so nghi m của phương trình thá nhat thì so nghi m của
cả hai phương trình nhieu nhat là k + 2 mà k ≤ m nên ta suy ra n ≤ m + 2.
Bài toán 2.4 (xem[5]). Cho f (x) là đa thác b c n có các h so bang ±1.
Biet rang đa thác có x = 1 là nghi m b®i cap m với m ≥ 2k
, k ≥ 2 và k
nguyên. Cháng minh n ≥ 2k+1
− 1.
Lài giai.
Goi f(x) là đa thác với các h so theo modulo 2 của f(x). Vì f(x) có các
h so là 1 và -1 nên
f(x) = xn
+ xn−1
+ · · · + x + 1.
Ta có f (x) = (x − 1)2k
g(x), trong đó n ≥ 2k
và g(x) là đa thác h so
nguyên.
De dàng cháng minh được Ci
k ≡ 0(mod2), 1 ≤ i ≤ 2k
− 1 nên
f(x) = xn
+ xn−1
+ · · · + x + 1 = (x2k
+ 1)g(x). (2.3)
Giả sả g(x) có b c không quá 2k
− 2. Ta có h so của x2k−1
ở ve phải của
(2.3) là 0. Đieu này mâu thuan vì h so của x2k−1
ở ve trái của (2.3) là 1. Do
đó b c của g(x) không nhỏ hơn 2k
− 1. V y n ≥ 2k
+ 2k
− 1 = 2k+1
− 1.
Bài toán 2.5. Cho đa thác với h so nguyên
P(x) = akxk
+ · · · + a1x + a0.
Ta ký hi u so các h so lẻ của đa thác là h(P ). Với i = 0, 1, 2, . . . ta đ t
Qi(x) = (1 + x)i
. Cháng minh rang neu i1, i2, . . . , in là các so nguyên thỏa
mãn đieu ki n 0 ≤ i1 < i2 < · · · < in thì ta có
h(Qi1 + Qi2 + · · · + Qin ) ≥ h(Qi1).
Lài giai.
Trước het ta có nh n xét: Neu i là m®t b®i so của 2 thì tat cả các h so
của Qi đeu chȁn, ngoại trà h so đau tiên và h so cuoi cùng.
23
Viết đề tài giá sinh viên – ZALO:0973.287.149-TEAMLUANVAN.COM
i
1
1
mB
i
Th t v y giả sả i = 2k
, k nguyên dương. Ta có h so của Qi(x) = (1+ x)i
là Cr
=
i!
r!(i − r)!
Giả sả 0 < r < i, r = 2l
r , r là so lẻ. Lúc đó Cr
=
i
Cr−1
nhưng Cr−1
là
1 1 i
r i−1 i−1
m®t so nguyên và l < k nên suy ra Cr
là so chȁn.
Đ t
Q = Qi1 + Qi2 + · · · + Qin.
Ta sě cháng minh bat đȁng thác ở đe bài bang quy nạp theo in.
Với in = 0 ho c in = 1 de thay bat đȁng thác đúng.
Giả sả bat đȁng thác đúng với các giá trị bé hơn in. Ta goi m là m®t lũy
thàa của 2 sao cho m ≤ in < 2m. Xét các trường hợp i1 ≥ m và i1 < m
Trường hợp 1 i1 ≥ m. Lúc đó ta có
Qi = (1 + x)m
A,
Q = (1 + x)m
B
với A, B là các đa thác có b c nhỏ hơn m.
Theo giả thiet qui nạp ta có h(A) ≤ h(B) nên
h(Qi ) = h((1 + x)m
A) = h(A + xm
A) = 2h(A)
≤ 2h(B) = h(B + xmB) = h((1 + x) )
= h(Q).
V y bat đȁng thác đúng ở trường hợp 1.
Trường hợp 2 i1 < m. Lúc đó ta lay r sao cho ir < m, ir+1 > m
Đ t
A = Qi1 + Qi2 + · · · + Qir ,
B(1 + x)m
= Qir+1 + Qir+2 + · · · + Qin.
Suy ra A và B có b c nhỏ hơn m. Lúc đó
h(Q) = h(A + (1 + x)m
B) = h(A + B + xm
B) = h(A + B) + h(B).
24
Viết đề tài giá sinh viên – ZALO:0973.287.149-TEAMLUANVAN.COM
Vì h so của A là lẻ khi và chỉ khi m®t trong hai h so tương áng của A − B
và của B lẻ nên
h(A − B) ≥ h(A − B + B) = h(A).
Vì các h so tương áng của A−B và của A+B ho c bang nhau ho c cùng
tính chȁn lẻ nên h(A − B) = h(A + B). Do đó h(A + B) + h(B) ≥ h(A).
Ta cũng có h(A) ≥ h(Qij ) theo giả thiet quy nạp. V y bat đȁng thác cũng
đúng trong trường hợp 2.
Bài toán 2.6 (xem[5]). Cho f (x) là đa thác b c n với h so nguyên và
g(x) = f2
(x) − p2
, p là so nguyên to. Khi đó so nghi m nguyên phân bi t
của g(x) không vượt quá max{n, min{8, 2n}}.
Lài giai.
Goi
A = {x ∈ Z|f(x) = p} ,
B = {x ∈ Z|f(x) = −p} ,
C = {x ∈ Z|g(x) = 0} .
Ta có A ∪ B = C, A ∩ B = φ nên |C| = |A| + |B|.
Giả sả |A| ≥ 5. Ta cháng minh B = φ.
Th t v y, neu B /= φ. Ta lay 5 phan tả xi ∈ A, (i = 1, 5) và b ∈ B ta có
f(x) − p = (x − x1) . . . (x − x5)Q(x),
ở đó Q(x) là đa thác h so nguyên. Do đó
f(b) − p = (b − x1) . . . (b − x5)Q(b),
mà f(b) + p = 0, suy ra
−2p = (b − x1) . . . (b − x5)Q(b),
25
Viết đề tài giá sinh viên – ZALO:0973.287.149-TEAMLUANVAN.COM
tác 2p được phân tích thành tích của ít nhat 5 so nguyên khác nhau.
Mâu thuan vì 2 = 2.1 = (−1)(−2), p = 1.p = (−1).(−p). Do đó B = φ.
V y |C| = |A| ≤ n.
Tương tự neu |B| ≥ 5 thì
|C| ≤ n. (2.4)
Xét trường hợp |B| ≤ 4 và |A| ≤ 4 ta có
|A| ≤ min{n, 4},
|B| ≤ min{n, 4}.
V y
|C| ≤ 2 min{n, 4} = min{2n, 8}. (2.5)
Tà (2.4) và (2.5) ta có đieu can cháng minh.
Bài toán 2.7. Cho các so nguyên a, b, c, biet a > 0 và đa thác ax2
+ bx + c
có hai nghi m khác nhau trong khoảng (0; 1). Cháng minh a ≥ 5. Tìm ít
nhat m®t c p so b, c đe a = 5.
Lài giai.
Vì đa thác P (x) = ax2
+ bx + c (a > 0) có hai nghi m khác nhau x1, x2
trong khoảng (0; 1) nên theo Định lý Viète ta có
0 <
Do đó a > c > 0, b < 0.
c
< 1,
a
b
< 0.
a
M t khác P (1) = a + b + c > 0 (do P (x) = ax2
+ bx + c có hai nghi m
khác nhau trong khoảng (0; 1) mà P (0) = c > 0). Suy ra a + c > −b > 0,
hay a2
+ 2ac + c2
> b2
. Do đó
(a − c)2
> b2
− 4ac > 0.
26
Viết đề tài giá sinh viên – ZALO:0973.287.149-TEAMLUANVAN.COM
Q Q
Q
P(k)
j=0,j=
/ k
p−2 P(k)
j=0,j=
/ k
.
p−k
p−1
M t khác đa thác P (x) = ax2
+ bx + c (a > 0) có hai nghi m khác nhau
x1, x2 trong khoảng (0; 1) nên ∆ = b2
− 4ac là so nguyên dương, suy ra
a − c ≥ 2.
Giả sả a ≤ 4 thì xảy ra ba trường hợp của c p so nguyên a, c: a = 4 và
c = 2, a = 3 và c = 1, a = 4 và c = 1.
Xảy ra ba trường hợp tương áng
4 > b2
− 32 > 0 ⇔ 36 > b2
> 32 (loại);
4 > b2
− 12 > 0 ⇔ 16 > b2
> 12 (loại);
9 > b2
− 16 > 0 ⇔ 25 > b2
> 16 (loại).
V y a ≥ 5.
Với a = 5, phương trình 5x2
− 5x + 1 = 0 có hai nghi m thu®c khoảng
(0; 1). Do đó b = −5, c = 1 là m®t c p so b, c thỏa mãn.
Bài toán 2.8 (xem[5]). Cho p là m®t so nguyên to và P(x) ∈ Z[x] thỏa mãn
các đieu ki n
i) P(0) = 0;
ii) P(n) ho c chia het cho p ho c chia cho p có so dư bang 1 với moi n ∈ N∗
.
Cháng minh s ≥ p − 1.
Lài giai.
Giả sả ngược lại s < p − 2.
Khi đó theo công thác n®i suy Lagrange thì
p−2
p−2
(x − j) p−2
p−2
(x − j)
k=0
j=0
Q
,j/=
k
(k − j) k=0 k!(−1) (p − 2 − k)!
Cho x = p − 1 ta thu được
p−2
p−2
(p − 1 − j) p−2
.
P (k)(−1)p−k
Ck
=
P (k)
P(p − 1) =
Σ Σ
Σ Σ
P(x) = =
j=0,j k
k=0 k!(−1)p−k(p − 2 − k)! k=0
27
Viết đề tài giá sinh viên – ZALO:0973.287.149-TEAMLUANVAN.COM
Σ
Σ
Theo giả thiet p nguyên to nên Ck
p−1 ≡ (−1)k
(mod p). Do v y
p−2
P(p − 1) ≡ (−1)p
P (k) (mod p).
k=0
Neu p = 2 thì 1 = P(1) ≡ (−1)2
P(0) (mod p), vô lý.
Neu p ≥ 3 thì p lẻ và vì v y
p−2
P(p − 1) ≡ − P (k) (mod p).
k=0
Tà đây suy ra
s = P(0) + P(1) + · · · + P(p − 2) + P(p − 1) ≡ 0 (mod p).
M t khác do giả thiet ii) thì s ≡ k (mod p) với 0 ≤ n ≤ p−1 và P (n) ≡ 1
(mod p) nên 1 ≤ s ≤ p − 1, tác là s /= 0 (mod p), mâu thuan với ket lu n
trên. V y đieu giả sả s < p − 2 là sai. Ta được đieu can cháng minh.
Bài toán 2.9. Cho P (x) là m®t đa thác không bang hang so, với h so
nguyên. Goi n(P ) là so nhǎng so nguyên k khác nhau sao cho [P (k)]2
= 1,
hãy cháng minh rang n(P ) − deg P ≤ 2 với deg P là b c của đa thác P (x).
Lài giai.
So k nói trong đau bài là so nghi m của hai phương trình
P (x) = 1, (2.6)
P(x) = −1. (2.7)
Neu phương trình (2.7) không có nghi m nguyên thì tat cả các so k đeu là
nghi m của (2.6), do đó n(P) không the vượt quá deg P, hay
n(P) − deg P ≤ 0.
Neu phương trình (2.6) không có nghi m nguyên cũng được xét tương tự.
Trường hợp cả hai phương trình (2.6) và (2.7) đeu có nghi m nguyên.
28
Viết đề tài giá sinh viên – ZALO:0973.287.149-TEAMLUANVAN.COM
Goi x1, x2, . . . xr là tat cả các nghi m nguyên khác nhau của (2.6) và
y1, y2, . . . ys là tat cả các nghi m nguyên khác nhau của (2.7).
The thì r + s = n(P) và
P(x) + 1 = (x − y1) . . . (x − ys)g(x) (2.8)
với g(x) là đa thác h so nguyên.
Trong (2.8) cho x = x1 ta được
2 = P (x1) + 1 = (x1 − y1) . . . (x1 − ys)g(x1).
Các hi u x1 − y1, . . . , x1 − ys là s so nguyên khác nhau và so 2 chỉ có các ước
là −1, 1, −2, 2. Tà đó suy ra s ≤ 3.
Tương tự ta thay rang r ≤ 3, do đó n(P) = r + n ≤ 6.
Ta hãy phân bi t các trường hợp
a) deg P ≥ 4. Khi đó n(P) − deg P ≤ 6 − deg P ≤ 2.
b) deg P ≤ 2. Vì n(P) là so các nghi m nguyên khác nhau của phương
trình [P(x)]2
= 1 nên
Do đó
c) deg P = 3.
n(P ) ≤ deg P 2
= 2 deg P
n(P) − deg P ≤ deg P ≤ 2.
Neu n(P) ≤ 5 thì n(P) − deg P ≤ 5 − 3 = 2.
Đe ket thúc cháng minh, ta hãy cháng tỏ rang không the xảy ra trường
hợp deg P = 3 với n(P ) = 6. Th t v y, neu trường hợp này xảy ra thì ta
phải có r = s = 3 và (2.8) trở thành
P(x) + 1 = A(x − y1)(x − y2)(x − y3) (2.9)
với A là so nguyên. Có the coi rang y1 < y2 < y3. Cho x = x1 ta được
2 = P (x1) + 1 = A(x1 − y1)(x1 − y2)(x1 − y3).
29
Viết đề tài giá sinh viên – ZALO:0973.287.149-TEAMLUANVAN.COM
Σ
Vì x1 − y1, x1 − y2, x1 − y3 là ba so nguyên khác nhau nên A /= ±2 tác là
A = ±1.
Neu A = 1, thì x1 − y1 > x1 − y2 > x1 − y3 và tích của ba so này bang 2
nên ba so ay phải là 1, −1, −2, đ c bi t x1 − y1 = 1.
Trong (2.9) lại cho x = x2 ta được
2 = (x2 − y1)(x2 − y2)(x2 − y3).
Tà đây suy ra x2 − y1 = 1. Đieu này vô lý vì x1 /= x2.
Trường hợp A = −1 cũng khảo sát tương tự.
V y ta được đieu can cháng minh.
Nh n xét 2.1. Bài toán (2.9) có the mở r®ng như sau: Cho P (x) là m®t đa
thác không bang hang so, với h so nguyên. Goi n(P ) là so nhǎng so nguyên
k khác nhau sao cho [P (k)]2
= r2
, hãy cháng minh rang n(P ) − deg P ≤ 2r
với deg P là b c của đa thác P (x). Đ c bi t, khi r = 0 ta có ket quả quen
thu®c: So nghi m của đa thác không vượt quá b c của đa thác đó.
Bài toán 2.10 (xem [4]). Cho f (x) ∈ Z[x], deg f = m ≥ 1 và f (x) có ít
nhat m®t nghi m hǎu tỉ. Giả sả phương trình |f(x)| = 1 có các nghi m tự
nhiên a1, a2, . . . , an, (a1 < a2 < · · · < an). Cháng minh rang
ln(ai
i<j
— aj ) <
m(m − 1)
.
2
Lài giai.
p
Goi (p, q ∈ Z, (p, q) = 1) là nghi m hǎu tỉ của P(x).
q
Theo giả thiet, ta có |p−qai| = 1 với moi i ∈ {1, 2, . . . , m}. Suy ra p−qai ∈
{1, −1} và m ≤ 2, do v y m = 2.
Do |f(a1)| = |f(a2)| = 1 nên suy ra
|f(a1)| = |f(a2)| ≤ 2,
|a2 − a1| ≤ |f(a2) − f(a1)| ≤ 2.
30
Viết đề tài giá sinh viên – ZALO:0973.287.149-TEAMLUANVAN.COM
Σ
≤
Σ
−
V y ta có
ln(ai
i>j
— aj) = ln(a2 — a1
) ln 2 < 1 =
2(2 − 1)
2
=
m(m − 1)
.
2
2.2 Đa thfíc v i các h so nguyên và đong dư thfíc
Định nghĩa 2.1 (xem [1]). Cho đa thác f (x) với các h so nguyên và so
nguyên dương m ≥ 2. Ta nói rang phương trình đong dư f (x) ≡ 0 (mod m)
có nghi m x0 ∈ Z neu f(x0) ≡ 0 (mod m).
Neu x0 ∈ Z là m®t nghi m của phương trình f(x) ≡ 0 (mod m) và t là
m®t so nguyên bat kỳ thì f(x0 + tm) ≡ f(x0) ≡ 0 (mod m).
Định lj 2.1 (xem [1]). Cho các so nguyên a và m, m ≥ 2, (a, m) = 1.
Khi đó phương trình ax ≡ b (mod m), b ∈ Z có nghi m duy nhat x0 ∈ Z
mà 0 ≤ x0 ≤ m − 1. Moi nghi m khác của phương trình này đeu có dạng
xt = x0 + mt, t ∈ Z.
ChGng minh.
Neu k, l ∈ Z, k =
/ l, 0 ≤ k ≤ m − 1, 0 ≤ l ≤ m − 1 thì
ak al (mod m).
Đieu này có nghĩa là bieu thác as − b, s = 0, 1, . . . , m − 1 cho m so dư khác
nhau khi chia cho m. V y ton tại duy nhat x0 ∈ Z, 0 ≤ x0 ≤ m − 1 sao cho
ax0 ≡ b (mod m).
Neu xt ∈ Z là m®t nghi m của phương trình đong dư ax ≡ b (mod m)
thì axt ≡ b (mod m). Suy ra a(xt − x0) chia het cho m, nhưng (a, m) = 1
v y xt − x0 chia het cho m. Suy ra xt = x0 + mt, t ∈ Z.
Định lj 2.2 (Công thác Taylor). Cho đa thác f (x) b c n, n ≥ 1 với các h
so thực và x0 ∈ R. Khi đó
n (k)
f(x) = f(x0 ) +
f (x0)
(x x
k! 0
k=1
)k
.
31
Viết đề tài giá sinh viên – ZALO:0973.287.149-TEAMLUANVAN.COM
Σ
−
i
i
1 2 r i
1 2 r
1 2 r i
ChGng minh.
Ton tại các hang so b0, b1, . . . ., bn sao cho
f(x) = bn(x − x0)n
+ bn−1(x − x0)n−1
+ · · · + b0.
Với 1 ≤ k ≤ n, ta có
fk
(x) = k!bk+[(k+1) . . . 2(x−x0)bk+1]+· · ·+[n(n−1) · · · (n−k+1)(x−x0)n−k
bn].
Suy ra f (k)
(x0) = k!bk, do đó bk =
f (k)
(x0)
.
k!
M t khác f(x0) = b0. V y
f(x) = f(x0
n
f(k)
(x )
) + 0
(x x0
k!
k=1
)k
.
Định lj 2.3 (xem [1]). Cho đa thác f(x) với các h so nguyên và m®t so
nguyên to p. Neu phương trình đong dư f(x) ≡ 0 (mod p) có đúng r nghi m
nguyên phân bi t x(1)
, x(1)
, . . . , x(1)
thu®c đoạn [1; p] sao cho f′(x(1)
) /= 0
(mod p), (1 ≤ i ≤ r) thì phương trình đong dư f(x) ≡ 0 (mod pk
) có đúng r
nghi m nguyên phân bi t thu®c đoạn [1; pk
] với moi k ≥ 1 : x(k)
, x(k)
, . . . , x(k)
và đoi với các nghi m này ta có f′(x(k)
) /= 0 (mod p) (1 ≤ i ≤ r).
ChGng minh.
Ta cháng minh khȁng định bang phương pháp quy nạp toán hoc theo k.
Với k = 1 thì khȁng định đúng.
Giả sả khȁng định đúng với k ≥ 1. Đieu đó có nghĩa là trong đoạn [1; pk
]
thì phương trình đong dư f(x) ≡ 0 (mod pk
) có đúng r nghi m nguyên phân
bi t x(k)
, x(k)
, . . . , x(k)
, đong thời f′(x(k)
) 0 (mod p) với 1 ≤ i ≤ r.
Giả sả x0 ∈ Z, x0 ∈ [1; pk+1
] là m®t nghi m của phương trình đong dư
f (x) ≡ 0 (mod pk+1
). Khi đó f (x0) ≡ 0 (mod pk+1
). Suy ra f (x0) ≡ 0
(mod pk
). Ton tại duy nhat i ∈ [1; r], t ∈ Z, t ∈ [0; p − 1] sao cho x0 =
x(k)
+ pk
t.
32
Viết đề tài giá sinh viên – ZALO:0973.287.149-TEAMLUANVAN.COM
i
i i
i
pk i
≡ x
i
i i
i i i i
i i
j!
Giả sả x = x(k)
+ pk
t (1 ≤ i ≤ r, t ∈ Z, t ∈ [0, p − 1]) thì x ∈ Z. Theo
công thác Taylor ta có
(k) (k) f ′′(x(k)
) f (n)
(x
(k)
)
f(x) = f(x ) + f′(x )pk
t + i
(pk
t)2
+ · · · + i
(pk
t)n
,
2! n!
trong đó n là b c của f(x).
f (j)
(x
(k)
)
Ta có i
∈ Z và jk ≥ k + 1, i ≥ 2.
Phương trình f(x) ≡ 0 (mod pk+1
) tương đương với
f(xk
) + f′(xk
)pk
t ≡ 0 (mod pk+1
)
hay
f(x(k)
) (k)
(chú ý rang
f(x(k)
)
pk
i
+ f′(x )t ≡ 0 (mod p)
∈ Z).
Đ t x(k+1)
= x(k)
+ pk
ti thì x(k+1)
∈ [1; pk+1
], x(k+1)
∈ Z và f(x(k+1)
) ≡ 0
i i i i i
(mod pk+1
).
M t khác, x(k+1) (k)
(mod p). Suy ra f′(x(k+1)
) ≡ f′(x(k)
) (mod p).
Suy ra f′(x(k+1)
) /= 0 (mod p) vì f′(x(k)
/= 0 (mod p).
V y phương trình đong dư f(x) ≡ 0 (mod pk+1
) có đúng r nghi m
nguyên phân bi t trong đoạn [1; pk+1
] : x(k+1)
, x(k+1)
, . . . , x(k+1)
, đong thời
1 2 r
f ′
(x(k+1)
) 0 (mod p), (1 ≤ i ≤ r). Như v y khȁng định cũng đúng với
k + 1. Theo nguyên lý quy nạp toán hoc thì khȁng định đúng với moi k ≥ 1.
Định lý 2.3 đã được cháng minh. Áp dụng định lý này có the giải được
bài toán sau.
Bài toán 2.11. Cho đa thác P (x) = x3
+ 153x2
− 111x + 38.
1. Hỏi trong đoạn [1; 32000
] có ít nhat bao nhiêu so nguyên dương a sao cho
P(a) chia het cho 32000
?
2. Hỏi trong đoạn [1; 32000
] có nhieu nhat là bao nhiêu so nguyên dương a
sao cho P (a) chia het cho 32000
?
33
Viết đề tài giá sinh viên – ZALO:0973.287.149-TEAMLUANVAN.COM
ChGng minh.
Giả sả x ∈ Z, 1 ≤ x ≤ 32000
và P(x).32000
. Do 153 ≡ 0 (mod 3), 111 ≡ 0
(mod 3), 38 ≡ 2 (mod 3), suy ra x = 3y + 1, (y ∈ Z, 1 ≤ y ≤ 31999
− 1).
Ta có
P(x) = P(3y + 1) = 27(y3
+ 52y2
+ 22y + 3).
Phương trình P (x).32000
tương đương với y3
+ 52y2
+ 22y + 3.31997
, suy ra
y = 3t + 1 ho c y = 3t, (t ∈ Z, 1 ≤ t ≤ 31998
− 1).
Neu y = 3t + 1 thì
y3
+ 52y2
+ 22y + 3 = 27t3
+ 495t2
+ 387t + 78
không chia het cho 9.
V y y = 3t, suy ra
y3
+ 52y2
+ 22t + 3 = 3(9t2
+ 156t2
+ 22t + 1).
Phương trình P (x).32000
tương đương với 9t3
+ 156t2
+ 22t + 1.31996.
Xét đa thác f(t) = 9t3
+ 156t2
+ 22t + 1.
Với t ∈ Z thì f(t) ≡ 0 (mod 3) hay 22t + 1 ≡ 0 (mod 3). Trong đoạn
[1; 3] thì phương trình đong dư 22t + 1 ≡ 0 (mod 3) có m®t nghi m duy
nhat t = 2. M t khác f′(2) ≡ 22 ≡ 1 (mod 3) suy ra f′(2) /= 0 (mod 3).
Theo Định lý 2.3, trong đoạn [1; 31996
] phương trình đong dư f (t) ≡ 0
(mod 31996
) có m®t nghi m nguyên duy nhat t0.
Với t ∈ Z, t ∈ [1; 32000
] : f (t) ≡ 0 (mod 31996
) khi và chỉ khi ton tại
h ∈ Z, 0 ≤ h ≤ 8 sao cho t = t0 + 31996
h. V y phương trình đong dư
f (t) ≡ 0 (mod 31996
) có đúng chín nghi m nguyên phân bi t trong đoạn
[1; 31998
− 1]. Tà đó suy ra rang trong đoạn [1; 32000
] có đúng chín so nguyên
dương a phân bi t sao cho P(a) chia het cho 32000
.
Định lj 2.4 (xem [1]). Cho đa thác f (x) khác hang so và có các h so
nguyên. Khi đó ton tại vô so so nguyên to p sao cho phương trình đong dư
f(x) ≡ 0 (mod p) có nghi m.
34
Viết đề tài giá sinh viên – ZALO:0973.287.149-TEAMLUANVAN.COM
ChGng minh.
Giả sả f(x) = a0xn
+a1xn−1
+· · ·+an, ai ∈ Z, 0 ≤ i ≤ n, n ≥ 1, a0 0.
Neu an = 0 thì f (x) = xg(x), trong đó g(x) là đa thác với các h so
nguyên, khi đó phương trình đong dư xg(x) ≡ 0 (mod p) có nghi m với moi
so nguyên to p.
Giả sả an 0 và phương trình đong dư xg(x) ≡ 0 (mod p) có nghi m
với moi so nguyên to p1, p2, . . . , pk. Với t ∈ Z, đ t xt = p1p2 . . . pkant. Khi
đó
f(xt) = a0(p1p2 . . . pkant)n
+ · · · + an−1p1p2 . . . pkant + an
= an(p1p2 . . . pkB + 1), B ∈ Z.
Chon t ∈ Z phù hợp, ta suy ra ton tại vô so so nguyên to p sao cho phương
trình đong dư f(x) ≡ 0 (mod p) có nghi m. Đây là đieu can cháng minh.
Áp dụng định lý 2.4 ta có the giải được bài toán sau.
Bài toán 2.12. Cho đa thác f (x) khác hang so và có các h so nguyên. Giả
sả n, k là các so nguyên dương. Cháng minh rang ton tại so nguyên x sao
cho các so f (x), f (x + 1), . . . , f (x + n − 1) đeu có ít nhat k ước nguyên to
phân bi t.
Lài giai.
Theo định lý 2.4, ton tại các so nguyên to p1, p2, . . . , pk, pk+1, . . . , pnk khác
nhau tàng đôi m®t và các so nguyên x1, x2, . . . , xnk sao cho
f(xj) ≡ 0 (mod pj) (1 ≤ j ≤ nk).
Theo định lý Trung Hoa ve so dư, ton tại so nguyên x sao cho
x ≡ xi+mk − m (mod pi+mk) (1 ≤ i ≤ k, 0 ≤ m ≤ n − 1).
Tà đó ta có đieu phải cháng minh.
35
Viết đề tài giá sinh viên – ZALO:0973.287.149-TEAMLUANVAN.COM
2.3 Bat đang thfíc phân thfíc sinh b i tam thfíc b c
hai trên m t khoảng
Cho tam thác b c hai với h so nguyên f(x) = ax2
+ bx + c. Trong mục
này ta xét đieu ki n can và đủ của α, β đe luôn có bat đȁng thác
au2
+ bu + c
2av + b
≥
av2
+ bv + c
2av + b
+ u − v, ∀u, v ∈ (α, β), u v
và xét m®t so áp dụng liên quan.
b
Nh n xét rang đieu ki n đe 2ax+b /= 0 với moi x ∈ (α, β) là −
2a
∈
/ (α, β).
b b
Đieu này tương đương với ho c −
2a
≤ α ho c −
2a
≥ β.
Tà đó suy ra
M nh đe 2.1. Cho tam thác b c hai với h so nguyên f(x) = ax2
+ bx + c.
Đieu ki n can đe có bat đȁng thác
au2
+ bu + c
2av + b
≥
b
av2
+ bv + c
2av + b
+ u − v, ∀u, v ∈ (α, β), u
b
v (2.10)
là ho c −
2a
≤ α ho c −
2a
≥ β.
Tiep theo, ta phát bieu và cháng minh đieu ki n đủ đe có (2.10).
Định lj 2.5. Giả sả f(x) = ax2
+ bx + c, a /= 0 là tam thác b c hai với h
so nguyên. Khi đó, đieu ki n can và đủ đe có bat đȁng thác
au2
+ bu + c
2av + b
≥
b
av2
+ bv + c
2av + b
+ u − v, ∀u, v ∈ (α, β), u v
là α ≥ −
2a
, trong đó α, β ∈ Z.
ChGng minh.
Trường hợp 1. Khi a > 0.
b
Khi đó với v > −
2a
thì
(2.10) ⇔ au2
+ bu + c ≥ av2
+ bv + c + (u − v)(2av + b), ∀u, v ∈ (α, β)
36
Viết đề tài giá sinh viên – ZALO:0973.287.149-TEAMLUANVAN.COM
2
2
2
2
⇔ a(u − v) ≥ 0, ∀u, v ∈ (α, β).
Đieu này là hien nhiên. V y (2.10) thỏa mãn.
b
Xét v < −
2a
thì
(2.10) ⇔ au2
+bu+c ≤ av2
+bv+c+(u−v)(2av+b), ∀u, v ∈ (α, β), u /= v
⇔ a(u − v) ≤ 0, ∀u, v ∈ (α, β), u /= v.
Đieu này không xảy ra. V y (2.10) không thỏa mãn.
Trường hợp 2. Khi a < 0.
b
Khi đó với v > −
2a
thì
(2.10) ⇔ au2
+bu+c ≤ av2
+bv +c +(u−v)(2av +b), ∀u, v ∈ (α, β), u v
⇔ a(u − v) ≤ 0, ∀u, v ∈ (α, β), u /= v.
Đieu này là hien nhiên. V y (2.10) thỏa mãn.
b
Xét v < −
2a
thì
(2.10) ⇔ au2
+bu+c ≥ av2
+bv+c+(u−v)(2av+b), ∀u, v ∈ (α, β), u /= v
⇔ a(u − v) ≥ 0, ∀u, v ∈ (α, β), u /= v.
Đieu này không xảy ra. V y (2.10) không thỏa mãn.
b
V y (2.10) đúng với v > −
2a
mà ta can (2.10) đúng với moi v ∈ (α, β)
b
nên suy ra α ≥ −
2a
. Ket hợp với đieu ki n can ta có đieu ki n can và đủ
được nêu trong Định lý 2.5.
Tương tự ta cũng có định lý sau:
Định lj 2.6. Giả sả f(x) = ax2
+ bx + c, a /= 0 là tam thác b c hai với h
so nguyên. Khi đó, đieu ki n can và đủ đe có bat đȁng thác
au2
+ bu + c
2av + b
≤
b
là β ≤ −
2a
.
Tà Định lý 2.5, ta có
av2
+ bv + c
2av + b
+ u − v, ∀u, v ∈ (α, β), u v
37
Viết đề tài giá sinh viên – ZALO:0973.287.149-TEAMLUANVAN.COM
/
1 2 3
1 2 3
H quả 2.1. Giả sả f(x) = ax2
+ bx + c, a = 0 là tam thác b c hai với
b
h so nguyên và α ≥ −
2a
. Khi đó, với moi b® so u1, u2, u3 ∈ (α, β) và
v1, v2, v3 ∈ (α, β) sao cho u1 + u2 + u3 = v1 + v2 + v3, ta đeu có
au2
+ bu1 + c au2
+ bu2 + c au2
+ bu3 + c
1
+ 2
+ 3
2av1 + b 2av2 + b 2av3 + b
av2
+ bv1 + c av2
+ bv2 + c av2
+ bv3 + c
+
2av1 + b
+
2av2 + b
.
2av3 + b
Tà Định lý 2.6, ta có
H quả 2.2. Giả sả f(x) = ax2
+ bx + c, a
b
0 là tam thác b c hai với
h so nguyên và β ≤ −
2a
. Khi đó, với moi b® so u1, u2, u3 ∈ (α, β) và
v1, v2, v3 ∈ (α, β) sao cho u1 + u2 + u3 = v1 + v2 + v3, ta đeu có
au2
+ bu1 + c au2
+ bu2 + c au2
+ bu3 + c
1
+ 2
+ 3
2av1 + b 2av2 + b 2av3 + b
av2
+ bv1 + c av2
+ bv2 + c av2
+ bv3 + c
+
2av1 + b
+
2av2 + b
.
2av3 + b
Tiep theo, ta xét m®t so áp dụng liên quan đen hàm phân thác.
Bài toán 2.13. Xét các so nguyên dương x, y, z có x + y + z = 12. Cháng
minh rang
Lài giai.
2x2
+ x
+
21
2y2
+ y
+
17
2z2
+ z
13
≥
29207
.
4641
Theo Định lý 2.5, ta có
2u2
+ u
4v + 1
≥
2v2
+ v
4v + 1
+ u − v, ∀u, v ∈ (0, +∞), u =
/ v. (2.11)
Chon 3 so v ∈ (0, +∞) sao cho 4v + 1 ∈ {21; 17; 13} và có tőng bang 12.
Ta thu được v1 = 5, v2 = 4, v3 = 3. Tiep theo, the vào (2.11), ta được
2x2
+ x
21
≥
2y2
+ y
17
≥
2z2
+ z
13
≥
55
21
+ x − 5, ∀x ∈ (0, +∞);
35
17
+ y − 4, ∀y ∈ (0, +∞);
21
13
+ z − 3, ∀z ∈ (0, +∞).
C®ng các ve tương áng ta thu được đieu can cháng minh.
≥
≤
38
Viết đề tài giá sinh viên – ZALO:0973.287.149-TEAMLUANVAN.COM
− −
− − −
Bài toán 2.14. Xét các so nguyên âm x, y, z có x + y + z = −10. Tìm giá
trị lớn nhat của
Lài giai.
2x x2
A =
12
2y y2
+
8
2z z2
+ .
6
Theo Định lý 2.6, ta có
2u u2
2 − 2v
≤
2v v2
2 − 2v
+ u − v, ∀u, v ∈ (−∞, 0), u v. (2.12)
Chon 3 so v ∈ (−∞, 0) sao cho 2 − 2v ∈ {12; 8; 6} và có tőng bang -10.
Ta thu được v1 = −5, v2 = −3, v3 = −2. Tiep theo, the vào (2.12), ta được
2x − x2
35
12
≤ −
12
+ x + 5, ∀x ∈ (−∞, 0);
2y − y2
15
8
≤ −
8
+ y + 3, ∀y ∈ (−∞, 0);
2z − z2
4
6
≤ −
3
+ z + 2, ∀z ∈ (−∞, 0).
49 49
C®ng các ve tương áng ta thu được A ≤ −
8
. Lại có A = −
8
khi x =
−5, y = −3, z = −2.
49
V y giá trị lớn nhat của A bang −
8
đạt được khi x = −5, y = −3, z =
−2.
Bài toán 2.15. Xét các so nguyên dương x, y, z có x + y + z = 10. Tìm giá
trị nhỏ nhat của bieu thác
A = 35(x2
+ x) + 55(y2
+ y) + 77(z2
+ z).
Lài giai.
B®i so chung nhỏ nhat của 35, 55, 77 là 385. Ta có
A
=
385
Theo Định lý 2.5, ta có
x2
+ x
+
11
y2
+ y
7
z2
+ z
+ .
5
u2
+ u
2v + 1
≥
v2
+ v
2v + 1
+ u − v, ∀u, v ∈ (0, +∞), u v. (2.13)
39
Viết đề tài giá sinh viên – ZALO:0973.287.149-TEAMLUANVAN.COM
− − −
Chon ba so v ∈ (0, +∞) sao cho 2v + 1 ∈ {11; 7; 5} và có tőng bang 10.
Ta thu được v1 = 5, v2 = 3, v3 = 2. Tiep theo, the vào (2.13), ta được
x2
+ x
11
≥
y2
+ y
7
≥
z2
+ z
5
≥
30
11
+ x − 5, ∀x ∈ (0, +∞),
12
7
+ y − 3, ∀y ∈ (0, +∞),
6
5
+ z − 2, ∀z ∈ (0, +∞).
C®ng các ve tương áng ta thu được
A = 2172 khi x = 5, y = 3, z = 2
A
385
≥
2172
385
hay A ≥ 2172, lại có
V y giá trị nhỏ nhat của A là 2172 đạt được khi x = 5, y = 3, z = 2.
Bài toán 2.16. Xét các so nguyên dương x, y, z có x + y + z = 10. Tìm giá
trị lớn nhat của bieu thác
x2
+ x 30
M = +
2x + 1
y2
+ y 12
+
2y + 1
z2
+ z 6
.
2z + 1
Lài giai. Theo Định lý 2.5, ta có
u2
+ u
2v + 1
≥
v2
+ v
2v + 1
+ u − v, ∀u, v ∈ (0, +∞), u v. (2.14)
Chon ba so u ∈ (0, +∞) sao cho u2
+ u ∈ {30; 12; 6} và có tőng bang 10.
Ta thu được u1 = 5, u2 = 3, u3 = 2. Tiep theo, the vào (2.14), ta được
30
2x + 1
≥
12
2y + 1
≥
6
2z + 1
≥
x2
+ x
2x + 1
+ 5 − x, ∀x ∈ (0, +∞),
y2
+ y
2y + 1
+ 3 − y, ∀y ∈ (0, +∞),
z2
+ z
2z + 1
+ 2 − z, ∀z ∈ (0, +∞).
C®ng các ve tương áng ta thu được
30 12 6 x2
+ x y2
+ y z2
+ z
2x + 1
+
2y + 1
+
2z + 1
≥
.
+ + .
2x + 1 2y + 1 2z + 1
Chuyen ve trái của bat đȁng thác này sang bên phải ta thu được M ≤ 0.
Ket lu n: Giá trị lớn nhat của M bang 0 khi x = 5, y = 3, z = 2.
40
Viết đề tài giá sinh viên – ZALO:0973.287.149-TEAMLUANVAN.COM
2.4 Bat đang thfíc sinh b i hàm phân tuyen tính trên
m t khoảng
Cho hàm phân thác hǎu tỉ với h so nguyên
ax + b
f(x) = , c > 0, ad − bc /= 0
.
cx + d
Trong mục này ta xét bài toán xác định các khoảng (α, β), (α < β) sao
cho áng với moi hàm phân thác hǎu tỉ với h so nguyên ở trên ta đeu có bat
đȁng thác
au + b
cu + d
ad − bc
≥
(cv + d)2
av + b
cv + d
ad − bc
(cv + d)2
+ u − v, ∀u, v ∈ (α, β), u v
và xét các áp dụng liên quan.
d
Nh n xét rang đe cx + d /= 0 với moi x ∈ (α, β) thì −
c
∈
/ (α, β). Đieu
d d
này tương đương với ho c −
c
≤ α ho c −
c
≥ β.
Tà đó suy ra
M nh đe 2.2. Cho hàm phân thác hǎu tỉ với h so nguyên
ax + b
f(x) = , c > 0, ad − bc /= 0
.
cx + d
Đieu ki n can đe có bat đȁng thác
au + b
cu + d
ad − bc
≥
(cv + d)2
d
av + b
cv + d
ad − bc
(cv + d)2
d
+ u − v, ∀u, v ∈ (α, β), u v (2.15)
là ho c −
c
≤ α ho c −
c
≥ β.
Tiep theo, ta phát bieu và cháng minh đieu ki n đủ đe có (2.15).
Định lj 2.7. Cho hàm phân thác hǎu tỉ với h so nguyên
ax + b
f(x) = , c > 0, ad − bc /= 0
.
cx + d
41
Viết đề tài giá sinh viên – ZALO:0973.287.149-TEAMLUANVAN.COM
−
−
— −
— −
cv + d (cv + d)2
cv + d (cv + d)2
Khi đó
Neu ad bc > 0 thì đieu ki n can và đủ đe có bat đȁng thác (2.15) là
d
β ≤ −
c
.
Neu ad bc < 0 thì đieu ki n can và đủ đe có bat đȁng thác (2.15) là
d
α ≥ −
c
.
ChGng minh.
Trường hợp 1. Xét ad − bc > 0. Khi đó,
au + b
(2.15) ⇔ ≥
av + b
+ (u − v)
h ad − bc i
, ∀u, v ∈ (α, β)
c(u v)2
(ad bc)
⇔ −
(cu + d)(cv + d)2
≥ 0, ∀u, v ∈ (α, β).
d d
Khi đó, với u > −
c
thì (2.15) không thỏa mãn, với u < −
c
thì (2.15) thỏa
d
mãn, mà ta can (2.15) thỏa mãn ∀u, v ∈ (α, β) nên suy ra β ≤ −
c
.
Trường hợp 2. Xét ad − bc < 0. Khi đó
au + b
(2.15) ⇔ ≤
av + b
+ (u − v)
ad − bc
, ∀u, v ∈ (α, β)
c(u v)2
(ad bc)
⇔ −
(cu + d)(cv + d)2
≤ 0, ∀u, v ∈ (α, β).
d
Khi đó, với u > −
c
thì (2.15) thỏa mãn, với u <
−d
c
thì (2.15) không thỏa
d
mãn, mà ta can (2.15) thỏa mãn ∀u, v ∈ (α, β) nên suy ra α ≥ −
c
.
V y ta có đieu can cháng minh.
Tương tự, ta cũng có định lý:
Định lj 2.8. Cho hàm phân thác hǎu tỉ với h so nguyên
ax + b
f(x) = , c > 0, ad − bc /= 0
.
cx + d
Xét bat đȁng thác
au + b
cu + d
ad − bc
≤
(cv + d)2
av + b
cv + d
ad − bc
(cv + d)2
+ u − v, ∀u, v ∈ (α, β), u v. (2.16)
cu + d
cu + d
42
Viết đề tài giá sinh viên – ZALO:0973.287.149-TEAMLUANVAN.COM
−
−
x + 1 y + 1 z + 1
Khi đó
Neu ad bc > 0 thì đieu ki n can và đủ đe có bat đȁng thác (2.16) là
d
α ≥ −
c
.
Neu ad bc < 0 thì đieu ki n can và đủ đe có bat đȁng thác (2.16) là
d
β ≤ −
c
.
Tà Định lý 2.7, Định lý 2.8 ta phát bieu ket quả đe sả dụng trong các bài
toán áp dụng ở phan sau.
H quả 2.3. Giả sả f(x) =
ax + b
cx + d
là phân thác hǎu tỉ h so nguyên thỏa
d
mãn: c > 0, ad − bc < 0, α ≥ −
c
ho c c > 0, ad − bc > 0, β ≤
d
—
c
, a.b, c, d ∈ Z. Khi đó, với moi b® so u1, u2, u3 ∈ (α, β) và v1, v2, v3 ∈
(α, β) sao cho u1 + u2 + u3 = v1 + v2 + v3, ta đeu có
auk + b
3 avk + b
3
Σ cuk + d ≥
Σ cvk + d .
k=1
ad − bc
(cvk + d)2
ax + b
k=1
ad − bc
(cvk + d)2
H quả 2.4. Giả sả f(x) =
cx + d
là phân thác hǎu tỉ h so nguyên thỏa
d
mãn: c > 0, ad − bc > 0, α ≥ −
c
ho c c > 0, ad − bc < 0, β ≤
d
—
c
, a.b, c, d ∈ Z. Khi đó, với moi b® so u1, u2, u3 ∈ (α, β) và v1, v2, v3 ∈
(α, β) sao cho u1 + u2 + u3 = v1 + v2 + v3, ta đeu có
auk + b
3 avk + b
3
Σ cuk + d ≤
Σ cvk + d .
k=1
ad − bc
(cvk + d)2
k=1
ad − bc
(cvk + d)2
Bài toán 2.17. Xét các so nguyên dương x, y, z có x + y + z = 10. Tìm giá
trị lớn nhat của
A = 36
−x + 2
+ 16
−y + 2
+ 9
−z + 2
.
43
Viết đề tài giá sinh viên – ZALO:0973.287.149-TEAMLUANVAN.COM
u + 1
x + 1
y + 1
z + 1
u + 1
x + 1 y + 1 z + 1
Lài giai. Theo h quả 2.3, ta có
−u + 2
u + 1
3
≥
−
(v + 1)2
−v + 2
v + 1
−3
(v + 1)2
+ u − v, ∀u, v ∈ (0, +∞), u /= v
⇔ (v + 1)2 −u + 2
≤ (−v + 2)(v + 1) − 3(u − v), u, v ∈ (0, +∞), u /= v.
(2.17)
Chon ba so v ∈ (0, +∞) sao cho (v + 1)2
∈ {36; 16; 9} và có tőng bang 10.
Ta thu được v1 = 5, v2 = 3, v3 = 2.
Tiep theo, the vào (2.17), ta được
36
−x + 2
≤ −18 − 3(x − 5), ∀x ∈ (0, +∞),
16
−y + 2
≤ −4 − 3(y − 3), ∀y ∈ (0, +∞),
9
−z + 2
≤ −3(z − 2), ∀z ∈ (0, +∞).
C®ng các ve tương áng ta thu được A ≤ −22. Lại có A = −22 khi x =
5, y = 3, z = 2.
V y giá trị lớn nhat của A bang -22, đạt được khi x = 5, y = 3, z = 2.
Bài toán 2.18. Xét các so nguyên âm x, y, z có x + y + z = −9. Cháng
minh bat đȁng thác
9(2x − 1)
+
4(2y − 1)
+
2z − 1
≤ 46.
Lài giai. Theo h quả 2.4, ta có
2u − 1
u + 1
3 ≤
(v + 1)2
2v − 1
v + 1
3
(v + 1)2
+ u − v, ∀u, v ∈ (−∞; 0), u v
⇔ (v + 1)2 2u − 1
≤ (2v − 1)(v + 1) + 3(u − v), u, v ∈ (−∞; 0), u v.
(2.18)
44
Viết đề tài giá sinh viên – ZALO:0973.287.149-TEAMLUANVAN.COM
x + 1
y + 1
z + 1
u + 1
Chon ba so v ∈ (−∞; 0) sao cho (v + 1)2
∈ {9; 4; 1} và có tőng bang -9. Ta
thu được v1 = −4, v2 = −3, v3 = −2.
Tiep theo, the vào (2.18), ta được
9
2x − 1
≤ 27 + 3(x + 4), ∀x ∈ (−∞; 0),
4
2y − 1
≤ 14 + 3(y + 3), ∀y ∈ (−∞; 0),
2z − 2
≤ 5 + 3(z + 2), ∀z ∈ (−∞; 0).
C®ng các ve tương áng ta thu được đpcm.
Bài toán 2.19. Xét các so nguyên dương x, y, z có x + y + z = 10. Tìm giá
trị lớn nhat của bieu thác
P = 36
x − 1
+ 16
y − 1
+ 9
z − 1
.
x + 1 y + 1 z + 1
Lài giai. Theo h quả 2.4, ta có
u − 1
u + 1
2 ≤
(v + 1)2
v − 1
v + 1
2
(v + 1)2
+ u − v, ∀u, v ∈ (0, +∞), u /= v
⇔ (v + 1)2 u − 1
≤ v2
− 1 + 2(u − v), ∀u, v ∈ (0, +∞), u /= v. (2.19)
Chon ba so v ∈ (0, +∞) sao cho (v + 1)2
∈ {36; 16; 9} và có tőng bang 10.
Ta thu được v1 = 5, v2 = 3, v3 = 2. Tiep theo, the vào (2.19), ta được
36
x − 1
x + 1
16
y − 1
y + 1
≤ 24 + 2(x − 5), ∀x ∈ (0, +∞),
≤ 8 + 2(y − 3), ∀y ∈ (0, +∞),
9
z − 1
z + 1
≤ 3 + 2(z − 2), ∀z ∈ (0, +∞).
C®ng các ve tương áng ta thu được P ≤ 35, ∀x, y, z ∈ (0, +∞). V y
max P = 35, đạt được khi x = 5, y = 3, z = 2.
45
Viết đề tài giá sinh viên – ZALO:0973.287.149-TEAMLUANVAN.COM
Σ
Σ
Σ
Σ
2.5 Phân thfíc chính quy và m t so tính chat
Định nghĩa 2.2 (xem [4]). Hàm f(x) xác định trên t p các so thực dương,
không đong nhat với 0 được goi là phân thác chính quy neu
n
f(x) = akxαk
,
k=1
n
Ví dn 2.1. De dàng kiem cháng các hàm so sau đây là phân thác chính quy
f (x) = 7 + 3x + x2
+
1
1
x2
1
+
x3
,
f (x) = 4x2
+ 3x3
+
1
2
x5
3
+
x4
.
Định nghĩa 2.3 (xem [4]). Hàm f(x) xác định trên t p các so thực dương,
không đong nhat với 0 được goi là phân thác chính quy hǎu tỉ neu
n
f(x) = akxαk
,
k=1
n
Tà định nghĩa, ta de dàng suy ra các tính chat sau:
Tính chat 2.1. Neu f (x) là hàm phân thác chính quy thì f (x) > 0 áng với
moi x > 0.
Tính chat 2.2. Neu f(x) và g(x) là các hàm phân thác chính quy thì với
moi c p so dương α, β, hàm so h(x) = αf (x) + βg(x) cũng là hàm phân thác
chính quy.
Tính chat 2.3. Neu f (x) và g(x) là các hàm phân thác chính quy thì hàm
so h(x) = f(g(x)) cũng là hàm phân thác chính quy.
Tính chat 2.4. Neu f (x) là hàm phân thác chính quy thì hàm so h(x) =
[f(x)]m
với m nguyên dương cũng là hàm phân thác chính quy.
akαk = 0, ∀k = 1, n.
k=1
akαk = 0, ∀k = 1, n.
k=1
trong đó ak > 0; αk ∈ Z;
trong đó ak ∈ N; αk ∈ Z;
46
Viết đề tài giá sinh viên – ZALO:0973.287.149-TEAMLUANVAN.COM
Σ
Σ
f(x1, x2, . . . , xm) =
Σ
akxαk1
xαk2
. . . xαkm
,
f(x1, x2, . . . , xm) = akxαk1
xαk2
. . . xαkm
,
Định nghĩa 2.4 (xem [4]). Hàm so f (x1, x2, . . . , xm) không đong nhat 0,
xác định với xi dương, ∀i = 1, n được goi là hàm phân thác chính quy hǎu
tỉ m bien neu
1 2 m
k=1
trong đó
ak ∈ N; αkj ∈ Z; ∀k = 1, n; ∀j = 1, m,
a1α11 + a2α21 + · · · + anαn1 = 0,
a1α12 + a2α22 + · · · + anαn2 = 0,
. . . ,
a1α1m + a2α2m + · · · + anαnm = 0.
Tà định nghĩa trên, ta có tính chat sau:
Tính chat 2.5. Hàm so f (x1, x2, . . . , xm) là hàm phân thác chính quy hǎu
tỉ khi và chỉ khi các hàm phân thác thành phan f (1, . . . , 1, xk, 1, . . . , 1) cũng
là hàm phân thác chính quy hǎu tỉ.
Ví dn 2.2. De dàng kiem tra được hàm so
f(x, y) = 2x3
y8
+ 3x4
y9
+
6
+
1
x3y7 y
là hàm phân thác chính quy, và các hàm phân thác thành phan là: f1(x) =
2x3
+ 3x4
+ 6x−3
+ 1; f2(y) = 2y8
+ 3y9
+ 6y−7
+ y−1
.
Định lj 2.9 (xem [4]). Với moi hàm phân thác chính quy
nk
1 2 m
k=1
Σ
n
ta đeu có f(x1, . . . , xm) ≥
ChGng minh.
k=1
ak, ∀xi dương, i = 1, n.
Áp dụng bat đȁng thác AM - GM ta có
n
akxαk1
xαk2
. . . xαkm
f(x1, x2, . . . , xm) 1 2 m
= k=1
a1 + a2 + · · · + an a1 + a2 + · · · + an
n
47
Viết đề tài giá sinh viên – ZALO:0973.287.149-TEAMLUANVAN.COM
Σ
Σ Σ
−
Σ
−
(x) = x
a
Σ
p
n n
Σ
akαkj = 0,
ak αk −
p
= ak
p
= q − p.
p
= 0.
Σ Σ
k=1
Σ
n
akαk1
n
akαk2
n
akαkn
1
a1+a2+···+an
≥ xk=1
Σ
n
k=1
2
. . . xk=1
= 1
Dau "=" xảy ra khi và chỉ khi x1 = x2 = · · · = xm = 1.
H quả 2.5. Với moi hàm phân thác chính quy f(x1, x2, . . . , xm) với xi
dương, ∀i = 1, n, ta đeu có min f(x1, x2, . . . , xm) = f(1, 1, . . . , 1).
Nh n xét 2.2. Với moi hàm phân thác dạng g(x) =
n
k=1
akxαk
, ak ≥ 0, đ t
n n
ak = p, akαk = q thì hàm so
k=1 k=1
−q
f(x) = x p g(x)
là m®t hàm phân thác chính quy.
ChGng minh.
Ta có
n n
nên
f(x) =
q
x p g
q
p
k=1
kxαk
= ak
k=1
xαk− q
Σ q Σ Σ q q
Đieu đó cháng tỏ f(x) là phân thác chính quy.
Tà đó ta thu được định lí quan trong sau đây:
Định lj 2.10 (xem [4]). Moi hàm phân thác chính quy dạng
đeu có tính chat
n
g(x) = akxαk
, ak ≥ 0, k = 1, n
k=1
q
g(x) ≥ g(1)xp , ∀x > 0,
k=1
k=1
m
x
1
n
do j = 1, m
.
k=1
akαk −
48
Viết đề tài giá sinh viên – ZALO:0973.287.149-TEAMLUANVAN.COM
Σ Σ
trong đó
ChGng minh.
n
ak = p,
k=1
n
akαk = q.
k=1
−q
Theo nh n xét trên, ta có f(x) = x p g(x) là m®t phân thác chính quy
nên theo H quả 2.5, ta suy ra f(x) ≥ f(1) mà f(1) = g(1) nên ta có đieu
can cháng minh.
Ví dn 2.3. Tìm giá trị nhỏ nhat của bieu thác:
f(x, y) = 2x−3
y3
+ x5
y−1
+ xy−5
(x > 0, y > 0).
Ta de dàng kiem tra được f(x, y) là phân thác chính quy hai bien x; y.
Ta có f(1, 1) = 4
f(x, y) x−3y3 x5y−1 xy−5 −3 3 5 −1 1 −5
= +
4 2
⇔f(x, y) ≥ 4.
4
+
2
≥ x 2 y2 x4 y 4 x4 y 4 = 1
V y min f(x, y) = 4 khi và chỉ khi x = 1, y = 1.
49
Viết đề tài giá sinh viên – ZALO:0973.287.149-TEAMLUANVAN.COM
Chương 3
M t so bài toán liên quan đen bat
đang thfíc và cfic trị trên t p so
nguyên
3.1 Bat đang thfíc trên t p so nguyên
Trong mục này sě trình bày m®t so bài toán ve bat đȁng thác trong t p
so nguyên. Đe cháng minh bat đȁng thác trên t p so nguyên ta van áp dụng
các bat đȁng thác quen thu®c như: bat đȁng thác AM - GM..., tuy nhiên
không phải lúc nào cũng áp dụng trực tiep được mà can ky thu t tách, ghép
m®t cách khéo léo. đây ta sả dụng tính chat của so nguyên đe tách, ghép
chȁng hạn như: so nguyên dương n có the tách được thành tőng n so 1, ho c
tích ab (với a, b nguyên dương) có the tách thành tőng của b so a ho c tőng
của a so b. . . .Các bài toán dưới đây sě làm rõ thêm ý tưởng trên.
Bài toán 3.1 (xem[3]). Với a, b, c là các so tự nhiên dương. Cháng minh
rang 2 2 2 1
a + b + c
a + b + c
≥ aa
bb
cc a+b+c
.
50
Viết đề tài giá sinh viên – ZALO:0973.287.149-TEAMLUANVAN.COM
Σ
a2
+ b2
+ c2
a + b + c
1
n
Y n
Lài giai. Đ t
Ta có
n = a + b + c; a, b, c ∈ N∗,
x1 = x2 = · · · = xa = a,
xa+1 = xa+2 = · · · = xa+b = b,
xa+b+1 = xa+b+2 = · · · = xa+b+c = c.
a2
= a.a = x1 + x2 + · · · + xa,
b2
= b.b = xa+1 + xa+2 + · · · + xa+b,
c2
= c.c = xa+b+1 + xa+b+2 + · · · + xa+b+c,
a a+b n
aa
=
Y
xi, bb
=
Y
xi, cc
=
Y
xi.
Suy ra
i=1
n
i=a
n
i=a+b
!1
Ta có đieu can cháng minh.
i=1 i=1
Bài toán 3.2 (xem[3]). Với a, b, c là các so tự nhiên dương. Cháng minh
rang
ab + bc + ca a b c
Lài giai. Đ t
a + b + c
≥ ba+b+c ca+b+c aa+b+c .
n = a + b + c; a, b, c ∈ N∗,
x1 = x2 = · · · = xa = b,
xa+1 = xa+2 = · · · = xa+b = c,
xa+b+1 = xa+b+2 = · · · = xa+b+c = a.
1
a+b+c
.
aa
.bb
.cc
=
xi
= xi ≥
51
Viết đề tài giá sinh viên – ZALO:0973.287.149-TEAMLUANVAN.COM
Σ Y !
Ta thu được
ab = x1 + x2 + · · · + xb,
bc = xb+1 + xb+2 + · · · + xb+c,
ca = xb+c+1 + xb+c+2 + · · · + xb+c+a,
ba
= x1x2 . . . xa,
cb
= xa+1xa+2 . . . xa+b,
ac
= xa+b+1xa+b+2 . . . xa+b+c,
n
ab + bc + ca =
1
(ab + bc + ac) =
1 Σ
x ,
a + b + c n n
i
i=1
n !1
a
ba+b+c
b
ca+b+c
c
aa+b+c =
1
ba
cb
ac n
= n
xi .
i=1
Áp dụng bat đȁng thác AM-GM ta có
n
1
n
i=1
xi ≥
1
n n
xi
i=1
với n là sô nguyên dương, xi là so thực dương, ∀i = 1, n.
Tà đó suy ra đieu can cháng minh.
Bài toán 3.3 (xem[3]). Với a, b, c là các so tự nhiên dương. Cháng minh
rang
a b a b
Lài giai. Đ t
(1 + a)a+b (1 + b)a+b ≥ 1 + aa+b ba+b .
n = a + b; a, b ∈ N∗,
x1 = x2 = · · · = xa = a,
xa+1 = xa+2 = · · · = xa+b = b.
Y
52
Viết đề tài giá sinh viên – ZALO:0973.287.149-TEAMLUANVAN.COM
n !
n !
Y Y !
(1 + xi)
xi
i=1
i=1
Ta thu được
1
Y n
1
Y n
Áp dụng Bat đȁng thác 1.4.1 ta có
n
i=1
(1 + xi)
1
n
≥ 1 +
1
n n
xi
i=1
với n là sô nguyên dương, xi là so thực dương, ∀i = 1, n. Tà đó suy ra đieu
can cháng minh.
Bài toán 3.4 (xem[3]). Với a, b, c là các so tự nhiên dương. Cháng minh
rang
Lài giai. Đ t
(a + b + c)2
(a4
+ b4
+ c4
) ≥ (a2
+ b2
+ c2
)3
.
n = a + b + c; a, b, c ∈ N∗,
x1 = x2 = · · · = xa = a,
xa+1 = xa+2 = · · · = xa+b = b,
xa+b+1 = xa+b+2 = · · · = xa+b+c = c.
!
a b
(1 + a)a+b .(1 + b)a+b = ,
a b
aa+b .ba+b = .
53
Viết đề tài giá sinh viên – ZALO:0973.287.149-TEAMLUANVAN.COM
b = x a+b
c = x a+b+c
Σ
Σ
x
Σ
x
Σ
!
!
3
Σ
!
3
!
i=1
Khi đó ta được
a2
= x1 + x2 + · · · + xa,
b2
= xa+1 + xa+2 + · · · + xa+b,
c2
= xa+b+1 + xa+b+2 + · · · + xa+b+c,
a4
= x3
+ x3
+ · · · + x3
,
1
4 3
a+1
2
3
a+2
a
+ · · · + x3
,
4 3
a+b+1
3
a+b+2 + · · · + x3
,
Σ
n
(a2
+ b2
+ c2
)3
=
n
i=1
3
xi .
Áp dụng Bat đȁng thác 1.4.4 ta được
1
n
n i ≥
i=1
3
1
n3
xi
i=1
với n là sô nguyên dương, xi là so thực dương, ∀i = 1, n.
Hay
n
n2
i ≥
i=1
n
i=1
3
xi .
Đây là đieu can cháng minh.
Bài toán 3.5 (xem[3]). Với a, b, c là các so tự nhiên dương. Cháng minh
rang
(a + b + c)2
(ab3
+ bc3
+ ca3
) ≥ (ab + bc + ca)3
.
+ x
+ x
x
n
(a + b + c)2
(a4
+ b4
+ c4
) = n2 3
i ,
!
!
54
Viết đề tài giá sinh viên – ZALO:0973.287.149-TEAMLUANVAN.COM
a+1 a+b
ca = x a+b+c
Σ
Σ
Σ
x
Σ
x
Σ
!
!
3
Σ
!
3
!
Lài giai. Đ t
Khi đó ta được
n = a + b + c; a, b, c ∈ N∗,
x1 = x2 = · · · = xa = b,
xa+1 = xa+2 = · · · = xa+b = c,
xa+b+1 = xa+b+2 = · · · = xa+b+c = a.
ab = x1 + x2 + · · · + xa,
bc = xa+1 + xa+2 + · · · + xa+b,
ca = xa+b+1 + xa+b+2 + · · · + xa+b+c,
ab3
= x3
+ x3
+ · · · + x3
,
1
bc3
= x3
2
3
a+2
a
+ · · · + x3
,
3 3
a+b+1
3
a+b+2 + · · · + x3
,
(a + b + c)2
(ab3
+ bc3
+ ca3
) = n2
n
3
i
i=1
(ab + bc + ca)3
=
n
i=1
3
xi .
Áp dụng Bat đȁng thác 1.4.4 ta được
1
n
n i ≥
i=1
3
1
n3
xi
i=1
với n là sô nguyên dương, xi là so thực dương, ∀i = 1, n.
Hay
n
n2
i ≥
i=1
n
i=1
3
xi .
Đây là đieu can cháng minh.
Bài toán 3.6 (xem[3]). Với a, b, c là các so tự nhiên dương. Cháng minh
+ x
+ x
x
n
!
,
!
55
Viết đề tài giá sinh viên – ZALO:0973.287.149-TEAMLUANVAN.COM
a
i
√ Σ
a+b q
i
√
aΣ
+b+c q
i
Σ
n
1 + x2
≤ ,n2 +
xi
i
i=1
rang
a
√
1 + a2 + b
√
1 + b2 + c
√
1 + c2 ≥
q
(a + b + c)2 + (a2 + b2 + c2)2.
Lài giai. Đ t
n = a + b + c; a, b, c ∈ N∗,
x1 = x2 = · · · = xa = a,
xa+1 = xa+2 = · · · = xa+b = b,
xa+b+1 = xa+b+2 = · · · = xa+b+c = c.
Khi đó ta được
a
√
1 + a2 =
Σ
i=1
q
1 + x2
,
b 1 + b2 = 1 + x2
,
c 1 + c2 =
i=a
1 + x2
,
i=a+b
√ √ √ Σ
i=1 q
a 1 + a2 + b 1 + b2 + c
n
1 + c2 =
n 1 + x2
,
a2
+ b2
+ c2
= xi.
i=1
Áp dụng Bat đȁng thác 1.4.2 ta có
Σ q
‚
. Σ
n
!2
với n là sô nguyên dương, xi là so thực dương, ∀i = 1, n.
Hay
a
√
1 + a2 + b
√
1 + b2 + c
√
1 + c2 ≥
q
(a + b + c)2 + (a2 + b2 + c2)2.
Đây là đieu can cháng minh.
i=1
i
56
Viết đề tài giá sinh viên – ZALO:0973.287.149-TEAMLUANVAN.COM
√
q
1 + x ,
√ Σ
a+b q
i
n
1 + x2
≥ ,n2 + xi
i
i=1
Bài toán 3.7 (xem[3]). Với a, b là các so tự nhiên dương. Cháng minh rang
a
√
1 + b2 + b
√
1 + a2 ≥
q
(a + b)2 + 4a2b2.
Lài giai. Đ t
Khi đó ta được
n = a + b; a, b ∈ N∗,
x1 = x2 = · · · = xa = b,
xa+1 = xa+2 = · · · = xa+b = a.
a 1 + b2 = 2
i
i=1
b 1 + a2 = 1 + x2
,
i=a
√ √ Σ
i=1 q
a 1 + a2 + b 1 + b2 =
n
Σ
a
1 + x2
,
a+b
!2 Σ
n
!2
4a2
b2
= (ab + ba)2
=
Theo Bat đȁng thác 1.4.2, ta có
i=1
xi + xi
i=a
= xi .
i=1
Σ q
‚
. Σ
n
!2
với n là sô nguyên dương, xi là so thực dương, ∀i = 1, n.
Hay
a
√
1 + b2 + b
√
1 + a2 ≥
q
(a + b)2 + 4a2b2.
Đây là đieu can cháng minh.
Bài toán 3.8 (xem[3]). Với a, b là các so tự nhiên dương. Cháng minh rang
a2 b2 c2 (a + b + c)(a2
+ b2
+ c2
)
1 + a
+
1 + b
+
1 + c
≤
(a + b + c) + (a2 + b2 + c2)
.
i=1
i
a
Σ
Σ
57
Viết đề tài giá sinh viên – ZALO:0973.287.149-TEAMLUANVAN.COM
a
Σ
Σ
Σ
1 + a 1 + b a + b + 2ab
i=a
Lài giai. Đ t
Khi đó ta được
n = a + b + c; a, b, c ∈ N∗,
x1 = x2 = · · · = xa = a,
xa+1 = xa+2 = · · · = xa+b = b,
xa+b+1 = xa+b+2 = · · · = xa+b+c = c.
a2
=
Σ xi
,
1 + a
2
i=1
a+b
1 + xi
b
=
Σ xi
,
c2 a+b+c
=
xi
,
2 2 2 i=1
a b c
+ + =
Σ xi
,
1 + a 1 + b 1 + c
n
n
1 + xi
a2
+ b2
+ c2
= xi.
i=1
Áp dụng Bat đȁng thác 1.4.3 ta có
Σ
n
Σ xi
n xi
i=1
i=1
1 + xi
n
n + xi
i=1
với n là sô nguyên dương, xi là so thực dương, ∀i = 1, n.
Hay
a2 b2 c2 (a + b + c)(a2
+ b2
+ c2
)
1 + a
+
1 + b
+
1 + c
≤
Đây là đieu can cháng minh.
(a + b + c) + (a2 + b2 + c2)
.
Bài toán 3.9 (xem[3]). Với a, b là các so tự nhiên dương. Cháng minh rang
ab
1
+
1
≤
2ab(a + b)
.
n
1 + b 1 + xi
1 + c i=a+b
1 + xi
≤
58
Viết đề tài giá sinh viên – ZALO:0973.287.149-TEAMLUANVAN.COM
b
Σ
Σ Σ
Σ
Lài giai. Đ t
Khi đó ta được
n = a + b; a, b ∈ N∗,
x1 = x2 = · · · = xb = a,
xb+1 = xb+2 = · · · = xa+b = b.
ab Σ xi
= ,
1 + a
ba
=
i=1
a+b
1 + xi
xi
,
1 + b 1 + xi
i=b+1 n
ab ba
+ =
Σ xi
,
1 + a 1 + b i=1
b
1 + xi
a+b n
2ab = ab + ba = xi +
i=1
i=
Σ
b+1
xi = xi.
i=1
Áp dụng Bat đȁng thác 1.4.3, ta có
Σ
n
Σ xi
n xi
i=1
i=1
1 + xi
n
n + xi
i=1
với n là sô nguyên dương, xi là so thực dương, ∀i = 1, n.
Hay
ab
1
+
1
≤
2ab(a + b)
.
1 + a
Đây là đieu can cháng minh.
1 + b a + b + 2ab
3.2 Cfic trị trên t p so nguyên
Trong mục này ta trình bày các bài toán ve cực trị trên t p so nguyên. Đe
tìm giá trị lớn nhat, giá trị nhỏ nhat của m®t bieu thác trên t p so nguyên
n
≤
59
Viết đề tài giá sinh viên – ZALO:0973.287.149-TEAMLUANVAN.COM
−
2 #
−
ta can ket hợp giǎa vi c ước lượng giá trị bieu thác và các tính chat so hoc
của so nguyên như: tính chia het, đong dư, tính sap thá tự. . . .Đôi khi còn
phải ket hợp đánh giá bieu thác với giải phương trình nghi m nguyên đe tìm
được giá trị của bien khi bieu thác đạt cực trị. Các bài toán dưới đây sě làm
rõ hơn ý tưởng trên.
Bài toán 3.10. Với m, n là các so nguyên dương sao cho tőng m so dương
chȁn khác nhau và n so dương lẻ khác nhau bang 2369. Tìm giá trị lớn nhat
của P = 3m + 2n.
Lài giai.
Tőng của m so dương chȁn phân bi t nhỏ nhat là
m(m + 1)
2 + 4 + · · · + 2m = 2
2
Tőng của n so dương lẻ phân bi t nhỏ nhat là
= m2
+ m.
Do đó
1 + 3 + · · · + 2n − 1 = n2
.
hay
2369 ≥ n2
+ m2
+ m =
1 2
1
m + + n2
2 4
Ta có
1 2
m + + n2
2
9477
.
4
P = 3m + 2n = 3 m +
1
+ 2n −
3
≤
‚
.
,(32 + 22)
"
m +
2 2
1
+ n2
3
2 2
≤
r
13.
9477
−
3 = 174.
4 2
Xét h phương trình
3m + 2n = 174
m2
+ m + n2
= 2369.
(
≤
60
Viết đề tài giá sinh viên – ZALO:0973.287.149-TEAMLUANVAN.COM
m
Ta có 3m + 2n = 174 , suy ra n = 87 − 3
2
. Do đó m = 2t; n = 87 − 3t.
Suy ra (2t)2
+ 2t + (87 − 3t)2
= 2369 hay t = 20.
V y h trên có nghi m m = 40, n = 27.
Tóm lại P ≤ 174 với moi m, n thỏa mãn yêu cau đe bài, hơn nǎa áng với
m = 40, n = 27 thì P = 174. V y giá trị lớn nhat của P là 174.
Bài toán 3.11 (xem [2]). Cho k là so nguyên dương lớn hơn 2. Tìm giá trị
lớn nhat của bieu thác
f(x, y, z) = xy + 2xz + 3yz
trên mien D = {(x, y, z) : x, y, z nguyên dương và x + y + z = k}.
Lài giai.
Vì t p hợp D có hǎu hạn phan tả (x, y, z) nên P sě có giá trị lớn nhat
trên D. Giả sả
max
(x,y,z)∈D
f(x, y, z) = x0y0 + 2x0z0 + 3y0z0. (3.1)
Xét x0 ≥ 2. Khi đó có the xảy ra hai trường hợp
1. Neu z0 ≥ y0. De thay (x0 − 1, y0 + 1, z0) ∈ D và
f(x0 − 1, y0 + 1, z0) = x0y0 + 2x0z0 + 3y0z0 + (z0 − y0) + (x0 − 1).
Do x0 > 1 và z0 ≥ y0 nên tà trên suy ra
f(x0 − 1, y0 + 1, z0) > f(x0, y0, z0).
Đieu này mâu thuan với giả thiet (3.1).
V y không xảy ra trường hợp z0 ≥ y0.
2. Neu z0 < y0. De thay (x0 − 1, y0, z0 + 1) ∈ D và
f(x0 − 1, y0, z0 + 1) = x0y0 + 2x0z0 + 3y0z0 + 2(y0 − z0) + 2(x0 − 1).
Do x0 ≥ 2 và z0 < y0 nên tà trên suy ra
f(x0 − 1, y0 + 1, z0) > f(x0, y0, z0).
61
Viết đề tài giá sinh viên – ZALO:0973.287.149-TEAMLUANVAN.COM
Đieu này mâu thuan với giả thiet (3.1). Suy ra không xảy ra trường hợp
z0 < y0. V y không the xảy ra giả thiet x0 ≥ 2. Tà đây suy ra x0 = 1.
Khi x0 = 1, bài toán trở thành: Tìm giá trị lớn nhat của bieu thác
f1(y, z) = y + 2z + 3yz
trên mien D1 = {(y, z) : y, z nguyên dương và y + z = k − 1}, ở đây k là so
nguyên dương lớn hơn 2.
Do t p hợp D1 có hǎu hạn phan tả (y, z) nên f1(y, z) sě có giá trị lớn
nhat trên D1. Giả sả
Ta sě cháng minh
bang phản cháng.
max
(y,z)∈D1
f1(y, z) = y1 + 2z1 + 3y1z1.
z1 ≥ y1 (3.2)
Th t v y, giả sả z1 < y1. De thay (y1 − 1, z1 + 1) ∈ D1 và
f1(y1 − 1, z1 + 1) = y1 + 2z1 + 3y1z1 + 3(y1 − z1 − 1) + 1.
Do z1 < y1 và z1, y1 là so nguyên nên z1 + 1 ≤ y1. Suy ra
f(y1 − 1, z1 + 1) > f(y1, z1).
Vô lý. V y giả thiet z1 < y1 sai , (3.2) đúng nghĩa là z1 ≤ y1
Ta lại cháng minh
bang phản cháng.
Th t v y, giả sả
z1 − y1 ≤ 1 (3.3)
z1 − y1 > 1 (3.4)
De thay (y1 + 1, z1 − 1) ∈ D1 và
f1(y1 + 1, z1 − 1) = y1 + 2z1 + 3y1z1 + 3(z1 − y1 − 1).
62
Viết đề tài giá sinh viên – ZALO:0973.287.149-TEAMLUANVAN.COM
−
1 1
−
Do z1 − y1 > 1 và z1, y1 là so nguyên nên z1 − y1 − 1 ≥ 1, suy ra
f1(y1 − 1, z1 + 1) > f1(y1, z1).
Đieu này mâu thuan với giả thiet. Cháng tỏ đieu giả sả (3.4) là sai. V y (3.3)
đúng. Tà (3.2) và (3.3) ta có 0 ≤ z1 − y1 ≤ 1.
Như v y có hai khả năng xảy ra
1. Neu z1 − y1 = 0 hay z1 = y1 , suy ra
z1 = y1 =
k − 1
.
2
Vì the đe z1, y1 nguyên thì k − 1 chia het cho 2 hay k lẻ. Lúc này
max
(x,y,z)∈D
f(x, y, z) = f 1,
k − 1
,
2
k − 1
2
= 3
k − 1
2
(k 1)2
+ 3
4
3k2
+ 1
= .
4
2. Neu z1 − y1 = 1 hay z1 = y1 + 1 , tà z1 + y1 = k − 1, suy ra
y =
k − 2
, z =
k
.
2 2
Vì the đe z1, y1 nguyên thì k chia het cho 2 hay k chȁn . Lúc này
max
(x,y,z)inD
f(x, y, z) = f 1,
k − 2
,
k
2 2
3k2
4
= .
4
Ket lu n max
(x,y,z)∈D
f(x, y, z) =
3k2
+ 1
4
3k2
− 4
4
khi k lẻ
khi k chȁn.
Bài toán 3.12 (xem [2] ). Cho k là so nguyên dương lớn hơn 2. Tìm giá trị
lớn nhat của hàm so
f(x, y, z) = xyz
trên mien D = {(x, y, z) : x, y, z nguyên dương và x + y + z = k}.
Lài giai.
Không giảm tính tőng quát, giả sả x ≥ y ≥ z.
63
Viết đề tài giá sinh viên – ZALO:0973.287.149-TEAMLUANVAN.COM
Vì D là t p hǎu hạn các phan tả (x, y, z) nên f(x, y, z) sě đạt giá trị lớn
nhat trên D. Giả sả
max
(x,y,z)∈D
f(x, y, z) = f(x0, y0, z0) = x0y0z0 vớix0 ≥ y0 ≥ z0. (3.5)
Giả sả x0 − z0 > 1. Khi đó chỉ có the xảy ra ba trường hợp
1. x0 = y0 > z0 + 1.
Ta có x0 + y0 + z0 = k hay x0 + (y0 − 1) + (z0 + 1) = k.
M t khác do x0 = y0 > z0 +1 và z0 > 0 nên x0, (y0 −1), (z0 +1) cũng nguyên
dương, tác là (x0, y0 − 1, z0 + 1) ∈ D.
Lại có
f(x0, y0 − 1, z0 + 1) = x0(y0 − 1)(z0 + 1) = x0y0z0 + x0(y0 − z0 − 1).
Do x0 = y0 > z0 + 1 nên tà trên suy ra
f(x0, y0 − 1, z0 + 1) > x0y0z0 = f(x0, y0, z0).
Đieu này mâu thuan với (3.5).V y trường hợp 1 không the xảy ra.
2. Neu x0 > y0 > z0 .
Xét b® ba nguyên dương (x0 − 1, y0, z0 + 1). Do x0 > y0 > z0 và x0, y0, z0
nguyên dương nên x0 ≥ y0 + 1 và y0 ≥ z0 + 1, suy ra x0 − 1 > 0. Rõ ràng
(x0 − 1) + y0 + (z0 + 1) = k nên (x0 − 1, y0, z0 + 1) ∈ D.
M t khác
f(x0 − 1, y0, z0 + 1) = y0(x0 − 1)(z0 + 1) = x0y0z0 + y0(x0 − z0 − 1) > x0y0z0.
Suy ra
f(x0 − 1, y0, z0 + 1) > f(x0, y0, z0).
Đieu này mâu thuan với (3.5). Suy ra trường hợp 2 không the xảy ra.
3. x0 − 1 > y0 > z0.
L p lu n tương tự như trên ta có (x0 − 1) + (y0 + 1) + z0 = k nên
(x0 − 1, y0 + 1, z0) ∈ D và
f(x0 − 1, y0 + 1, z0) = x0y0z0 + z0(x0 − y0 − 1) > f(x0, y0, z0).
64
Viết đề tài giá sinh viên – ZALO:0973.287.149-TEAMLUANVAN.COM
−
−
Đieu này mâu thuan với (3.5). Suy ra trường hợp 3 không the xảy ra.
Đieu đó cháng tỏ giả thiet x0 − z0 > 1 là sai. V y ta phải có x0 − z0 ≤ 1.
Do v y chỉ có the xảy ra hai trường hợp
a. Neu x0 − z0 = 0. Ket hợp với x ≥ y ≥ z và x0 + y0 + z0 = k, ta có
k
x0 = y0 = z0 =
3
.
Đieu này xảy ra khi và chỉ khi k chia het cho 3.
b. Neu x0 − z0 = 1. Lúc này lại có hai khả năng
b1. x0 = y0 + 1 = z0 + 1. Suy ra
k + 2
x = , y = z =
k − 1
.
0
3 0 0
3
Trường hợp này xảy khi k ≡ 1 (mod 3).
b2. x0 = y0 = z0 + 1.
Suy ra
x0 = y0
k + 1
= , z0
3
=
k − 2
.
3
Trường hợp này xảy khi k ≡ 2 (mod 3). Ket lu n
max
(x,y,z)∈D
f(x, y, z) =
k3
27
khi k ≡ 0 (mod 3)
(k + 2)(k 1)2
27
khi k ≡ 1 (mod 3)
(k 2)(k + 1)2
27
khi k ≡ 2 (mod 3).
Bài toán 3.13 (xem [2]). Tìm giá trị nhỏ nhat của hàm so
f(x, y, z, t) = x2
+ y2
+ 2z2
+ t2
trên mien D = {(x, y, z, t) : x, y, z, t ∈ N; x2
− y2
+ t2
= 21; x2
+ 3y3
+ 4z2
=
101}.
Lài giai.
Lay (x, y, z, t) tùy ý thu®c D, ta có
x2
− y2
+ t2
= 21,
x2
+ 3y3
+ 4z2
= 101.
65
Viết đề tài giá sinh viên – ZALO:0973.287.149-TEAMLUANVAN.COM
C®ng tàng ve hai phương trình trên ta được 2x2
+ 2y2
+ 4z2
+ t2
= 122. Suy
ra
2(x2
+ y2
+ 2z2
+ t2
) = 122 + t2
.
Tà đó suy ra f(x, y, z, t) ≥ 61 với moi (x, y, z, t) ∈ D. Đȁng thác xảy ra khi
t = 0.
Xét h phương trình
H này tương đương với
t = 0
x2
− y2
+ t2
= 21
x2
+ 3y3
+ 4z2
= 101
x, y, z ∈ N.
t = 0
x2
− y2
= 21
x2
+ 3y3
+ 4z2
= 101
x, y, z ∈ N.
Vì x2
− y2
= 21 và x, y nguyên dương nên
x + y = 21
x − y = 1
ho c
x + y = 7
x − y = 3.
Suy ra x = 11, y = 10 ho c x = 5, y = 2.
Tà x2
+ 3y3
+ 4z2
= 101, ta có x = 5, y = 2, z = 4. Như v y (5, 2, 4, 0) ∈
D, f(5, 2, 4, 0) = 61 .
V y max
(x,y,x)∈D
f(x, y, z) = 61.
Bài toán 3.14 (xem [2]). Tìm giá trị nhỏ nhat của hàm so
f(x) = |5x2
+ 11xy − 5y2
|
(
(
66
Viết đề tài giá sinh viên – ZALO:0973.287.149-TEAMLUANVAN.COM
2 2
. .
2 2
.5x2
+ 11xy − 5y2. /= 1
.
2 2
trên mien D = {(x, y) : x, y ∈ N; y là so lẻ }.
Lài giai.
Lay (x, y) tùy ý thu®c D, ta có y là so tự nhiên lẻ nên 5y2
là so lẻ.
Neu x là so chȁn thì 5x2
+ 11xy là so chȁn, suy ra 5x2
+ 11xy − 5y2
là so
lẻ.
Neu x là so lẻ thì 5x2
, 11xy là so lẻ , suy ra 5x2
+ 11xy − 5y2
là so lẻ.
V y trong moi trường hợp, ta đeu có 5x2
+ 11xy − 5y2
là so lẻ.
Ta sě cháng minh |5x2
+ 11xy − 5y2
| = 1. Th t v y, giả sả
Suy ra
|5x + 11xy − 5y | = 1.
20 .5x2
+ 11xy − 5y2. = 20
hay
⇔| (10x + 11y) − 221y | = 20,
(10x + 11y)2
− 221y2
= 20 ho c (10x + 11y)2
− 221y2
= −20.
Do 221 = 13.17, bình phương m®t so nguyên tùy ý đong dư với 1, 3,
4, -1, -3, -4 theo modulo 13, 20 đong dư 7 theo modulo 13, -20 đong dư -7
theo modulo 13 nên không ton tại x, y thỏa mãn hai đȁng thác trên. V y
L p lu n tương tự ta cũng có 5x2
+ 11xy − 5y2
=
/ 3.
Ket hợp với |5x2
+ 11xy − 5y2
| là so lẻ, nên suy ra
|5x + 11xy − 5y | ≥ 5.
Tác là ta có f(x, y) ≥ 5 với moi (x, y) thu®c D, lại có (0, 1) ∈ D, f(0, 1) = 5.
V y giá trị nhỏ nhat của hàm f(x, y) trên mien D là 5 đạt được khi
(x, y) = (0, 1).
67
Viết đề tài giá sinh viên – ZALO:0973.287.149-TEAMLUANVAN.COM
−
−
Bài toán 3.15 (xem [2]). Tìm giá trị lớn nhat và nhỏ nhat của hàm so
f(x, y, z) =
x 2z
+
y t
trên mien D = {(x, y, z) : x, y, z, t ∈ Z; 1 ≤ x ≤ y ≤ z ≤ t ≤ 100}.
Lài giai.
1. Lay (x, y, z, t) tùy ý thu®c D. Do 1 ≤ x ≤ y ≤ z ≤ t ≤ t nên
x
y
≤ 1,
2z
t
≤ 2.
Suy ra f(x, y, z, t) ≤ 3 với moi (x, y, z, t) thu®c D. Rõ ràng (1, 1, 1, 1) ∈ D
và f(1, 1, 1, 1) = 3. V y
max
(x,y,z,t)∈D
f(x, y, z, t) = 3.
2. Lay (x, y, z, t) tùy ý thu®c D. Do 1 ≤ x ≤ y ≤ z ≤ t ≤ 100 nên
x z 1 y
hay
f(x, y, z, t) = + 2 ≥
y t
1
+ 2
y 100
y
f(x, y, z, t) ≥
1 y
+ .
y 50
Xét hàm so g(x) =
thay
a. Neu y ≤ 7 thì
y
+
50
với 1 ≤ y ≤ 100 và y là so nguyên. Ta nh n
Suy ra
g(y) g(7) =
(7 − y)(50 − 7y)
350y
≥ 0.
g(y) ≥ g(7), ∀y = 1, 2, 3, 4, 5, 6, 7.
b. Neu y ≥ 8 thì
g(y) g(8) =
(8 − y)(50 − 8y)
350y
≥ 0.
68
Viết đề tài giá sinh viên – ZALO:0973.287.149-TEAMLUANVAN.COM
Suy ra
g(y) ≥ g(8), ∀y = 9, 10, . . . , 100.
Tà đó suy ra g(y) ≥ min g(7), g(8), ∀y = 1, 2, . . . .., 100 hay
Tà đó, ta có
g(y) ≥ min {g(7), g(8)} = min {
99
99
,
350
114
400
} =
99
.
350
và f(1, 7, 7, 100) =
f(x, y, z, t) ≥
99
. Do đó
350
350
, ∀(x, y, z, t) ∈ D
90
min
(x,y,z,t)∈D
f(x, y, z, t) = .
350
69
Viết đề tài giá sinh viên – ZALO:0973.287.149-TEAMLUANVAN.COM
Ket lu n
Lu n văn “Bat đȁng thác và bài toán cực trị trong lớp các đa thác
và phân thác h so nguyên ” đã giải quyet được nhǎng van đe sau:
1. Lu n văn đã trình bày chi tiet m®t so dạng toán ve đa thác và
phân thác với h so nguyên.
2. Trình bày các dạng toán ve ước lượng mien giá trị của đa thác
và phân thác đại so và m®t so bài toán cực trị tương áng.
3. Cuoi cùng, lu n văn trình bày m®t so bài toán ve bat đȁng thác
và cực trị trên t p so nguyên trong các đe toán thi hoc sinh giỏi
trong nước, Olympic khu vực và quoc te.
Viết đề tài giá sinh viên – ZALO:0973.287.149-TEAMLUANVAN.COM
Tài li u tham khảo
[A] Tieng Vi t
[1] Tran Xuân Đáng (2010), Đa thúc với các h so nguyên và đong dư thúc,
K yeu ”Chuyên đe boi dưỡng hè 2010”, ĐHKHTN, Hà N®i, 2010.
[2] Phan Huy Khải (2008), Giá tr lớn nhat và nhó nhat của hàm so, NXB
Giáo dục.
[3] Nguyen Vũ Lương (2006), Các bài giảng đay đủ ve bat đȁng thúc Cauchy,
NXB ĐHQGHN.
[4] Nguyen Văn M u (2003), Đa thúc đại so và phân thúc hũu ty, NXB Giáo
dục.
[5] Nguyen Văn M u, Lê Ngoc Lăng, Phạm the Long, Nguyen Minh Tuan
(2006), Các đe thi olympic Toán sinh viên toàn quoc, NXB Giáo dục.
[6] Nguyen Văn M u, Nguyen Văn Ngoc (2009), Đa thúc đoi xúng và áp
dựng, NXB Giáo dục.
[B] Tieng Anh
[7] Victor Prasolov (2001), Polynomial in serie Algorithms and Computation
in Mathematics, Vol.11, Springer-Verlag, Berlin-Heidelberg, 2010.
[8] Borcea, J., R. Pereira, M. Putinar (2009), Hausdorff geometry of complex
polynomials, positive charge distributions and normal operators, June 29,
2008-July 6, 2008.

More Related Content

Similar to Bat Đang Thức Và Bài Toán Cực Tr± Trong L P Các Đa Thức Và Phân Thức H So Nguyên.docx

Bài Toán Cực Trị Với Điều Kiện Ràng Buộc Bất Đẳng Thức, Hệ Bất Đẳng Thức.docx
Bài Toán Cực Trị Với Điều Kiện Ràng Buộc Bất Đẳng Thức, Hệ Bất Đẳng Thức.docxBài Toán Cực Trị Với Điều Kiện Ràng Buộc Bất Đẳng Thức, Hệ Bất Đẳng Thức.docx
Bài Toán Cực Trị Với Điều Kiện Ràng Buộc Bất Đẳng Thức, Hệ Bất Đẳng Thức.docxDV Viết Luận văn luanvanmaster.com ZALO 0973287149
 
Hàm Đơn Đi›U, Tựa Đơn Đi›U Và Một Số Ứng Dụng Của Phép Đơn Đi›U Hóa Hàm Số.docx
Hàm Đơn Đi›U, Tựa Đơn Đi›U Và Một Số Ứng Dụng Của Phép Đơn Đi›U Hóa Hàm Số.docxHàm Đơn Đi›U, Tựa Đơn Đi›U Và Một Số Ứng Dụng Của Phép Đơn Đi›U Hóa Hàm Số.docx
Hàm Đơn Đi›U, Tựa Đơn Đi›U Và Một Số Ứng Dụng Của Phép Đơn Đi›U Hóa Hàm Số.docxDV Viết Luận văn luanvanmaster.com ZALO 0973287149
 
Một số dạng toán về đa thức qua các kỳ thi Olympic 6732069.pdf
Một số dạng toán về đa thức qua các kỳ thi Olympic 6732069.pdfMột số dạng toán về đa thức qua các kỳ thi Olympic 6732069.pdf
Một số dạng toán về đa thức qua các kỳ thi Olympic 6732069.pdfTieuNgocLy
 

Similar to Bat Đang Thức Và Bài Toán Cực Tr± Trong L P Các Đa Thức Và Phân Thức H So Nguyên.docx (20)

Phương Pháp Lư Ng Giác Giải Phương Trình Đa Thức Và M T So Dạng Toán.docx
Phương Pháp Lư Ng Giác Giải Phương Trình Đa Thức Và M T So Dạng Toán.docxPhương Pháp Lư Ng Giác Giải Phương Trình Đa Thức Và M T So Dạng Toán.docx
Phương Pháp Lư Ng Giác Giải Phương Trình Đa Thức Và M T So Dạng Toán.docx
 
Bài Toán Cực Trị Với Điều Kiện Ràng Buộc Bất Đẳng Thức, Hệ Bất Đẳng Thức.docx
Bài Toán Cực Trị Với Điều Kiện Ràng Buộc Bất Đẳng Thức, Hệ Bất Đẳng Thức.docxBài Toán Cực Trị Với Điều Kiện Ràng Buộc Bất Đẳng Thức, Hệ Bất Đẳng Thức.docx
Bài Toán Cực Trị Với Điều Kiện Ràng Buộc Bất Đẳng Thức, Hệ Bất Đẳng Thức.docx
 
M T So L P Phương Trình Diophantine.docx
M T So L P Phương Trình Diophantine.docxM T So L P Phương Trình Diophantine.docx
M T So L P Phương Trình Diophantine.docx
 
Hàm Đơn Đi›U, Tựa Đơn Đi›U Và Một Số Ứng Dụng Của Phép Đơn Đi›U Hóa Hàm Số.docx
Hàm Đơn Đi›U, Tựa Đơn Đi›U Và Một Số Ứng Dụng Của Phép Đơn Đi›U Hóa Hàm Số.docxHàm Đơn Đi›U, Tựa Đơn Đi›U Và Một Số Ứng Dụng Của Phép Đơn Đi›U Hóa Hàm Số.docx
Hàm Đơn Đi›U, Tựa Đơn Đi›U Và Một Số Ứng Dụng Của Phép Đơn Đi›U Hóa Hàm Số.docx
 
Một số dạng toán về đa thức qua các kỳ thi Olympic 6732069.pdf
Một số dạng toán về đa thức qua các kỳ thi Olympic 6732069.pdfMột số dạng toán về đa thức qua các kỳ thi Olympic 6732069.pdf
Một số dạng toán về đa thức qua các kỳ thi Olympic 6732069.pdf
 
M T So Dạng Toán Ve Dãy So Sinh B I Các Hàm So Sơ Cap.docx
M T So Dạng Toán Ve Dãy So Sinh B I Các Hàm So Sơ Cap.docxM T So Dạng Toán Ve Dãy So Sinh B I Các Hàm So Sơ Cap.docx
M T So Dạng Toán Ve Dãy So Sinh B I Các Hàm So Sơ Cap.docx
 
Về Tổng Gauss Và Một Số Ứng Dụng.docx
Về Tổng Gauss Và Một Số Ứng Dụng.docxVề Tổng Gauss Và Một Số Ứng Dụng.docx
Về Tổng Gauss Và Một Số Ứng Dụng.docx
 
Toán Tử Sai Phân Và Ứng Dụng Vào Giải Toán Sơ Cấp.docx
Toán Tử Sai Phân Và Ứng Dụng Vào Giải Toán Sơ Cấp.docxToán Tử Sai Phân Và Ứng Dụng Vào Giải Toán Sơ Cấp.docx
Toán Tử Sai Phân Và Ứng Dụng Vào Giải Toán Sơ Cấp.docx
 
Bat Đang Thức V I Hàm Loi B Ph N Và Ứng Dụng.docx
Bat Đang Thức V I Hàm Loi B Ph N Và Ứng Dụng.docxBat Đang Thức V I Hàm Loi B Ph N Và Ứng Dụng.docx
Bat Đang Thức V I Hàm Loi B Ph N Và Ứng Dụng.docx
 
M T So Dạng Toán Cực Tr± Trong L P Hàm Mũ Và Hàm Hyperbolic.docx
M T So Dạng Toán Cực Tr± Trong L P Hàm Mũ Và Hàm Hyperbolic.docxM T So Dạng Toán Cực Tr± Trong L P Hàm Mũ Và Hàm Hyperbolic.docx
M T So Dạng Toán Cực Tr± Trong L P Hàm Mũ Và Hàm Hyperbolic.docx
 
Đang thức, bat đang thức tích phân trong l p đa thức và phân thức hữu ty và m...
Đang thức, bat đang thức tích phân trong l p đa thức và phân thức hữu ty và m...Đang thức, bat đang thức tích phân trong l p đa thức và phân thức hữu ty và m...
Đang thức, bat đang thức tích phân trong l p đa thức và phân thức hữu ty và m...
 
Bất đẳng thức Trong lớp các hàm lượng giác và lượng giác ngược.docx
Bất đẳng thức Trong lớp các hàm lượng giác và lượng giác ngược.docxBất đẳng thức Trong lớp các hàm lượng giác và lượng giác ngược.docx
Bất đẳng thức Trong lớp các hàm lượng giác và lượng giác ngược.docx
 
Ve H Phương Trình Phi Tuyen Và Ứng Dụng.docx
Ve H Phương Trình Phi Tuyen Và Ứng Dụng.docxVe H Phương Trình Phi Tuyen Và Ứng Dụng.docx
Ve H Phương Trình Phi Tuyen Và Ứng Dụng.docx
 
M t so dạng toán Liên quan đen xác suat r i rạc và ứng dụng.docx
M t so dạng toán Liên quan đen xác suat r i rạc  và ứng dụng.docxM t so dạng toán Liên quan đen xác suat r i rạc  và ứng dụng.docx
M t so dạng toán Liên quan đen xác suat r i rạc và ứng dụng.docx
 
M T So Dạng Toán Liên Quan Đen Xác Suat R I Rạc Và Ứng Dụng.docx
M T So Dạng Toán Liên Quan Đen Xác Suat R I Rạc Và Ứng Dụng.docxM T So Dạng Toán Liên Quan Đen Xác Suat R I Rạc Và Ứng Dụng.docx
M T So Dạng Toán Liên Quan Đen Xác Suat R I Rạc Và Ứng Dụng.docx
 
Bài Toán Đổi Tiền Của Frobenius.docx
Bài Toán Đổi Tiền Của Frobenius.docxBài Toán Đổi Tiền Của Frobenius.docx
Bài Toán Đổi Tiền Của Frobenius.docx
 
Kĩ thuật tổng hợp Giải bất phương trình hỗn hợp.docx
Kĩ thuật tổng hợp Giải bất phương trình hỗn hợp.docxKĩ thuật tổng hợp Giải bất phương trình hỗn hợp.docx
Kĩ thuật tổng hợp Giải bất phương trình hỗn hợp.docx
 
Ve Bat Đang Thức Ho¨ Lder Và Áp Dụng.docx
Ve  Bat  Đang  Thức  Ho¨ Lder  Và  Áp  Dụng.docxVe  Bat  Đang  Thức  Ho¨ Lder  Và  Áp  Dụng.docx
Ve Bat Đang Thức Ho¨ Lder Và Áp Dụng.docx
 
Luận văn: Phép biến đổi phân tuyến tính, HAY, 9đ
Luận văn: Phép biến đổi phân tuyến tính, HAY, 9đLuận văn: Phép biến đổi phân tuyến tính, HAY, 9đ
Luận văn: Phép biến đổi phân tuyến tính, HAY, 9đ
 
Một số phương pháp giải các đề thi olympic Về phương trình diophant.docx
Một số phương pháp giải các đề thi olympic Về phương trình diophant.docxMột số phương pháp giải các đề thi olympic Về phương trình diophant.docx
Một số phương pháp giải các đề thi olympic Về phương trình diophant.docx
 

More from DV Viết Luận văn luanvanmaster.com ZALO 0973287149

More from DV Viết Luận văn luanvanmaster.com ZALO 0973287149 (20)

Ảnh Hưởng Của Marketing Quan Hệ Đến Lòng Trung Thành Của Khách Hàng.Tình Huốn...
Ảnh Hưởng Của Marketing Quan Hệ Đến Lòng Trung Thành Của Khách Hàng.Tình Huốn...Ảnh Hưởng Của Marketing Quan Hệ Đến Lòng Trung Thành Của Khách Hàng.Tình Huốn...
Ảnh Hưởng Của Marketing Quan Hệ Đến Lòng Trung Thành Của Khách Hàng.Tình Huốn...
 
Phát triển nguồn nhân lực tại Uỷ ban nhân dân huyện Trà Bồng, tỉnh Quảng Ngãi...
Phát triển nguồn nhân lực tại Uỷ ban nhân dân huyện Trà Bồng, tỉnh Quảng Ngãi...Phát triển nguồn nhân lực tại Uỷ ban nhân dân huyện Trà Bồng, tỉnh Quảng Ngãi...
Phát triển nguồn nhân lực tại Uỷ ban nhân dân huyện Trà Bồng, tỉnh Quảng Ngãi...
 
Báo cáo tốt Nghiệp tài chính hợp nhất tại tổng công ty Indochina gol...
Báo cáo tốt Nghiệp  tài chính hợp nhất tại tổng công ty Indochina gol...Báo cáo tốt Nghiệp  tài chính hợp nhất tại tổng công ty Indochina gol...
Báo cáo tốt Nghiệp tài chính hợp nhất tại tổng công ty Indochina gol...
 
Tạo động lực thúc đẩy nhân viên làm việc tại ngân hàng TMCP Ngoại Thương Việt...
Tạo động lực thúc đẩy nhân viên làm việc tại ngân hàng TMCP Ngoại Thương Việt...Tạo động lực thúc đẩy nhân viên làm việc tại ngân hàng TMCP Ngoại Thương Việt...
Tạo động lực thúc đẩy nhân viên làm việc tại ngân hàng TMCP Ngoại Thương Việt...
 
Phát triển công nghiệp trên địa bàn Thành phố Tam Kỳ, Tỉnh Quảng Na...
Phát triển công nghiệp trên địa bàn Thành phố Tam Kỳ, Tỉnh Quảng Na...Phát triển công nghiệp trên địa bàn Thành phố Tam Kỳ, Tỉnh Quảng Na...
Phát triển công nghiệp trên địa bàn Thành phố Tam Kỳ, Tỉnh Quảng Na...
 
Giải pháp phát triển cho vay xuất nhập khẩu tại ngân hàng NN&PTNN ch...
Giải pháp phát triển cho vay xuất nhập khẩu tại ngân hàng NN&PTNN ch...Giải pháp phát triển cho vay xuất nhập khẩu tại ngân hàng NN&PTNN ch...
Giải pháp phát triển cho vay xuất nhập khẩu tại ngân hàng NN&PTNN ch...
 
Hoàn thiện công tác lập báo cáo tài chính hợp nhất tại tổng công ...
Hoàn thiện công tác lập báo cáo tài chính hợp nhất tại tổng công ...Hoàn thiện công tác lập báo cáo tài chính hợp nhất tại tổng công ...
Hoàn thiện công tác lập báo cáo tài chính hợp nhất tại tổng công ...
 
Luận Văn Thạc Sĩ Quản trị thành tích nhân viên tại Cục Hải quan TP Đà Nẵng.doc
Luận Văn Thạc Sĩ  Quản trị thành tích nhân viên tại Cục Hải quan TP Đà Nẵng.docLuận Văn Thạc Sĩ  Quản trị thành tích nhân viên tại Cục Hải quan TP Đà Nẵng.doc
Luận Văn Thạc Sĩ Quản trị thành tích nhân viên tại Cục Hải quan TP Đà Nẵng.doc
 
Hoàn thiện công tác quản lý thuế thu nhập cá nhân tại cục thuế Tỉ...
Hoàn thiện công tác quản lý thuế thu nhập cá nhân tại cục thuế Tỉ...Hoàn thiện công tác quản lý thuế thu nhập cá nhân tại cục thuế Tỉ...
Hoàn thiện công tác quản lý thuế thu nhập cá nhân tại cục thuế Tỉ...
 
Đề Tài Phát triển bền vững nông nghiệp Huyện Ba Tơ, Tỉnh Quảng Ngãi....
Đề Tài Phát triển bền vững nông nghiệp Huyện Ba Tơ, Tỉnh Quảng Ngãi....Đề Tài Phát triển bền vững nông nghiệp Huyện Ba Tơ, Tỉnh Quảng Ngãi....
Đề Tài Phát triển bền vững nông nghiệp Huyện Ba Tơ, Tỉnh Quảng Ngãi....
 
Hoàn thiện công tác bảo trợ xã hội trên địa bàn huyện Phong Điền, tỉnh Thừa T...
Hoàn thiện công tác bảo trợ xã hội trên địa bàn huyện Phong Điền, tỉnh Thừa T...Hoàn thiện công tác bảo trợ xã hội trên địa bàn huyện Phong Điền, tỉnh Thừa T...
Hoàn thiện công tác bảo trợ xã hội trên địa bàn huyện Phong Điền, tỉnh Thừa T...
 
Đề Tài Luận VănPhát triển sản phẩm du lịch tại thành phố Đà Nẵng.doc
Đề Tài Luận VănPhát triển sản phẩm du lịch tại thành phố Đà Nẵng.docĐề Tài Luận VănPhát triển sản phẩm du lịch tại thành phố Đà Nẵng.doc
Đề Tài Luận VănPhát triển sản phẩm du lịch tại thành phố Đà Nẵng.doc
 
Đào tạo nghề cho lao động thuộc diện thu hồi đất trên địa bàn Thàn...
Đào tạo nghề cho lao động thuộc diện thu hồi đất trên địa bàn Thàn...Đào tạo nghề cho lao động thuộc diện thu hồi đất trên địa bàn Thàn...
Đào tạo nghề cho lao động thuộc diện thu hồi đất trên địa bàn Thàn...
 
Tóm Tắt Luận Văn Thạc Sĩ Quản Trị Kinh Doanh Xây dựng chính sách Marketing tạ...
Tóm Tắt Luận Văn Thạc Sĩ Quản Trị Kinh Doanh Xây dựng chính sách Marketing tạ...Tóm Tắt Luận Văn Thạc Sĩ Quản Trị Kinh Doanh Xây dựng chính sách Marketing tạ...
Tóm Tắt Luận Văn Thạc Sĩ Quản Trị Kinh Doanh Xây dựng chính sách Marketing tạ...
 
Đề Tài Nghiên cứu rủi ro cảm nhận đối với mua hàng thời trang trực tuyến.docx
Đề Tài Nghiên cứu rủi ro cảm nhận đối với mua hàng thời trang trực tuyến.docxĐề Tài Nghiên cứu rủi ro cảm nhận đối với mua hàng thời trang trực tuyến.docx
Đề Tài Nghiên cứu rủi ro cảm nhận đối với mua hàng thời trang trực tuyến.docx
 
Giải pháp nâng cao động lực thúc đẩy người lao động tại công ty khai...
Giải pháp nâng cao động lực thúc đẩy người lao động tại công ty khai...Giải pháp nâng cao động lực thúc đẩy người lao động tại công ty khai...
Giải pháp nâng cao động lực thúc đẩy người lao động tại công ty khai...
 
Giải pháp phát triển dịch vụ ngân hàng điện tử tại ngân hàng đầu ...
Giải pháp phát triển dịch vụ ngân hàng điện tử tại ngân hàng đầu ...Giải pháp phát triển dịch vụ ngân hàng điện tử tại ngân hàng đầu ...
Giải pháp phát triển dịch vụ ngân hàng điện tử tại ngân hàng đầu ...
 
Giải pháp phát triển dịch vụ ngân hàng điện tử tại ngân hàng đầu ...
Giải pháp phát triển dịch vụ ngân hàng điện tử tại ngân hàng đầu ...Giải pháp phát triển dịch vụ ngân hàng điện tử tại ngân hàng đầu ...
Giải pháp phát triển dịch vụ ngân hàng điện tử tại ngân hàng đầu ...
 
Quản trị quan hệ khách hàng tại Chi nhánh Viettel Đà Nẵng – Tập đoàn Viễn thô...
Quản trị quan hệ khách hàng tại Chi nhánh Viettel Đà Nẵng – Tập đoàn Viễn thô...Quản trị quan hệ khách hàng tại Chi nhánh Viettel Đà Nẵng – Tập đoàn Viễn thô...
Quản trị quan hệ khách hàng tại Chi nhánh Viettel Đà Nẵng – Tập đoàn Viễn thô...
 
Đề Tài Đánh giá thành tích đội ngũ giảng viên trường Đại Học Phạm ...
Đề Tài Đánh giá thành tích đội ngũ giảng viên trường Đại Học Phạm ...Đề Tài Đánh giá thành tích đội ngũ giảng viên trường Đại Học Phạm ...
Đề Tài Đánh giá thành tích đội ngũ giảng viên trường Đại Học Phạm ...
 

Recently uploaded

Phân tích báo cáo tài chính tại công ty TNHH xây dựng và thương mại Thịnh An
Phân tích báo cáo tài chính tại công ty TNHH xây dựng và thương mại Thịnh AnPhân tích báo cáo tài chính tại công ty TNHH xây dựng và thương mại Thịnh An
Phân tích báo cáo tài chính tại công ty TNHH xây dựng và thương mại Thịnh Anlamluanvan.net Viết thuê luận văn
 
Báo cáo tốt nghiệp Đánh giá rủi ro quá trình xử lí nước cấp tại Chi nhánh Cấp...
Báo cáo tốt nghiệp Đánh giá rủi ro quá trình xử lí nước cấp tại Chi nhánh Cấp...Báo cáo tốt nghiệp Đánh giá rủi ro quá trình xử lí nước cấp tại Chi nhánh Cấp...
Báo cáo tốt nghiệp Đánh giá rủi ro quá trình xử lí nước cấp tại Chi nhánh Cấp...lamluanvan.net Viết thuê luận văn
 
Báo cáo tốt nghiệp Hoàn thiện an toàn lao động điện công ty trách nhiệm hữu h...
Báo cáo tốt nghiệp Hoàn thiện an toàn lao động điện công ty trách nhiệm hữu h...Báo cáo tốt nghiệp Hoàn thiện an toàn lao động điện công ty trách nhiệm hữu h...
Báo cáo tốt nghiệp Hoàn thiện an toàn lao động điện công ty trách nhiệm hữu h...lamluanvan.net Viết thuê luận văn
 
35 ĐỀ LUYỆN THI ĐÁNH GIÁ NĂNG LỰC ĐẠI HỌC QUỐC GIA THÀNH PHỐ HỒ CHÍ MINH NĂM ...
35 ĐỀ LUYỆN THI ĐÁNH GIÁ NĂNG LỰC ĐẠI HỌC QUỐC GIA THÀNH PHỐ HỒ CHÍ MINH NĂM ...35 ĐỀ LUYỆN THI ĐÁNH GIÁ NĂNG LỰC ĐẠI HỌC QUỐC GIA THÀNH PHỐ HỒ CHÍ MINH NĂM ...
35 ĐỀ LUYỆN THI ĐÁNH GIÁ NĂNG LỰC ĐẠI HỌC QUỐC GIA THÀNH PHỐ HỒ CHÍ MINH NĂM ...Nguyen Thanh Tu Collection
 
Hoàn thiện công tác kiểm soát chi NSNN qua Kho bạc Nhà nước huyện Tri Tôn – t...
Hoàn thiện công tác kiểm soát chi NSNN qua Kho bạc Nhà nước huyện Tri Tôn – t...Hoàn thiện công tác kiểm soát chi NSNN qua Kho bạc Nhà nước huyện Tri Tôn – t...
Hoàn thiện công tác kiểm soát chi NSNN qua Kho bạc Nhà nước huyện Tri Tôn – t...lamluanvan.net Viết thuê luận văn
 
TỔNG HỢP HƠN 100 ĐỀ THI THỬ TỐT NGHIỆP THPT TIẾNG ANH 2024 - TỪ CÁC TRƯỜNG, ...
TỔNG HỢP HƠN 100 ĐỀ THI THỬ TỐT NGHIỆP THPT TIẾNG ANH 2024 - TỪ CÁC TRƯỜNG, ...TỔNG HỢP HƠN 100 ĐỀ THI THỬ TỐT NGHIỆP THPT TIẾNG ANH 2024 - TỪ CÁC TRƯỜNG, ...
TỔNG HỢP HƠN 100 ĐỀ THI THỬ TỐT NGHIỆP THPT TIẾNG ANH 2024 - TỪ CÁC TRƯỜNG, ...Nguyen Thanh Tu Collection
 
40 ĐỀ LUYỆN THI ĐÁNH GIÁ NĂNG LỰC ĐẠI HỌC QUỐC GIA HÀ NỘI NĂM 2024 (ĐỀ 1-20) ...
40 ĐỀ LUYỆN THI ĐÁNH GIÁ NĂNG LỰC ĐẠI HỌC QUỐC GIA HÀ NỘI NĂM 2024 (ĐỀ 1-20) ...40 ĐỀ LUYỆN THI ĐÁNH GIÁ NĂNG LỰC ĐẠI HỌC QUỐC GIA HÀ NỘI NĂM 2024 (ĐỀ 1-20) ...
40 ĐỀ LUYỆN THI ĐÁNH GIÁ NĂNG LỰC ĐẠI HỌC QUỐC GIA HÀ NỘI NĂM 2024 (ĐỀ 1-20) ...Nguyen Thanh Tu Collection
 
Báo cáo tốt nghiệp Hoàn thiện công tác đào tạo và phát triển nguồn nhân lực c...
Báo cáo tốt nghiệp Hoàn thiện công tác đào tạo và phát triển nguồn nhân lực c...Báo cáo tốt nghiệp Hoàn thiện công tác đào tạo và phát triển nguồn nhân lực c...
Báo cáo tốt nghiệp Hoàn thiện công tác đào tạo và phát triển nguồn nhân lực c...lamluanvan.net Viết thuê luận văn
 
Nhân tố ảnh hưởng tới động lực làm việc của kiểm toán viên tại Chi nhánh Công...
Nhân tố ảnh hưởng tới động lực làm việc của kiểm toán viên tại Chi nhánh Công...Nhân tố ảnh hưởng tới động lực làm việc của kiểm toán viên tại Chi nhánh Công...
Nhân tố ảnh hưởng tới động lực làm việc của kiểm toán viên tại Chi nhánh Công...lamluanvan.net Viết thuê luận văn
 
Báo cáo tốt nghiệp Đánh giá rủi ro môi trường ô nhiễm hữu cơ trong nước thải ...
Báo cáo tốt nghiệp Đánh giá rủi ro môi trường ô nhiễm hữu cơ trong nước thải ...Báo cáo tốt nghiệp Đánh giá rủi ro môi trường ô nhiễm hữu cơ trong nước thải ...
Báo cáo tốt nghiệp Đánh giá rủi ro môi trường ô nhiễm hữu cơ trong nước thải ...lamluanvan.net Viết thuê luận văn
 
40 ĐỀ LUYỆN THI ĐÁNH GIÁ NĂNG LỰC ĐẠI HỌC QUỐC GIA HÀ NỘI NĂM 2024 (ĐỀ 31-39)...
40 ĐỀ LUYỆN THI ĐÁNH GIÁ NĂNG LỰC ĐẠI HỌC QUỐC GIA HÀ NỘI NĂM 2024 (ĐỀ 31-39)...40 ĐỀ LUYỆN THI ĐÁNH GIÁ NĂNG LỰC ĐẠI HỌC QUỐC GIA HÀ NỘI NĂM 2024 (ĐỀ 31-39)...
40 ĐỀ LUYỆN THI ĐÁNH GIÁ NĂNG LỰC ĐẠI HỌC QUỐC GIA HÀ NỘI NĂM 2024 (ĐỀ 31-39)...Nguyen Thanh Tu Collection
 
Báo cáo thực tập tốt nghiệp Phân tích hiệu quả hoạt động huy động và cho vay ...
Báo cáo thực tập tốt nghiệp Phân tích hiệu quả hoạt động huy động và cho vay ...Báo cáo thực tập tốt nghiệp Phân tích hiệu quả hoạt động huy động và cho vay ...
Báo cáo thực tập tốt nghiệp Phân tích hiệu quả hoạt động huy động và cho vay ...lamluanvan.net Viết thuê luận văn
 
Báo cáo tốt nghiệp Kế toán tiền gửi ngân hàng tại công ty TNHH Một Thành Viên...
Báo cáo tốt nghiệp Kế toán tiền gửi ngân hàng tại công ty TNHH Một Thành Viên...Báo cáo tốt nghiệp Kế toán tiền gửi ngân hàng tại công ty TNHH Một Thành Viên...
Báo cáo tốt nghiệp Kế toán tiền gửi ngân hàng tại công ty TNHH Một Thành Viên...lamluanvan.net Viết thuê luận văn
 
Hệ thống ca dao than thân người Việt từ góc nhìn thi pháp
Hệ thống ca dao than thân người Việt từ góc nhìn thi phápHệ thống ca dao than thân người Việt từ góc nhìn thi pháp
Hệ thống ca dao than thân người Việt từ góc nhìn thi pháplamluanvan.net Viết thuê luận văn
 
TỔNG HỢP HƠN 100 ĐỀ THI THỬ TỐT NGHIỆP THPT VẬT LÝ 2024 - TỪ CÁC TRƯỜNG, TRƯ...
TỔNG HỢP HƠN 100 ĐỀ THI THỬ TỐT NGHIỆP THPT VẬT LÝ 2024 - TỪ CÁC TRƯỜNG, TRƯ...TỔNG HỢP HƠN 100 ĐỀ THI THỬ TỐT NGHIỆP THPT VẬT LÝ 2024 - TỪ CÁC TRƯỜNG, TRƯ...
TỔNG HỢP HƠN 100 ĐỀ THI THỬ TỐT NGHIỆP THPT VẬT LÝ 2024 - TỪ CÁC TRƯỜNG, TRƯ...Nguyen Thanh Tu Collection
 
30 ĐỀ PHÁT TRIỂN THEO CẤU TRÚC ĐỀ MINH HỌA BGD NGÀY 22-3-2024 KỲ THI TỐT NGHI...
30 ĐỀ PHÁT TRIỂN THEO CẤU TRÚC ĐỀ MINH HỌA BGD NGÀY 22-3-2024 KỲ THI TỐT NGHI...30 ĐỀ PHÁT TRIỂN THEO CẤU TRÚC ĐỀ MINH HỌA BGD NGÀY 22-3-2024 KỲ THI TỐT NGHI...
30 ĐỀ PHÁT TRIỂN THEO CẤU TRÚC ĐỀ MINH HỌA BGD NGÀY 22-3-2024 KỲ THI TỐT NGHI...Nguyen Thanh Tu Collection
 
Báo cáo tốt nghiệp Đánh giá công tác đào tạo và phát triển nguồn nhân lực tại...
Báo cáo tốt nghiệp Đánh giá công tác đào tạo và phát triển nguồn nhân lực tại...Báo cáo tốt nghiệp Đánh giá công tác đào tạo và phát triển nguồn nhân lực tại...
Báo cáo tốt nghiệp Đánh giá công tác đào tạo và phát triển nguồn nhân lực tại...lamluanvan.net Viết thuê luận văn
 
Vận dụng thi pháp học vào phân tích truyện ngắn Chiếc thuyền ...
Vận dụng thi pháp học vào phân tích truyện ngắn Chiếc thuyền ...Vận dụng thi pháp học vào phân tích truyện ngắn Chiếc thuyền ...
Vận dụng thi pháp học vào phân tích truyện ngắn Chiếc thuyền ...lamluanvan.net Viết thuê luận văn
 
Báo cáo bài tập lớn E - Marketing Xây dựng kế hoạch marketing điện tử cho nhã...
Báo cáo bài tập lớn E - Marketing Xây dựng kế hoạch marketing điện tử cho nhã...Báo cáo bài tập lớn E - Marketing Xây dựng kế hoạch marketing điện tử cho nhã...
Báo cáo bài tập lớn E - Marketing Xây dựng kế hoạch marketing điện tử cho nhã...lamluanvan.net Viết thuê luận văn
 
Báo cáo thực tập tốt nghiệp Phân tích thực trạng hoạt động bán hàng tại Công ...
Báo cáo thực tập tốt nghiệp Phân tích thực trạng hoạt động bán hàng tại Công ...Báo cáo thực tập tốt nghiệp Phân tích thực trạng hoạt động bán hàng tại Công ...
Báo cáo thực tập tốt nghiệp Phân tích thực trạng hoạt động bán hàng tại Công ...lamluanvan.net Viết thuê luận văn
 

Recently uploaded (20)

Phân tích báo cáo tài chính tại công ty TNHH xây dựng và thương mại Thịnh An
Phân tích báo cáo tài chính tại công ty TNHH xây dựng và thương mại Thịnh AnPhân tích báo cáo tài chính tại công ty TNHH xây dựng và thương mại Thịnh An
Phân tích báo cáo tài chính tại công ty TNHH xây dựng và thương mại Thịnh An
 
Báo cáo tốt nghiệp Đánh giá rủi ro quá trình xử lí nước cấp tại Chi nhánh Cấp...
Báo cáo tốt nghiệp Đánh giá rủi ro quá trình xử lí nước cấp tại Chi nhánh Cấp...Báo cáo tốt nghiệp Đánh giá rủi ro quá trình xử lí nước cấp tại Chi nhánh Cấp...
Báo cáo tốt nghiệp Đánh giá rủi ro quá trình xử lí nước cấp tại Chi nhánh Cấp...
 
Báo cáo tốt nghiệp Hoàn thiện an toàn lao động điện công ty trách nhiệm hữu h...
Báo cáo tốt nghiệp Hoàn thiện an toàn lao động điện công ty trách nhiệm hữu h...Báo cáo tốt nghiệp Hoàn thiện an toàn lao động điện công ty trách nhiệm hữu h...
Báo cáo tốt nghiệp Hoàn thiện an toàn lao động điện công ty trách nhiệm hữu h...
 
35 ĐỀ LUYỆN THI ĐÁNH GIÁ NĂNG LỰC ĐẠI HỌC QUỐC GIA THÀNH PHỐ HỒ CHÍ MINH NĂM ...
35 ĐỀ LUYỆN THI ĐÁNH GIÁ NĂNG LỰC ĐẠI HỌC QUỐC GIA THÀNH PHỐ HỒ CHÍ MINH NĂM ...35 ĐỀ LUYỆN THI ĐÁNH GIÁ NĂNG LỰC ĐẠI HỌC QUỐC GIA THÀNH PHỐ HỒ CHÍ MINH NĂM ...
35 ĐỀ LUYỆN THI ĐÁNH GIÁ NĂNG LỰC ĐẠI HỌC QUỐC GIA THÀNH PHỐ HỒ CHÍ MINH NĂM ...
 
Hoàn thiện công tác kiểm soát chi NSNN qua Kho bạc Nhà nước huyện Tri Tôn – t...
Hoàn thiện công tác kiểm soát chi NSNN qua Kho bạc Nhà nước huyện Tri Tôn – t...Hoàn thiện công tác kiểm soát chi NSNN qua Kho bạc Nhà nước huyện Tri Tôn – t...
Hoàn thiện công tác kiểm soát chi NSNN qua Kho bạc Nhà nước huyện Tri Tôn – t...
 
TỔNG HỢP HƠN 100 ĐỀ THI THỬ TỐT NGHIỆP THPT TIẾNG ANH 2024 - TỪ CÁC TRƯỜNG, ...
TỔNG HỢP HƠN 100 ĐỀ THI THỬ TỐT NGHIỆP THPT TIẾNG ANH 2024 - TỪ CÁC TRƯỜNG, ...TỔNG HỢP HƠN 100 ĐỀ THI THỬ TỐT NGHIỆP THPT TIẾNG ANH 2024 - TỪ CÁC TRƯỜNG, ...
TỔNG HỢP HƠN 100 ĐỀ THI THỬ TỐT NGHIỆP THPT TIẾNG ANH 2024 - TỪ CÁC TRƯỜNG, ...
 
40 ĐỀ LUYỆN THI ĐÁNH GIÁ NĂNG LỰC ĐẠI HỌC QUỐC GIA HÀ NỘI NĂM 2024 (ĐỀ 1-20) ...
40 ĐỀ LUYỆN THI ĐÁNH GIÁ NĂNG LỰC ĐẠI HỌC QUỐC GIA HÀ NỘI NĂM 2024 (ĐỀ 1-20) ...40 ĐỀ LUYỆN THI ĐÁNH GIÁ NĂNG LỰC ĐẠI HỌC QUỐC GIA HÀ NỘI NĂM 2024 (ĐỀ 1-20) ...
40 ĐỀ LUYỆN THI ĐÁNH GIÁ NĂNG LỰC ĐẠI HỌC QUỐC GIA HÀ NỘI NĂM 2024 (ĐỀ 1-20) ...
 
Báo cáo tốt nghiệp Hoàn thiện công tác đào tạo và phát triển nguồn nhân lực c...
Báo cáo tốt nghiệp Hoàn thiện công tác đào tạo và phát triển nguồn nhân lực c...Báo cáo tốt nghiệp Hoàn thiện công tác đào tạo và phát triển nguồn nhân lực c...
Báo cáo tốt nghiệp Hoàn thiện công tác đào tạo và phát triển nguồn nhân lực c...
 
Nhân tố ảnh hưởng tới động lực làm việc của kiểm toán viên tại Chi nhánh Công...
Nhân tố ảnh hưởng tới động lực làm việc của kiểm toán viên tại Chi nhánh Công...Nhân tố ảnh hưởng tới động lực làm việc của kiểm toán viên tại Chi nhánh Công...
Nhân tố ảnh hưởng tới động lực làm việc của kiểm toán viên tại Chi nhánh Công...
 
Báo cáo tốt nghiệp Đánh giá rủi ro môi trường ô nhiễm hữu cơ trong nước thải ...
Báo cáo tốt nghiệp Đánh giá rủi ro môi trường ô nhiễm hữu cơ trong nước thải ...Báo cáo tốt nghiệp Đánh giá rủi ro môi trường ô nhiễm hữu cơ trong nước thải ...
Báo cáo tốt nghiệp Đánh giá rủi ro môi trường ô nhiễm hữu cơ trong nước thải ...
 
40 ĐỀ LUYỆN THI ĐÁNH GIÁ NĂNG LỰC ĐẠI HỌC QUỐC GIA HÀ NỘI NĂM 2024 (ĐỀ 31-39)...
40 ĐỀ LUYỆN THI ĐÁNH GIÁ NĂNG LỰC ĐẠI HỌC QUỐC GIA HÀ NỘI NĂM 2024 (ĐỀ 31-39)...40 ĐỀ LUYỆN THI ĐÁNH GIÁ NĂNG LỰC ĐẠI HỌC QUỐC GIA HÀ NỘI NĂM 2024 (ĐỀ 31-39)...
40 ĐỀ LUYỆN THI ĐÁNH GIÁ NĂNG LỰC ĐẠI HỌC QUỐC GIA HÀ NỘI NĂM 2024 (ĐỀ 31-39)...
 
Báo cáo thực tập tốt nghiệp Phân tích hiệu quả hoạt động huy động và cho vay ...
Báo cáo thực tập tốt nghiệp Phân tích hiệu quả hoạt động huy động và cho vay ...Báo cáo thực tập tốt nghiệp Phân tích hiệu quả hoạt động huy động và cho vay ...
Báo cáo thực tập tốt nghiệp Phân tích hiệu quả hoạt động huy động và cho vay ...
 
Báo cáo tốt nghiệp Kế toán tiền gửi ngân hàng tại công ty TNHH Một Thành Viên...
Báo cáo tốt nghiệp Kế toán tiền gửi ngân hàng tại công ty TNHH Một Thành Viên...Báo cáo tốt nghiệp Kế toán tiền gửi ngân hàng tại công ty TNHH Một Thành Viên...
Báo cáo tốt nghiệp Kế toán tiền gửi ngân hàng tại công ty TNHH Một Thành Viên...
 
Hệ thống ca dao than thân người Việt từ góc nhìn thi pháp
Hệ thống ca dao than thân người Việt từ góc nhìn thi phápHệ thống ca dao than thân người Việt từ góc nhìn thi pháp
Hệ thống ca dao than thân người Việt từ góc nhìn thi pháp
 
TỔNG HỢP HƠN 100 ĐỀ THI THỬ TỐT NGHIỆP THPT VẬT LÝ 2024 - TỪ CÁC TRƯỜNG, TRƯ...
TỔNG HỢP HƠN 100 ĐỀ THI THỬ TỐT NGHIỆP THPT VẬT LÝ 2024 - TỪ CÁC TRƯỜNG, TRƯ...TỔNG HỢP HƠN 100 ĐỀ THI THỬ TỐT NGHIỆP THPT VẬT LÝ 2024 - TỪ CÁC TRƯỜNG, TRƯ...
TỔNG HỢP HƠN 100 ĐỀ THI THỬ TỐT NGHIỆP THPT VẬT LÝ 2024 - TỪ CÁC TRƯỜNG, TRƯ...
 
30 ĐỀ PHÁT TRIỂN THEO CẤU TRÚC ĐỀ MINH HỌA BGD NGÀY 22-3-2024 KỲ THI TỐT NGHI...
30 ĐỀ PHÁT TRIỂN THEO CẤU TRÚC ĐỀ MINH HỌA BGD NGÀY 22-3-2024 KỲ THI TỐT NGHI...30 ĐỀ PHÁT TRIỂN THEO CẤU TRÚC ĐỀ MINH HỌA BGD NGÀY 22-3-2024 KỲ THI TỐT NGHI...
30 ĐỀ PHÁT TRIỂN THEO CẤU TRÚC ĐỀ MINH HỌA BGD NGÀY 22-3-2024 KỲ THI TỐT NGHI...
 
Báo cáo tốt nghiệp Đánh giá công tác đào tạo và phát triển nguồn nhân lực tại...
Báo cáo tốt nghiệp Đánh giá công tác đào tạo và phát triển nguồn nhân lực tại...Báo cáo tốt nghiệp Đánh giá công tác đào tạo và phát triển nguồn nhân lực tại...
Báo cáo tốt nghiệp Đánh giá công tác đào tạo và phát triển nguồn nhân lực tại...
 
Vận dụng thi pháp học vào phân tích truyện ngắn Chiếc thuyền ...
Vận dụng thi pháp học vào phân tích truyện ngắn Chiếc thuyền ...Vận dụng thi pháp học vào phân tích truyện ngắn Chiếc thuyền ...
Vận dụng thi pháp học vào phân tích truyện ngắn Chiếc thuyền ...
 
Báo cáo bài tập lớn E - Marketing Xây dựng kế hoạch marketing điện tử cho nhã...
Báo cáo bài tập lớn E - Marketing Xây dựng kế hoạch marketing điện tử cho nhã...Báo cáo bài tập lớn E - Marketing Xây dựng kế hoạch marketing điện tử cho nhã...
Báo cáo bài tập lớn E - Marketing Xây dựng kế hoạch marketing điện tử cho nhã...
 
Báo cáo thực tập tốt nghiệp Phân tích thực trạng hoạt động bán hàng tại Công ...
Báo cáo thực tập tốt nghiệp Phân tích thực trạng hoạt động bán hàng tại Công ...Báo cáo thực tập tốt nghiệp Phân tích thực trạng hoạt động bán hàng tại Công ...
Báo cáo thực tập tốt nghiệp Phân tích thực trạng hoạt động bán hàng tại Công ...
 

Bat Đang Thức Và Bài Toán Cực Tr± Trong L P Các Đa Thức Và Phân Thức H So Nguyên.docx

  • 1. Tải tài liệu tại sividoc.com Viết đề tài giá sinh viên – ZALO:0973.287.149-TEAMLUANVAN.COM ĐẠI HOC THÁI NGUYÊN TRƯ NG ĐẠI HOC KHOA HOC NGUYEN NGOC HÀ BAT ĐANG THỨC VÀ BÀI TOÁN CỰC TR± TRONG L P CÁC ĐA THỨC VÀ PHÂN THỨC H SO NGUYÊN LU N VĂN THẠC SỸ TOÁN HOC THÁI NGUYÊN - NĂM 2016
  • 2. Tải tài liệu tại sividoc.com Viết đề tài giá sinh viên – ZALO:0973.287.149-TEAMLUANVAN.COM ĐẠI HOC THÁI NGUYÊN TRƯ NG ĐẠI HOC KHOA HOC NGUYEN NGOC HÀ BAT ĐANG THỨC VÀ BÀI TOÁN CỰC TR± TRONG L P CÁC ĐA THỨC VÀ PHÂN THỨC H SO NGUYÊN LU N VĂN THẠC SỸ TOÁN HOC Chuyên ngành: PHƯƠNG PHÁP TOÁN SƠ CAP Mã so: 60 46 01 13 Ngư i hư ng dȁn khoa hoc: GS. TSKH. NGUYEN VĂN M U THÁI NGUYÊN - NĂM 2016
  • 3. i Viết đề tài giá sinh viên – ZALO:0973.287.149-TEAMLUANVAN.COM Mnc lnc M đau 1 1 M t so kien thfíc chuan bị 3 1.1 M®t so tính chat cơ bản của đa thác với h so nguyên . . . . 3 1.2 M®t so tính chat của phân thác hǎu tỉ với h so nguyên . . . 8 1.3 Định lý Viète............................................................................... 12 1.4 M®t so bat đȁng thác cơ bản ............................................................ 13 2 Các dạng toán ve đa thfíc và phân thfíc v i h so nguyên 19 2.1 Các dạng toán ve đa thác m®t bien với h so nguyên và h thác Viète.......................................................................................19 2.2 Đa thác với các h so nguyên và đong dư thác .............................30 2.3 Bat đȁng thác phân thác sinh bởi tam thác b c hai trên m®t khoảng............................................................................................35 2.4 Bat đȁng thác sinh bởi hàm phân tuyen tính trên m®t khoảng 40 2.5 Phân thác chính quy và m®t so tính chat .....................................45 3 M t so bài toán liên quan đen bat đang thfíc và cfic trị trên t p so nguyên 49 3.1 Bat đȁng thác trên t p so nguyên .................................................49 3.2 Cực trị trên t p so nguyên .......................................................... 58 Ket lu n 69 Tài li u tham khảo 70
  • 4. 1 Viết đề tài giá sinh viên – ZALO:0973.287.149-TEAMLUANVAN.COM M đau Chuyên đe đa thác là m®t chuyên đe rat quan trong ở b c trung hoc phő thông. Đa thác không chỉ là đoi tượng nghiên cáu trong tâm của Đại so mà còn là công cụ đac lực trong nhieu lĩnh vực khác của toán hoc. Trong các kì thi hoc sinh giỏi toán các cap, Olympic Toán sinh viên, các bài toán liên quan tới đa thác nói chung và đ c bi t là các bài toán ve bat đȁng thác, cực trị của đa thác, phân thác có h so nguyên thường xuyên được đe c p. Nhǎng dạng toán này thường được xem là thu®c loại khó, hơn nǎa phan kien thác ve đa thác, phân thác h so nguyên lại không nam trong chương trình chính thác của So hoc và Đại so b c trung hoc phő thông. Đe đáp áng nhu cau boi dương giáo viên và boi dương hoc sinh giỏi ve chuyên đe đa thác, tôi đã làm lu n văn: Bat đȁng thác và bài toán cực trị trong lớp các đa thác và phân thác h so nguyên. Lu n văn gom phan mở đau, ba chương, phan ket lu n và danh mục tài li u tham khảo. Chương I trình bày các kien thác cơ bản ve đa thác và phân thác h so nguyên, định lý Viète, m®t so bat đȁng thác cơ bản. Chương II trình bày m®t so dạng toán ve bat đȁng thác và cực trị trong lớp các đa thác và phân thác h so nguyên, phân thác chính quy và áp dụng. Chương III trình bày m®t so bat đȁng thác và bài toán cực trị trên t p so nguyên. Lu n văn có the được xem như m®t tài li u boi dương giáo viên và boi dương hoc sinh giỏi ve chuyên đe đa thác. Có the sả dụng lu n văn trong vi c giảng dạy hoc sinh thi hoc sinh giỏi các cap, Olympic sinh viên.
  • 5. 2 Viết đề tài giá sinh viên – ZALO:0973.287.149-TEAMLUANVAN.COM Lu n văn được hoàn thành dưới sự hướng dan trực tiep của GS.TSKH Nguyen Văn M u. Tác giả xin bày tỏ lòng biet ơn sâu sac ve sự chỉ bảo t n tình của Thay trong suot quá trình xây dựng đe cương cũng như hoàn thành lu n văn. Tác giả xin gải lời cảm ơn chân thành tới TS Nguyen Thị Thu Thủy và các quí thay cô đã đoc, kiem tra, đánh giá và đưa ra nhǎng ý kien quý báu đe lu n văn được hoàn thi n hơn. Tác giả xin chân thành cảm ơn quý Thay Cô trong Ban Giám hi u, phòng sau Đại hoc, khoa Toán Tin trường Đại hoc Khoa hoc, Đại hoc Thái Nguyên tạo đieu ki n thu n lợi trong suot quá trình hoc t p và hoàn thành lu n văn. Trong khuôn khő m®t lu n văn, tác giả chưa the trình bày được het các van đe ve đa thác và phân thác h so nguyên. Tuy bản thân đã có nhieu co gang, no lực nghiên cáu, song do đieu ki n và trình đ® còn hạn che nên nhǎng ket quả đạt được trong lu n văn còn rat khiêm ton. Tác giả kính mong nh n được ý kien đóng góp quí báu của các thay cô đe bản lu n văn được hoàn thi n hơn. Tác giả xin chân thành cảm ơn! Thái Nguyên, ngày 20 tháng 5 năm 2016. Hoc viên Nguyen Ngoc Hà
  • 6. 3 Viết đề tài giá sinh viên – ZALO:0973.287.149-TEAMLUANVAN.COM Chương 1 M t so kien thfíc chuan bị Trong chương này sě trình bày m®t so kien thác cơ bản: định nghĩa, m®t so tính chat ve sự chia het, ve nghi m nguyên, ve h so . . . của đa thác và phân thác h so nguyên. đây ta sě sả dụng m®t so ký hi u: Cho đa thác f(x) = anxn + an−1xn−1 + · · · + a1x + a0, neu f(x) có các h so là so nguyên thì ta ký hi u f(x) ∈ Z[x], neu f(x) có các h so là so hǎu tỉ thì ta ký hi u f(x) ∈ Q[x]. 1.1 M t so tính chat cơ bản của đa thfíc v i h so nguyên Định lj 1.1 (xem [4]). Cho đa thác f(x) = anxn +an−1xn−1 +· · ·+a1x+a0 ∈ Z[x], an 0, a là so nguyên. Khi đó [f(x) − f(a)].(x − a). ChGng minh. Ta có f(a) = anan + an−1an−1 + · · · + a1a + a0, f(x) − f(a) = an (xn − an ) + an−1 xn−1 − an−1 + · · · + a1(x − a).(x − a). V y ta có đieu can cháng minh.
  • 7. 4 Viết đề tài giá sinh viên – ZALO:0973.287.149-TEAMLUANVAN.COM q − q q q Bài toán 1.1 (xem [4]). Cháng minh rang neu phân so toi giản 1) là nghi m của đa thác với h so nguyên f(x) = anxn + an−1xn−1 + · · · + a1x + a0 thì p là ước của a0 và q là ước của an. Lài giai. p , ((p, q) = q p Giả sả phân thác toi giản q là nghi m của đa thác f(x). Khi đó, ta có f p = an p n + an−1 p n−1 + · · · + a1 p + a0 = 0. Tà đó, ta có và anpn = −q(an−1pn−1 + · · · + a1qn−2 p + a0qn−1 ) (1.1) a0qn = −p(anpn−1 + an−1pn−2 q + · · · + a1qn−1 ). (1.2) Tà (1.1) suy ra anpn chia het cho q mà (p, q) = 1 nên an chia het cho q. Tà (1.2)suy ra a0qn chia het cho p mà (p, q) = 1 nên a0 chia het cho p. p Bài toán 1.2 (xem [4]). Cháng minh rang neu phân thác toi giản 1) là nghi m của đa thác với h so nguyên f(x) = anxn + an−1xn−1 + · · · + a1x + a0 thì p − mq là ước của f(m) với m là so nguyên. Lài giai. Phân tích f(x) theo các lũy thàa của (x − m) ta được , ((p, q) = q f(x) = an(x − m)n + bn−1(x − m)n−1 + · · · + b1(x − m) + b0 = g(x − m). Nh n xét rang các h so b0, bn 1 là các so nguyên vì m là m®t so nguyên. p Ta có f(m) = b0. Thay x bởi ta thu được đȁng thác q f p = g p − m = g p − mq = 0. q q q
  • 8. 5 Viết đề tài giá sinh viên – ZALO:0973.287.149-TEAMLUANVAN.COM Do đó p − mq q b0 = f(m). là nghi m của g(x). theo Bài toán 2.6 thì p − mq là ước của Bài toán 1.3. Cho đa thác f(x) có h so nguyên thỏa mãn đieu ki n f (0), f (1), . . . , f (m − 1) đeu không chia het cho m (m là so nguyên dương cho trước, m > 1). Cháng minh rang f(x) = 0 không có nghi m nguyên. Lài giai. Giả sả f(x) = 0 có nghi m nguyên là x = c, khi đó f(x) = (x − c)g(x), g(x) ∈ Z[x]. Ta có f(0) = (0 − c)g(0), f(1) = (1 − c)g(1), . . . f(m − 1) = (m − 1 − c)g(m − 1). Vì 0 − c, 1 − c, . . . , m − 1 − c là m so nguyên liên tiep nên phải có m®t so chia het cho m. Vì v y trong m so f(0), f(1), . . . , f(m − 1) phải có ít nhat m®t so chia het cho m. Đieu này trái với giả thiet. V y f(x) = 0 không có nghi m nguyên. Bài toán 1.4. Cho đa thác P(x) với các h so nguyên, chia het cho 3 khi x lay các giá trị nguyên k, k + 1, k + 2. Cháng minh rang P(m) .3 với moi so nguyên m. ChGng minh. Với hai so nguyên m và n phân bi t, ta có P(m) − P(n).(m − n). Ta có các so P (m) − P (k), P (m) − P (k + 1) và P (m) − P (k + 2) theo thá tự đó lan lượt chia het cho m − k, m − (k + 1), m − (k + 2) với moi m ∈ / {k, k + 1, k + 2}.
  • 9. 6 Viết đề tài giá sinh viên – ZALO:0973.287.149-TEAMLUANVAN.COM Vì m − k, m − (k + 1), m − (k + 2) là ba so nguyên liên tiep nên trong đó có m®t so chia het cho 3. Do đó trong các so P (m) − P (k), P (m) − P (k + 1) và P(m) − P(k + 2) có m®t so chia het cho 3. M t khác, theo giả thiet, các so P (k), P (k + 1), P (k + 2) đeu chia het cho 3. V y P(m).3 với moi so nguyên m. Bài toán 1.5 (xem [4]). Cho đa thác f (x) ∈ Z[x]. Cháng minh rang neu phương trình f (x) = 1 có nhieu hơn 3 nghi m nguyên phân bi t thì phương trình f(x) = −1 không có nghi m nguyên. Lài giai. Giả sả phương trình f (x) = −1 có nghi m nguyên là a thì f (a) = −1. Goi x1, x2, x3, x4 là 4 nghi m nguyên phân bi t của phương trình f(x) = 1, thì Suy ra f(x) − 1 = (x − x1)(x − x2)(x − x3)(x − x4)g(x). f(a) − 1 = −2 = (a − x1)(a − x2)(a − x3)(a − x4)g(a), trong đó (a − x1), (a − x2), (a − x3), (a − x4) là 4 so nguyên phân bi t. Nhưng −2 không the phân tích được thành tích của 4 so nguyên khác nhau nên đieu giả sả ở trên là sai. V y phương trình f(x) = −1 không có nghi m nguyên. Bài toán 1.6. Giả sả P (x) là đa thác b c 1991 với h so nguyên. Xét đa thác Q(x) = P 2 (x) − 9. Cháng minh rang so nghi m nguyên của đa thác Q(x) nhỏ hơn 1996. Lài giai. Giả sả so nghi m của đa thác Q(x) không nhỏ hơn 1996. Q(x) = 0 ⇔ P2 (x) − 9 = 0 ⇔ [P(x) − 3][P(x) + 3] = 0.
  • 10. 7 Viết đề tài giá sinh viên – ZALO:0973.287.149-TEAMLUANVAN.COM Goi x1, x2, . . . , xk là các nghi m nguyên của P(x) = 3, (x1 < x2 < · · · < xk) và y1, y2, . . . , yl là các nghi m nguyên của P(x) = −3, (y1 < y2 < · · · < yl). Rõ ràng xi /= yj, ∀i, j. Vì deg P(x) = 1991 nên k ≤ 1991; l ≤ 1991. M t khác k + l chính là so nghi m của đa thác Q(x) nên theo giả thiet phản cháng thì k + l ≥ 1996. Tà đó ta có k ≥ 5, l ≥ 5, suy ra ton tại i0, j0 (1 ≤ i0 ≤ k; 1 ≤ j0 ≤ l) sao cho |xi0 − yj0| ≥ 7. (1.3) Giả sả P(x) = a1991x1991 + a1990x1990 + · · · + a1x + a0 với ai ∈ Z, i = 0, 1991. The thì tà ai ∈ Z, i = 0, 1991, P(xi0) = 3 và P (yi0) = −3, suy ra P(xi0 ) − P (yi0 ) = 6. Vì P(x) là đa thác với h so nguyên và xi0, yi0 là các so nguyên nên ta có P(yi ) − P(xi ). i — xi . Như v y 6.(yi0 0 — xi0), suy ra 0 .y 0 0 yi0 − xi0 ≤ 6. (1.4) Tà (1.3) và (1.4) suy ra mâu thuan. V y giả thiet phản cháng là sai, tác là đa thác Q(x) = P2 (x) − 9 không the có quá 1995 nghi m nguyên. Nh n xét 1.1. Ta có the cháng minh (1.3) như sau: Vì k ≥ 5, l ≥ 5, xi / = yj, ∀i, j do đó có ít nhat ba nghi m trong so các nghi m yj (j = 1, l) nhỏ hơn (ho c lớn hơn) các nghi m xi (i = 1, k). Giả sả x1 < x2 < · · · < xk < yp < yp+1 < yp+2
  • 11. 8 Viết đề tài giá sinh viên – ZALO:0973.287.149-TEAMLUANVAN.COM (ở đây yp, yp+1, yp+2 là 3 trong so các nghi m y1, y2, y3, . . . , yl). Chú ý là ở đây các xi (i = 1, k), yp, yp+1, yp+2 đeu là so nguyên nên |yp+2 − x1| = yp+2 − x1 ≥ k − 1 + 3 = k + 2. Do k ≥ 5 nên suy ra |yp+2 − xi| ≥ 7. 1.2 M t so tính chat của phân thfíc hfiu tỉ v i h so nguyên Định nghĩa 1.1 (xem [4]). Hàm so f : R → R có dạng P (x) f(x) = Q(x) được goi là phân thác hǎu tỉ, trong đó P(x), Q(x) là các đa thác. Neu đa thác P(x) và Q(x) là các đa thác có h so hǎu tỉ thì bang vi c quy đong mau so ta sě đưa f(x) ve dạng f(x) = P1(x) Q1(x) trong đó P1(x) và Q1(x) là các đa thác có h so nguyên. P (x) Do v y phân thác hǎu tỉ f(x) = Q(x) được goi là phân thác hǎu tỉ có h so nguyên neu như P(x), Q(x) ∈ Q[x]. Bài toán 1.7 (xem [4]). Cho phân thác hǎu tỉ 1 f(x) = ax + b ∈ Q với moi x ∈ Z. Cháng minh rang a, b ∈ Q. ChGng minh. 1 1 f(x) = ∈ Q với moi x ∈ Z nên ax + b = ∈ Q với moi x ∈ Z. ax + b f(x) V y ax + b ∈ Q[x] hay a, b ∈ Q.
  • 12. 9 Viết đề tài giá sinh viên – ZALO:0973.287.149-TEAMLUANVAN.COM Bài toán 1.8 (xem [4]). Cháng minh rang neu f(x) = C 1 ax + b ∈ Q với moi x ∈ Z thì f(x) có dạng f(x) = ChGng minh. 1 Ax + B với A, B, C thu®c Z. f(x) = ∈ Q với moi x ∈ Z nên theo Bài toán 1.7 ta có a, b ∈ Q. ax + b Đ t a = m ,b = n e f (m, n, e, f ∈ Z). Khi đó 1 f(x) = m e x + nf = = mfx + ne C Ax + B n f với A, B, C thu®c Z. Bài toán 1.9 (xem [4]). Cho phân thác hǎu tỉ f(x) = x ∈ Z. Cháng minh rang f(x) có the bieu dien dưới dạng Ax + B ax + b cx + d ∈ Q với moi ChGng minh. f(x) = Cx + D (A, B, C, D ∈ Z). (1.5) Neu ad − bc = 0 thì f(x) = const nên bieu dien (1.5) là hien nhiên. Xét trường hợp ad − bc /= 0. Neu c = 0 thì bieu dien (1.5) là hien nhiên. Neu c = 0 thì sả dụng phân tích f(x) − f(0) x 1 = αx + β ∈ Q với moi x ∈ Z. Áp dụng Bài toán 1.8 ta sě được dạng bieu dien (1.5). Nh n xét rang ket quả của Bài toán 1.9 cũng đúng trong trường hợp tőng quát. Bài toán 1.10 (xem [4]). Cho phân thác hǎu tỉ P (x) f(x) = ∈ Q ∀x ∈ Z, (P(x), Q(x)) = 1. Q(x) Cháng minh rang f(x) có the bieu dien được dưới dạng phân thác của hai đa thác với h so nguyên.
  • 13. 10 Viết đề tài giá sinh viên – ZALO:0973.287.149-TEAMLUANVAN.COM p x + qk k k=1 ChGng minh. Giả sả m P(x) = a0 + a1x + · · · + amx , n Q(x) = b0 + b1x + · · · + bnx . Tại x = j (j = 0, 1, . . . , m + n) hàm f (x) nh n các giá trị hǎu tỉ tương áng là cj. Khi đó ta có h phương trình tuyen tính với m + n + 2 ȁn: a0, a1, . . . , am, b0, b1, . . . , bn dạng a0 + a1k + · · · + amkm − b0ck − b1ckk − · · · − bnckkn = 0, trong đó k = 0, 1, . . . , m + n. Hai nghi m của h này cho ta hai c p đa thác P (x), Q(x) và P1(x), Q1(x) có tính chat P(k) − ckQ(k) = 0, P1(k) − ckQ1(k) = 0, ∀k = 0, m + n. Hai c p nghi m này cho ta đa thác g(x) = P (x)Q1(x) − P1(x)Q(x), deg g(x) ≤ m + n nh n giá trị 0 tại m + n + 1 điem nên g(x) ≡ 0. Do P (x) và Q(x) nguyên to cùng nhau nên P (x) = cP1(x); Q(x) = cQ1(x). V y h đã cho chỉ có m®t nghi m với sự sai khác m®t thàa tỉ l và như v y ton tại ma tr n cap m + n + 1 trong ma tr n h so của h phương trình đe định thác của nó khác 0 và nghi m đã nh n được là các so hǎu tỉ. Đây là đieu can cháng minh. Bài toán 1.11 (xem [4]). Cho p là m®t so nguyên dương, q ∈ [0, 1]. Giả sả x ∈ [qp+1 , 1] và f(x) = Y x − q .
  • 14. 11 Viết đề tài giá sinh viên – ZALO:0973.287.149-TEAMLUANVAN.COM p = . . − . . − − . . . . x+ q j p . . − . . − j p − p . . ta có x ≥ qi . V y nên . . k − Cháng minh rang |f(x)| ≤ Y 1 − q . ChGng minh. k=1 1 + qk Ta có 0 < qp+1 < qp < · · · < q < 1. Với qj+1 ≤ x ≤ qj thì khi i ≥ j + 1 Xét hi u x − qi x − qi .x + qi . 1 − qi x qi x + qi . 2qi (x − 1) x + qi − 1 + qi = Do đó p (x + qi) (1 + qi) ≤ 0. p k= Y j+1 Với k = 1, . . . , j, ta có x qk .x + qk . ≤ k= Y j+1 1 qk 1 + qk . Ta lại có x − qj−(k−1) j−(k−1) qj−(k−1) x = . qj−(k−1) + x qj−(k−1) − x 1 − qk 2 qj+1 − x V y nên qj−(k−1) − x − 1 + qk = (1 + qk) x + qj−(k−1) ≤ 0. Tà đó ta được hay k Y =1 k Y =1 x qk .x + qk . ≤ x qk .x + qk . ≤ k Y =1 k Y =1 1 qk 1 + qk . 1 − qk 1 + qk |f(x)| ≤ k Y =1 1 − qk .1 + qk . .
  • 15. 12 Viết đề tài giá sinh viên – ZALO:0973.287.149-TEAMLUANVAN.COM − − − a3 4 ≥ a3 . 1.3 Định lj Viète Định lj 1.2 (Định lý Viète (xem [6])). Giả sả f(x) ∈ R[x] có dạng f(x) = anxn + an−1xn−1 + · · · + a1x + a0, deg f = n. Khi đó, neu f có n nghi m x1, x2, . . . , xn (có the không phân bi t) thì x1 + x2 + · · · + xn = an−1 , an x1x2 + x1x3 + · · · + xn−1xn = an−2 , an x1x2x3 + x1x2x4 + · · · + xn−2xn−1xn = an−3 , an . . . x1x2 ChGng minh. . . . xn = ( 1)n a0 . an Do f(x) là đa thác b c n và có n nghi m x1, x2, . . . , xn nên f(x) = an(x − x1)(x − x2)........(x − xn) = anxn − an(x1 + x2 + · · · + xn)xn−1 + · · · + (−1)n anx1x2 ....... xn. Dựa vào so sánh h so của f(x) theo cách khai trien trên và f(x) = anxn + an−1xn−1 + ····+ a1x + a0, ta được đieu can cháng minh. Bài toán 1.12. Cho đa thác f(x) = x4 +a1x3 +a2x2 +a3x+a4 có 4 nghi m không âm (có the không phân bi t). Cháng minh 4 4 Lài giai.
  • 16. 13 Viết đề tài giá sinh viên – ZALO:0973.287.149-TEAMLUANVAN.COM 1 2 3 4 1 2 3 4 n n Goi 4 nghi m không âm của đa thác là x1, x2, x3, x4. Theo định lý Vi ét ta có x1x2x3x4 = a4 ≥ 0, x1x2x3 + x1x2x4 + x1x3x4 + x2x3x4 = −a3 ≥ 0. Áp dụng bat đȁng thác AM - GM cho 4 so không âm ta có −a3 = x1x2x3 + x1x2x4 + x1x3x4 + x2x3x4 ≥ 4 q 4 x3 x3 x3 x3 . Suy ra −a3 ≥ q 4 x3 x3 x3 x3 , −a3 ≥ q 4 a3 ⇔ a3 4 ≥ a3 . 4 4 4 4 Đȁng thác xảy ra khi và chỉ khi x1x2x3 = x1x2x4 = x1x3x4 = x2x3x4 ⇔ x1 = x2 = x3 = x4. Có the mở r®ng Bài toán 1.12 cho đa thác b c n như sau Bài toán 1.13. Cho đa thác b c n f(x) = a0xn + a1xn−1 + · · · + an−1x + an(a0 = / 0) có n nghi m không âm (có the không phân bi t). Cháng minh an−1 n ≥ an−1 . Cách giải bài toán này cũng tương tự như trên. 1.4 M t so bat đang thfíc cơ bản Trong mục này sě trình bày bat đȁng thác AM - GM, đây là m®t bat đȁng thác quen thu®c. Tà bat đȁng thác AM - MG ta cháng minh được m®t so bat đȁng thác dưới đây. Các bat đȁng thác này được áp dụng trong phan sau của lu n văn. 4
  • 17. 14 Viết đề tài giá sinh viên – ZALO:0973.287.149-TEAMLUANVAN.COM Σ Y ! Y Y ! n n Y 1 n Y x n Y 1 n Y x n 1 Σ 1 1 Σ x i=1 Bat đang thfíc AM - GM (xem[3]) n 1 n i=1 xi ≥ 1 n n xi i=1 (1.6) với n là so nguyên dương, xi là so thực không âm, ∀i = 1, n. Bat đang thfíc 1.4.1 (xem[3]) n i=1 (1 + xi) 1 n ≥ 1 + 1 n n xi i=1 (1.7) với n là sô nguyên dương, xi là so thực dương, ∀i = 1, n. ChGng minh. Bat đȁng thác (1.7) tương đương với n !1 n !1 i=1 1 + xi i=1 1 + xi Áp dụng bat đȁng thác AM - GM ta có n !1 n !1 n n i=1 1 + xi i=1 1 + xi n i=1 1 + xi n i=1 1 + xi = 1 Σ 1 + xi = 1. Đây là đieu can cháng minh. Bat đang thfíc 1.4.2 (xem[3]) n i=1 1 + xi 1 + xi Σ q ‚ . Σ n !2 Σ n !2 x2 + y2 ≥ , xi + yi (1.8) với n là sô nguyên dương, xi, yi là so thực dương, ∀i = 1, n. i=1 i i i + ≤ ) i + . ) i + 1 ≥ ! i=1
  • 18. 15 Viết đề tài giá sinh viên – ZALO:0973.287.149-TEAMLUANVAN.COM 2 k k ‚ . , Σ 1 1 2 2 1 1 2 2 i=1 ! ! Σ 2 2 2 2 2 i=1 ChGng minh. Ta cháng minh bat đȁng thác (1.8) đúng với n = 2, tác là q x2 + y2 + q x2 + y2 ≥ q (x1 + x2)2 + (y1 + y2)2. (1.9) Th t v y, bình phương hai ve của (1.9), ta có q (x2 + y2 )(x2 + y2 ) ≥ x1x2 + y1y2 ⇔(x1 + y1)(x2 + y2) ≥ (x1x2 + y1y2) ⇔(x1y2 − x2y1) ≥ 0. V y (1.9) đúng. Giả sả bat đȁng thác (1.8) đúng với n = k, tác là Σ q ‚ . Σ k !2 Σ k !2 x2 + y2 ≥ , xi + yi . Ta can cháng minh bat đȁng thác đúng với n = k + 1. Xét Σ k+1 q Σ q q Suy ra i=1 i i i i i=1 k+1 k+1 Σ k+1 q ‚ . Σ k !2 Σ k !2 q x2 + y2 ≥ , xi + yi + x2 + y2 i=1 i i i=1 k+1 2 ≥ xi + i=1 i=1 k+1 i=1 2 yi . k+1 k+1 V y bat đȁng thác được cháng minh. Bat đang thfíc 1.4.3 (xem[3]) Σ n Σ n xiyi xi + yi ≤ xi i=1 Σ n yi i=1 Σ n (1.10) i=1 i=1 n . i=1 i i Σ i=1 x2 + y2 = x2 + y2 + x2 + y2 xi + yi
  • 19. 16 Viết đề tài giá sinh viên – ZALO:0973.287.149-TEAMLUANVAN.COM n y Σ n 2 Σ Σ !2 √ y Σ ! Σ n n 2 1 2 ≥ Σ yi Σ i ⇔ ≥ ! Σ n i với n là sô nguyên dương, xi, yi là so thực dương,∀i = 1, n. ChGng minh. Bat đȁng thác (1.10) tương đương với n n xi Σ n Σ xiyi − y ≤ i=1 i=1 − Σ y i=1 xi + yi Σ xi + Σ yi i=1 i=1 2 yi i=1 . i=1 Ta có i=1 xi + yi n i=1 xi + n yi i=1 Σ n !2 Σ n yi = i=1 i=1 √ xi + yi xi + yi n ≤ i=1 2 i xi + yi n i=1 xi + Σ i=1 yi ! . Ta có đieu can cháng minh. Bat đang thfíc 1.4.4 (xem[3]) 1 Σ xm ≥ 1 n m xi (1.11) n i i=1 n i=1 với m, n là so nguyên dương, xi là so thực không âm, ∀i = 1, n. ChGng minh. Ta cháng minh bat đȁng thác (1.11) bang phương pháp qui nạp theo n. Với n = 2 ta có (1.11) là xm + xm x1 + x2 m 2 n n i y . i
  • 20. 17 Viết đề tài giá sinh viên – ZALO:0973.287.149-TEAMLUANVAN.COM 2 1 2 2 2 Σ Σ Σ 1 i Σ Σ x Σ Σ 1 2 ≥ 1 2 1 2 1 2 ≥ 1 2 1 2 1 i ! 2k i=1 Ta có xm + xm x1 + x2 xm−1 + xm−1 xm + xm − x1xm−1 − x2xm − 21 4 ⇔ xm−1 − xm−1 (x1 − x2) ≥ 0. Đieu này đúng với moi so thực dương x1, x2 và m nguyên dương. Tà đó, ta suy ra xm + xm x1 + x2 xm−1 + xm−1 x1 + x2 2 xm−2 + xm−2 ≥ · · · ≥ x1 + x2 m Giả sả (1.11) đúng với n = k, ta sě cháng minh (1.11) đúng với n = 2k. Th t v y 1 Σ xm = 1 1 Σ k 2k xm + xm 2k i i=1 2 k i i=1 k i i=k+1 1 " 1 Σ k !m 1 2k !m# 1 Σ 2k !m Bat đȁng thác sě được cháng minh khi ta cháng minh được (1.11) đúng với n = k + 1 thì sě đúng với n = k. Th t v y, ta can cháng minh hay k k i=1 xm ≥ 1 k xi k i=1 k P = m i=1 1 k xi k i=1 ≥ (k + 1) 1 k k i=1 m xi . xi k k 2 2 2 2 2 2 2 2 ≥ 0 2k ≥ ≥ i=1 xi + i=k+1 ≥ xi . ⇔ . ! !m ! + !m
  • 21. 18 Viết đề tài giá sinh viên – ZALO:0973.287.149-TEAMLUANVAN.COM xi + k xi Áp dụng giả thiet quy nạp, ta có " 1 Σ k 1 Σ !#m 1 Σ k !m V y bat đȁng thác (1.11) đã được cháng minh. i=1 k xi i=1 i=1 k + 1 k P ≥ (k + 1) ≥ (k + 1) .
  • 22. 19 Viết đề tài giá sinh viên – ZALO:0973.287.149-TEAMLUANVAN.COM √ Chương 2 Các dạng toán ve đa thfíc và phân thfíc v i h so nguyên 2.1 Các dạng toán ve đa thfíc m t bien v i h so nguyên và h thfíc Viète Trong mục này, sě trình bày các bài toán ước lượng m®t so đại lượng liên quan đen đa thác h so nguyên như: ước lượng h so, ước lượng nghi m, ước lượng mien giá trị của đa thác,. . . Bài toán 2.1. Cho tam thác b c hai f(x) = ax2 + 1998x + c với a, c ∈ Z, |a| < 2000, |c| < 2000 và f(x) có hai nghi m phân bi t x1, x2. Cháng minh 1 Lài giai. |x1 − x2| ≥ . 998 f(x) có bi t thác ∆ ′ = 9992 − ac, do f(x) có hai nghi m phân bi t x1, x2 nên ∆ ′ > 0, ∆ ′ ∈ Z và |x1 − x2| = 2 ∆′ . |a| Neu ∆ ′ ≥ 2 thì |x1 − x2| = 2 √ ∆′ > |a| 2 √ ∆′ 2000 ≥ 2 √ 2 > 2000 1 998
  • 23. 20 Viết đề tài giá sinh viên – ZALO:0973.287.149-TEAMLUANVAN.COM Σ 1 n xi xi a0 a0 i=1 (do |a| < 2000). Neu ∆ ′ = 1 thì ac = 9992 − 1 = 998.1000 = 24 .53 .449. Do đó a là m®t ước so của 24 .53 .449, lại có 22 .449 = 1996 là m®t ước so của 24 .53 .449 và |a| < 2000 nên a ≤ 1996. Tà đây ta suy ra |x1 − x2| = 2 √ ∆′ = |a| 2 |a| ≥ 2 = 1996 1 . 998 Bài toán 2.2. Cho đa thác P (x) = xn + an−1xn−1 + · · · + a1x + a0 và |ai| = 1(i = 0, 1, . . . , n − 1). Cháng minh rang neu P(x) có n nghi m thực thì n ≤ 3. Lài giai. Giả sả P(x) có n nghi m là x1, x2, . . . , xn. Áp dụng định lý Viète, ta có n xi = −an−1, i=1n Suy ra Σ Σ n !2 i,j= Σ 1,i<j Σ xixj = an−2. Do x1x2 . . . xn = (−1)n a0 và |a0| = 1 nên xi = / 0 ∀i. De thay xi là nghi m của đa thác Q(x) = a0xn + a1xn−1 + · · · + an−1x + 1. Áp dụng Định lý Viète, ta có n Σ 1 = −a1 , Σ 1 = a2 . i,j=1,i<j xixj i,j=1,i<j i=1 i=1 n n xi 2 = — 2 xixj = 3 (do|an−1| = 1, |an−2| = 1). (2.1)
  • 24. 21 Viết đề tài giá sinh viên – ZALO:0973.287.149-TEAMLUANVAN.COM ! 2 Σ Σ Σ Suy ra 1 x 2 = Σ n 1 2 x − 2 1 x x = 3, ( do |a1| = 1, |a2| = 1). (2.2) i=1 i i=1 i i,j=1,i<j i j Tà (2.1) và (2.2), ket hợp với bat đȁng thác Cauchy, ta được n n 9 = Σ x 2 Σ 1 ≥ n . Suy ra n ≤ 3. i=1 i i=1 xi 2 Bài toán 2.3 (xem[5]). Cho đa thác P (x) có b c m > 0 và có các h so nguyên. Goi n là so các nghi m nguyên phân bi t của hai phương trình P(x) = 1 và P(x) = −1. Cháng minh rang n ≤ m + 2. Lài giai. Xét hai đa thác A(x) và B(x) với các h so nguyên A(x) = anxn + an−1xn−1 + · · · + a1x + a0, B(x) = anxn + an−1xn−1 + · · · + a1x + a0 + 2. Goi r, s là các nghi m nguyên tương áng của hai đa thác trên, tác là A(r) = 0, B(s) = 0. Ta có n A(r) − B(s) = ai(ri − si ) − 2 = 0. i=1 Tà đây suy ra neu r − s /= 0 thì 2 chia het cho r − s, tác là chỉ xảy ra m®t trong ba trường hợp r và s hơn kém nhau 0, 1 ho c 2 đơn vị. Ta sě áp dụng nh n xét trên đe giải bài toán. Giả sả r là nghi m nguyên nhỏ nhat trong tat cả các nghi m nguyên của hai phương trình P (x) = 1 và P (x) = −1. Do đa thác P (x) có b c m nên moi phương trình trên không có quá m nghi m. Theo nh n xét trên neu r là nghi m của phương trình này, s là nghi m của phương trình kia và r là nghi m nguyên nhỏ nhat trong tat cả các nghi m nguyên của hai phương trình thì s = r, s = r + 1 ho c s = r + 2. n n
  • 25. 22 Viết đề tài giá sinh viên – ZALO:0973.287.149-TEAMLUANVAN.COM 2 Do v y neu k là so nghi m của phương trình thá nhat thì so nghi m của cả hai phương trình nhieu nhat là k + 2 mà k ≤ m nên ta suy ra n ≤ m + 2. Bài toán 2.4 (xem[5]). Cho f (x) là đa thác b c n có các h so bang ±1. Biet rang đa thác có x = 1 là nghi m b®i cap m với m ≥ 2k , k ≥ 2 và k nguyên. Cháng minh n ≥ 2k+1 − 1. Lài giai. Goi f(x) là đa thác với các h so theo modulo 2 của f(x). Vì f(x) có các h so là 1 và -1 nên f(x) = xn + xn−1 + · · · + x + 1. Ta có f (x) = (x − 1)2k g(x), trong đó n ≥ 2k và g(x) là đa thác h so nguyên. De dàng cháng minh được Ci k ≡ 0(mod2), 1 ≤ i ≤ 2k − 1 nên f(x) = xn + xn−1 + · · · + x + 1 = (x2k + 1)g(x). (2.3) Giả sả g(x) có b c không quá 2k − 2. Ta có h so của x2k−1 ở ve phải của (2.3) là 0. Đieu này mâu thuan vì h so của x2k−1 ở ve trái của (2.3) là 1. Do đó b c của g(x) không nhỏ hơn 2k − 1. V y n ≥ 2k + 2k − 1 = 2k+1 − 1. Bài toán 2.5. Cho đa thác với h so nguyên P(x) = akxk + · · · + a1x + a0. Ta ký hi u so các h so lẻ của đa thác là h(P ). Với i = 0, 1, 2, . . . ta đ t Qi(x) = (1 + x)i . Cháng minh rang neu i1, i2, . . . , in là các so nguyên thỏa mãn đieu ki n 0 ≤ i1 < i2 < · · · < in thì ta có h(Qi1 + Qi2 + · · · + Qin ) ≥ h(Qi1). Lài giai. Trước het ta có nh n xét: Neu i là m®t b®i so của 2 thì tat cả các h so của Qi đeu chȁn, ngoại trà h so đau tiên và h so cuoi cùng.
  • 26. 23 Viết đề tài giá sinh viên – ZALO:0973.287.149-TEAMLUANVAN.COM i 1 1 mB i Th t v y giả sả i = 2k , k nguyên dương. Ta có h so của Qi(x) = (1+ x)i là Cr = i! r!(i − r)! Giả sả 0 < r < i, r = 2l r , r là so lẻ. Lúc đó Cr = i Cr−1 nhưng Cr−1 là 1 1 i r i−1 i−1 m®t so nguyên và l < k nên suy ra Cr là so chȁn. Đ t Q = Qi1 + Qi2 + · · · + Qin. Ta sě cháng minh bat đȁng thác ở đe bài bang quy nạp theo in. Với in = 0 ho c in = 1 de thay bat đȁng thác đúng. Giả sả bat đȁng thác đúng với các giá trị bé hơn in. Ta goi m là m®t lũy thàa của 2 sao cho m ≤ in < 2m. Xét các trường hợp i1 ≥ m và i1 < m Trường hợp 1 i1 ≥ m. Lúc đó ta có Qi = (1 + x)m A, Q = (1 + x)m B với A, B là các đa thác có b c nhỏ hơn m. Theo giả thiet qui nạp ta có h(A) ≤ h(B) nên h(Qi ) = h((1 + x)m A) = h(A + xm A) = 2h(A) ≤ 2h(B) = h(B + xmB) = h((1 + x) ) = h(Q). V y bat đȁng thác đúng ở trường hợp 1. Trường hợp 2 i1 < m. Lúc đó ta lay r sao cho ir < m, ir+1 > m Đ t A = Qi1 + Qi2 + · · · + Qir , B(1 + x)m = Qir+1 + Qir+2 + · · · + Qin. Suy ra A và B có b c nhỏ hơn m. Lúc đó h(Q) = h(A + (1 + x)m B) = h(A + B + xm B) = h(A + B) + h(B).
  • 27. 24 Viết đề tài giá sinh viên – ZALO:0973.287.149-TEAMLUANVAN.COM Vì h so của A là lẻ khi và chỉ khi m®t trong hai h so tương áng của A − B và của B lẻ nên h(A − B) ≥ h(A − B + B) = h(A). Vì các h so tương áng của A−B và của A+B ho c bang nhau ho c cùng tính chȁn lẻ nên h(A − B) = h(A + B). Do đó h(A + B) + h(B) ≥ h(A). Ta cũng có h(A) ≥ h(Qij ) theo giả thiet quy nạp. V y bat đȁng thác cũng đúng trong trường hợp 2. Bài toán 2.6 (xem[5]). Cho f (x) là đa thác b c n với h so nguyên và g(x) = f2 (x) − p2 , p là so nguyên to. Khi đó so nghi m nguyên phân bi t của g(x) không vượt quá max{n, min{8, 2n}}. Lài giai. Goi A = {x ∈ Z|f(x) = p} , B = {x ∈ Z|f(x) = −p} , C = {x ∈ Z|g(x) = 0} . Ta có A ∪ B = C, A ∩ B = φ nên |C| = |A| + |B|. Giả sả |A| ≥ 5. Ta cháng minh B = φ. Th t v y, neu B /= φ. Ta lay 5 phan tả xi ∈ A, (i = 1, 5) và b ∈ B ta có f(x) − p = (x − x1) . . . (x − x5)Q(x), ở đó Q(x) là đa thác h so nguyên. Do đó f(b) − p = (b − x1) . . . (b − x5)Q(b), mà f(b) + p = 0, suy ra −2p = (b − x1) . . . (b − x5)Q(b),
  • 28. 25 Viết đề tài giá sinh viên – ZALO:0973.287.149-TEAMLUANVAN.COM tác 2p được phân tích thành tích của ít nhat 5 so nguyên khác nhau. Mâu thuan vì 2 = 2.1 = (−1)(−2), p = 1.p = (−1).(−p). Do đó B = φ. V y |C| = |A| ≤ n. Tương tự neu |B| ≥ 5 thì |C| ≤ n. (2.4) Xét trường hợp |B| ≤ 4 và |A| ≤ 4 ta có |A| ≤ min{n, 4}, |B| ≤ min{n, 4}. V y |C| ≤ 2 min{n, 4} = min{2n, 8}. (2.5) Tà (2.4) và (2.5) ta có đieu can cháng minh. Bài toán 2.7. Cho các so nguyên a, b, c, biet a > 0 và đa thác ax2 + bx + c có hai nghi m khác nhau trong khoảng (0; 1). Cháng minh a ≥ 5. Tìm ít nhat m®t c p so b, c đe a = 5. Lài giai. Vì đa thác P (x) = ax2 + bx + c (a > 0) có hai nghi m khác nhau x1, x2 trong khoảng (0; 1) nên theo Định lý Viète ta có 0 < Do đó a > c > 0, b < 0. c < 1, a b < 0. a M t khác P (1) = a + b + c > 0 (do P (x) = ax2 + bx + c có hai nghi m khác nhau trong khoảng (0; 1) mà P (0) = c > 0). Suy ra a + c > −b > 0, hay a2 + 2ac + c2 > b2 . Do đó (a − c)2 > b2 − 4ac > 0.
  • 29. 26 Viết đề tài giá sinh viên – ZALO:0973.287.149-TEAMLUANVAN.COM Q Q Q P(k) j=0,j= / k p−2 P(k) j=0,j= / k . p−k p−1 M t khác đa thác P (x) = ax2 + bx + c (a > 0) có hai nghi m khác nhau x1, x2 trong khoảng (0; 1) nên ∆ = b2 − 4ac là so nguyên dương, suy ra a − c ≥ 2. Giả sả a ≤ 4 thì xảy ra ba trường hợp của c p so nguyên a, c: a = 4 và c = 2, a = 3 và c = 1, a = 4 và c = 1. Xảy ra ba trường hợp tương áng 4 > b2 − 32 > 0 ⇔ 36 > b2 > 32 (loại); 4 > b2 − 12 > 0 ⇔ 16 > b2 > 12 (loại); 9 > b2 − 16 > 0 ⇔ 25 > b2 > 16 (loại). V y a ≥ 5. Với a = 5, phương trình 5x2 − 5x + 1 = 0 có hai nghi m thu®c khoảng (0; 1). Do đó b = −5, c = 1 là m®t c p so b, c thỏa mãn. Bài toán 2.8 (xem[5]). Cho p là m®t so nguyên to và P(x) ∈ Z[x] thỏa mãn các đieu ki n i) P(0) = 0; ii) P(n) ho c chia het cho p ho c chia cho p có so dư bang 1 với moi n ∈ N∗ . Cháng minh s ≥ p − 1. Lài giai. Giả sả ngược lại s < p − 2. Khi đó theo công thác n®i suy Lagrange thì p−2 p−2 (x − j) p−2 p−2 (x − j) k=0 j=0 Q ,j/= k (k − j) k=0 k!(−1) (p − 2 − k)! Cho x = p − 1 ta thu được p−2 p−2 (p − 1 − j) p−2 . P (k)(−1)p−k Ck = P (k) P(p − 1) = Σ Σ Σ Σ P(x) = = j=0,j k k=0 k!(−1)p−k(p − 2 − k)! k=0
  • 30. 27 Viết đề tài giá sinh viên – ZALO:0973.287.149-TEAMLUANVAN.COM Σ Σ Theo giả thiet p nguyên to nên Ck p−1 ≡ (−1)k (mod p). Do v y p−2 P(p − 1) ≡ (−1)p P (k) (mod p). k=0 Neu p = 2 thì 1 = P(1) ≡ (−1)2 P(0) (mod p), vô lý. Neu p ≥ 3 thì p lẻ và vì v y p−2 P(p − 1) ≡ − P (k) (mod p). k=0 Tà đây suy ra s = P(0) + P(1) + · · · + P(p − 2) + P(p − 1) ≡ 0 (mod p). M t khác do giả thiet ii) thì s ≡ k (mod p) với 0 ≤ n ≤ p−1 và P (n) ≡ 1 (mod p) nên 1 ≤ s ≤ p − 1, tác là s /= 0 (mod p), mâu thuan với ket lu n trên. V y đieu giả sả s < p − 2 là sai. Ta được đieu can cháng minh. Bài toán 2.9. Cho P (x) là m®t đa thác không bang hang so, với h so nguyên. Goi n(P ) là so nhǎng so nguyên k khác nhau sao cho [P (k)]2 = 1, hãy cháng minh rang n(P ) − deg P ≤ 2 với deg P là b c của đa thác P (x). Lài giai. So k nói trong đau bài là so nghi m của hai phương trình P (x) = 1, (2.6) P(x) = −1. (2.7) Neu phương trình (2.7) không có nghi m nguyên thì tat cả các so k đeu là nghi m của (2.6), do đó n(P) không the vượt quá deg P, hay n(P) − deg P ≤ 0. Neu phương trình (2.6) không có nghi m nguyên cũng được xét tương tự. Trường hợp cả hai phương trình (2.6) và (2.7) đeu có nghi m nguyên.
  • 31. 28 Viết đề tài giá sinh viên – ZALO:0973.287.149-TEAMLUANVAN.COM Goi x1, x2, . . . xr là tat cả các nghi m nguyên khác nhau của (2.6) và y1, y2, . . . ys là tat cả các nghi m nguyên khác nhau của (2.7). The thì r + s = n(P) và P(x) + 1 = (x − y1) . . . (x − ys)g(x) (2.8) với g(x) là đa thác h so nguyên. Trong (2.8) cho x = x1 ta được 2 = P (x1) + 1 = (x1 − y1) . . . (x1 − ys)g(x1). Các hi u x1 − y1, . . . , x1 − ys là s so nguyên khác nhau và so 2 chỉ có các ước là −1, 1, −2, 2. Tà đó suy ra s ≤ 3. Tương tự ta thay rang r ≤ 3, do đó n(P) = r + n ≤ 6. Ta hãy phân bi t các trường hợp a) deg P ≥ 4. Khi đó n(P) − deg P ≤ 6 − deg P ≤ 2. b) deg P ≤ 2. Vì n(P) là so các nghi m nguyên khác nhau của phương trình [P(x)]2 = 1 nên Do đó c) deg P = 3. n(P ) ≤ deg P 2 = 2 deg P n(P) − deg P ≤ deg P ≤ 2. Neu n(P) ≤ 5 thì n(P) − deg P ≤ 5 − 3 = 2. Đe ket thúc cháng minh, ta hãy cháng tỏ rang không the xảy ra trường hợp deg P = 3 với n(P ) = 6. Th t v y, neu trường hợp này xảy ra thì ta phải có r = s = 3 và (2.8) trở thành P(x) + 1 = A(x − y1)(x − y2)(x − y3) (2.9) với A là so nguyên. Có the coi rang y1 < y2 < y3. Cho x = x1 ta được 2 = P (x1) + 1 = A(x1 − y1)(x1 − y2)(x1 − y3).
  • 32. 29 Viết đề tài giá sinh viên – ZALO:0973.287.149-TEAMLUANVAN.COM Σ Vì x1 − y1, x1 − y2, x1 − y3 là ba so nguyên khác nhau nên A /= ±2 tác là A = ±1. Neu A = 1, thì x1 − y1 > x1 − y2 > x1 − y3 và tích của ba so này bang 2 nên ba so ay phải là 1, −1, −2, đ c bi t x1 − y1 = 1. Trong (2.9) lại cho x = x2 ta được 2 = (x2 − y1)(x2 − y2)(x2 − y3). Tà đây suy ra x2 − y1 = 1. Đieu này vô lý vì x1 /= x2. Trường hợp A = −1 cũng khảo sát tương tự. V y ta được đieu can cháng minh. Nh n xét 2.1. Bài toán (2.9) có the mở r®ng như sau: Cho P (x) là m®t đa thác không bang hang so, với h so nguyên. Goi n(P ) là so nhǎng so nguyên k khác nhau sao cho [P (k)]2 = r2 , hãy cháng minh rang n(P ) − deg P ≤ 2r với deg P là b c của đa thác P (x). Đ c bi t, khi r = 0 ta có ket quả quen thu®c: So nghi m của đa thác không vượt quá b c của đa thác đó. Bài toán 2.10 (xem [4]). Cho f (x) ∈ Z[x], deg f = m ≥ 1 và f (x) có ít nhat m®t nghi m hǎu tỉ. Giả sả phương trình |f(x)| = 1 có các nghi m tự nhiên a1, a2, . . . , an, (a1 < a2 < · · · < an). Cháng minh rang ln(ai i<j — aj ) < m(m − 1) . 2 Lài giai. p Goi (p, q ∈ Z, (p, q) = 1) là nghi m hǎu tỉ của P(x). q Theo giả thiet, ta có |p−qai| = 1 với moi i ∈ {1, 2, . . . , m}. Suy ra p−qai ∈ {1, −1} và m ≤ 2, do v y m = 2. Do |f(a1)| = |f(a2)| = 1 nên suy ra |f(a1)| = |f(a2)| ≤ 2, |a2 − a1| ≤ |f(a2) − f(a1)| ≤ 2.
  • 33. 30 Viết đề tài giá sinh viên – ZALO:0973.287.149-TEAMLUANVAN.COM Σ ≤ Σ − V y ta có ln(ai i>j — aj) = ln(a2 — a1 ) ln 2 < 1 = 2(2 − 1) 2 = m(m − 1) . 2 2.2 Đa thfíc v i các h so nguyên và đong dư thfíc Định nghĩa 2.1 (xem [1]). Cho đa thác f (x) với các h so nguyên và so nguyên dương m ≥ 2. Ta nói rang phương trình đong dư f (x) ≡ 0 (mod m) có nghi m x0 ∈ Z neu f(x0) ≡ 0 (mod m). Neu x0 ∈ Z là m®t nghi m của phương trình f(x) ≡ 0 (mod m) và t là m®t so nguyên bat kỳ thì f(x0 + tm) ≡ f(x0) ≡ 0 (mod m). Định lj 2.1 (xem [1]). Cho các so nguyên a và m, m ≥ 2, (a, m) = 1. Khi đó phương trình ax ≡ b (mod m), b ∈ Z có nghi m duy nhat x0 ∈ Z mà 0 ≤ x0 ≤ m − 1. Moi nghi m khác của phương trình này đeu có dạng xt = x0 + mt, t ∈ Z. ChGng minh. Neu k, l ∈ Z, k = / l, 0 ≤ k ≤ m − 1, 0 ≤ l ≤ m − 1 thì ak al (mod m). Đieu này có nghĩa là bieu thác as − b, s = 0, 1, . . . , m − 1 cho m so dư khác nhau khi chia cho m. V y ton tại duy nhat x0 ∈ Z, 0 ≤ x0 ≤ m − 1 sao cho ax0 ≡ b (mod m). Neu xt ∈ Z là m®t nghi m của phương trình đong dư ax ≡ b (mod m) thì axt ≡ b (mod m). Suy ra a(xt − x0) chia het cho m, nhưng (a, m) = 1 v y xt − x0 chia het cho m. Suy ra xt = x0 + mt, t ∈ Z. Định lj 2.2 (Công thác Taylor). Cho đa thác f (x) b c n, n ≥ 1 với các h so thực và x0 ∈ R. Khi đó n (k) f(x) = f(x0 ) + f (x0) (x x k! 0 k=1 )k .
  • 34. 31 Viết đề tài giá sinh viên – ZALO:0973.287.149-TEAMLUANVAN.COM Σ − i i 1 2 r i 1 2 r 1 2 r i ChGng minh. Ton tại các hang so b0, b1, . . . ., bn sao cho f(x) = bn(x − x0)n + bn−1(x − x0)n−1 + · · · + b0. Với 1 ≤ k ≤ n, ta có fk (x) = k!bk+[(k+1) . . . 2(x−x0)bk+1]+· · ·+[n(n−1) · · · (n−k+1)(x−x0)n−k bn]. Suy ra f (k) (x0) = k!bk, do đó bk = f (k) (x0) . k! M t khác f(x0) = b0. V y f(x) = f(x0 n f(k) (x ) ) + 0 (x x0 k! k=1 )k . Định lj 2.3 (xem [1]). Cho đa thác f(x) với các h so nguyên và m®t so nguyên to p. Neu phương trình đong dư f(x) ≡ 0 (mod p) có đúng r nghi m nguyên phân bi t x(1) , x(1) , . . . , x(1) thu®c đoạn [1; p] sao cho f′(x(1) ) /= 0 (mod p), (1 ≤ i ≤ r) thì phương trình đong dư f(x) ≡ 0 (mod pk ) có đúng r nghi m nguyên phân bi t thu®c đoạn [1; pk ] với moi k ≥ 1 : x(k) , x(k) , . . . , x(k) và đoi với các nghi m này ta có f′(x(k) ) /= 0 (mod p) (1 ≤ i ≤ r). ChGng minh. Ta cháng minh khȁng định bang phương pháp quy nạp toán hoc theo k. Với k = 1 thì khȁng định đúng. Giả sả khȁng định đúng với k ≥ 1. Đieu đó có nghĩa là trong đoạn [1; pk ] thì phương trình đong dư f(x) ≡ 0 (mod pk ) có đúng r nghi m nguyên phân bi t x(k) , x(k) , . . . , x(k) , đong thời f′(x(k) ) 0 (mod p) với 1 ≤ i ≤ r. Giả sả x0 ∈ Z, x0 ∈ [1; pk+1 ] là m®t nghi m của phương trình đong dư f (x) ≡ 0 (mod pk+1 ). Khi đó f (x0) ≡ 0 (mod pk+1 ). Suy ra f (x0) ≡ 0 (mod pk ). Ton tại duy nhat i ∈ [1; r], t ∈ Z, t ∈ [0; p − 1] sao cho x0 = x(k) + pk t.
  • 35. 32 Viết đề tài giá sinh viên – ZALO:0973.287.149-TEAMLUANVAN.COM i i i i pk i ≡ x i i i i i i i i i j! Giả sả x = x(k) + pk t (1 ≤ i ≤ r, t ∈ Z, t ∈ [0, p − 1]) thì x ∈ Z. Theo công thác Taylor ta có (k) (k) f ′′(x(k) ) f (n) (x (k) ) f(x) = f(x ) + f′(x )pk t + i (pk t)2 + · · · + i (pk t)n , 2! n! trong đó n là b c của f(x). f (j) (x (k) ) Ta có i ∈ Z và jk ≥ k + 1, i ≥ 2. Phương trình f(x) ≡ 0 (mod pk+1 ) tương đương với f(xk ) + f′(xk )pk t ≡ 0 (mod pk+1 ) hay f(x(k) ) (k) (chú ý rang f(x(k) ) pk i + f′(x )t ≡ 0 (mod p) ∈ Z). Đ t x(k+1) = x(k) + pk ti thì x(k+1) ∈ [1; pk+1 ], x(k+1) ∈ Z và f(x(k+1) ) ≡ 0 i i i i i (mod pk+1 ). M t khác, x(k+1) (k) (mod p). Suy ra f′(x(k+1) ) ≡ f′(x(k) ) (mod p). Suy ra f′(x(k+1) ) /= 0 (mod p) vì f′(x(k) /= 0 (mod p). V y phương trình đong dư f(x) ≡ 0 (mod pk+1 ) có đúng r nghi m nguyên phân bi t trong đoạn [1; pk+1 ] : x(k+1) , x(k+1) , . . . , x(k+1) , đong thời 1 2 r f ′ (x(k+1) ) 0 (mod p), (1 ≤ i ≤ r). Như v y khȁng định cũng đúng với k + 1. Theo nguyên lý quy nạp toán hoc thì khȁng định đúng với moi k ≥ 1. Định lý 2.3 đã được cháng minh. Áp dụng định lý này có the giải được bài toán sau. Bài toán 2.11. Cho đa thác P (x) = x3 + 153x2 − 111x + 38. 1. Hỏi trong đoạn [1; 32000 ] có ít nhat bao nhiêu so nguyên dương a sao cho P(a) chia het cho 32000 ? 2. Hỏi trong đoạn [1; 32000 ] có nhieu nhat là bao nhiêu so nguyên dương a sao cho P (a) chia het cho 32000 ?
  • 36. 33 Viết đề tài giá sinh viên – ZALO:0973.287.149-TEAMLUANVAN.COM ChGng minh. Giả sả x ∈ Z, 1 ≤ x ≤ 32000 và P(x).32000 . Do 153 ≡ 0 (mod 3), 111 ≡ 0 (mod 3), 38 ≡ 2 (mod 3), suy ra x = 3y + 1, (y ∈ Z, 1 ≤ y ≤ 31999 − 1). Ta có P(x) = P(3y + 1) = 27(y3 + 52y2 + 22y + 3). Phương trình P (x).32000 tương đương với y3 + 52y2 + 22y + 3.31997 , suy ra y = 3t + 1 ho c y = 3t, (t ∈ Z, 1 ≤ t ≤ 31998 − 1). Neu y = 3t + 1 thì y3 + 52y2 + 22y + 3 = 27t3 + 495t2 + 387t + 78 không chia het cho 9. V y y = 3t, suy ra y3 + 52y2 + 22t + 3 = 3(9t2 + 156t2 + 22t + 1). Phương trình P (x).32000 tương đương với 9t3 + 156t2 + 22t + 1.31996. Xét đa thác f(t) = 9t3 + 156t2 + 22t + 1. Với t ∈ Z thì f(t) ≡ 0 (mod 3) hay 22t + 1 ≡ 0 (mod 3). Trong đoạn [1; 3] thì phương trình đong dư 22t + 1 ≡ 0 (mod 3) có m®t nghi m duy nhat t = 2. M t khác f′(2) ≡ 22 ≡ 1 (mod 3) suy ra f′(2) /= 0 (mod 3). Theo Định lý 2.3, trong đoạn [1; 31996 ] phương trình đong dư f (t) ≡ 0 (mod 31996 ) có m®t nghi m nguyên duy nhat t0. Với t ∈ Z, t ∈ [1; 32000 ] : f (t) ≡ 0 (mod 31996 ) khi và chỉ khi ton tại h ∈ Z, 0 ≤ h ≤ 8 sao cho t = t0 + 31996 h. V y phương trình đong dư f (t) ≡ 0 (mod 31996 ) có đúng chín nghi m nguyên phân bi t trong đoạn [1; 31998 − 1]. Tà đó suy ra rang trong đoạn [1; 32000 ] có đúng chín so nguyên dương a phân bi t sao cho P(a) chia het cho 32000 . Định lj 2.4 (xem [1]). Cho đa thác f (x) khác hang so và có các h so nguyên. Khi đó ton tại vô so so nguyên to p sao cho phương trình đong dư f(x) ≡ 0 (mod p) có nghi m.
  • 37. 34 Viết đề tài giá sinh viên – ZALO:0973.287.149-TEAMLUANVAN.COM ChGng minh. Giả sả f(x) = a0xn +a1xn−1 +· · ·+an, ai ∈ Z, 0 ≤ i ≤ n, n ≥ 1, a0 0. Neu an = 0 thì f (x) = xg(x), trong đó g(x) là đa thác với các h so nguyên, khi đó phương trình đong dư xg(x) ≡ 0 (mod p) có nghi m với moi so nguyên to p. Giả sả an 0 và phương trình đong dư xg(x) ≡ 0 (mod p) có nghi m với moi so nguyên to p1, p2, . . . , pk. Với t ∈ Z, đ t xt = p1p2 . . . pkant. Khi đó f(xt) = a0(p1p2 . . . pkant)n + · · · + an−1p1p2 . . . pkant + an = an(p1p2 . . . pkB + 1), B ∈ Z. Chon t ∈ Z phù hợp, ta suy ra ton tại vô so so nguyên to p sao cho phương trình đong dư f(x) ≡ 0 (mod p) có nghi m. Đây là đieu can cháng minh. Áp dụng định lý 2.4 ta có the giải được bài toán sau. Bài toán 2.12. Cho đa thác f (x) khác hang so và có các h so nguyên. Giả sả n, k là các so nguyên dương. Cháng minh rang ton tại so nguyên x sao cho các so f (x), f (x + 1), . . . , f (x + n − 1) đeu có ít nhat k ước nguyên to phân bi t. Lài giai. Theo định lý 2.4, ton tại các so nguyên to p1, p2, . . . , pk, pk+1, . . . , pnk khác nhau tàng đôi m®t và các so nguyên x1, x2, . . . , xnk sao cho f(xj) ≡ 0 (mod pj) (1 ≤ j ≤ nk). Theo định lý Trung Hoa ve so dư, ton tại so nguyên x sao cho x ≡ xi+mk − m (mod pi+mk) (1 ≤ i ≤ k, 0 ≤ m ≤ n − 1). Tà đó ta có đieu phải cháng minh.
  • 38. 35 Viết đề tài giá sinh viên – ZALO:0973.287.149-TEAMLUANVAN.COM 2.3 Bat đang thfíc phân thfíc sinh b i tam thfíc b c hai trên m t khoảng Cho tam thác b c hai với h so nguyên f(x) = ax2 + bx + c. Trong mục này ta xét đieu ki n can và đủ của α, β đe luôn có bat đȁng thác au2 + bu + c 2av + b ≥ av2 + bv + c 2av + b + u − v, ∀u, v ∈ (α, β), u v và xét m®t so áp dụng liên quan. b Nh n xét rang đieu ki n đe 2ax+b /= 0 với moi x ∈ (α, β) là − 2a ∈ / (α, β). b b Đieu này tương đương với ho c − 2a ≤ α ho c − 2a ≥ β. Tà đó suy ra M nh đe 2.1. Cho tam thác b c hai với h so nguyên f(x) = ax2 + bx + c. Đieu ki n can đe có bat đȁng thác au2 + bu + c 2av + b ≥ b av2 + bv + c 2av + b + u − v, ∀u, v ∈ (α, β), u b v (2.10) là ho c − 2a ≤ α ho c − 2a ≥ β. Tiep theo, ta phát bieu và cháng minh đieu ki n đủ đe có (2.10). Định lj 2.5. Giả sả f(x) = ax2 + bx + c, a /= 0 là tam thác b c hai với h so nguyên. Khi đó, đieu ki n can và đủ đe có bat đȁng thác au2 + bu + c 2av + b ≥ b av2 + bv + c 2av + b + u − v, ∀u, v ∈ (α, β), u v là α ≥ − 2a , trong đó α, β ∈ Z. ChGng minh. Trường hợp 1. Khi a > 0. b Khi đó với v > − 2a thì (2.10) ⇔ au2 + bu + c ≥ av2 + bv + c + (u − v)(2av + b), ∀u, v ∈ (α, β)
  • 39. 36 Viết đề tài giá sinh viên – ZALO:0973.287.149-TEAMLUANVAN.COM 2 2 2 2 ⇔ a(u − v) ≥ 0, ∀u, v ∈ (α, β). Đieu này là hien nhiên. V y (2.10) thỏa mãn. b Xét v < − 2a thì (2.10) ⇔ au2 +bu+c ≤ av2 +bv+c+(u−v)(2av+b), ∀u, v ∈ (α, β), u /= v ⇔ a(u − v) ≤ 0, ∀u, v ∈ (α, β), u /= v. Đieu này không xảy ra. V y (2.10) không thỏa mãn. Trường hợp 2. Khi a < 0. b Khi đó với v > − 2a thì (2.10) ⇔ au2 +bu+c ≤ av2 +bv +c +(u−v)(2av +b), ∀u, v ∈ (α, β), u v ⇔ a(u − v) ≤ 0, ∀u, v ∈ (α, β), u /= v. Đieu này là hien nhiên. V y (2.10) thỏa mãn. b Xét v < − 2a thì (2.10) ⇔ au2 +bu+c ≥ av2 +bv+c+(u−v)(2av+b), ∀u, v ∈ (α, β), u /= v ⇔ a(u − v) ≥ 0, ∀u, v ∈ (α, β), u /= v. Đieu này không xảy ra. V y (2.10) không thỏa mãn. b V y (2.10) đúng với v > − 2a mà ta can (2.10) đúng với moi v ∈ (α, β) b nên suy ra α ≥ − 2a . Ket hợp với đieu ki n can ta có đieu ki n can và đủ được nêu trong Định lý 2.5. Tương tự ta cũng có định lý sau: Định lj 2.6. Giả sả f(x) = ax2 + bx + c, a /= 0 là tam thác b c hai với h so nguyên. Khi đó, đieu ki n can và đủ đe có bat đȁng thác au2 + bu + c 2av + b ≤ b là β ≤ − 2a . Tà Định lý 2.5, ta có av2 + bv + c 2av + b + u − v, ∀u, v ∈ (α, β), u v
  • 40. 37 Viết đề tài giá sinh viên – ZALO:0973.287.149-TEAMLUANVAN.COM / 1 2 3 1 2 3 H quả 2.1. Giả sả f(x) = ax2 + bx + c, a = 0 là tam thác b c hai với b h so nguyên và α ≥ − 2a . Khi đó, với moi b® so u1, u2, u3 ∈ (α, β) và v1, v2, v3 ∈ (α, β) sao cho u1 + u2 + u3 = v1 + v2 + v3, ta đeu có au2 + bu1 + c au2 + bu2 + c au2 + bu3 + c 1 + 2 + 3 2av1 + b 2av2 + b 2av3 + b av2 + bv1 + c av2 + bv2 + c av2 + bv3 + c + 2av1 + b + 2av2 + b . 2av3 + b Tà Định lý 2.6, ta có H quả 2.2. Giả sả f(x) = ax2 + bx + c, a b 0 là tam thác b c hai với h so nguyên và β ≤ − 2a . Khi đó, với moi b® so u1, u2, u3 ∈ (α, β) và v1, v2, v3 ∈ (α, β) sao cho u1 + u2 + u3 = v1 + v2 + v3, ta đeu có au2 + bu1 + c au2 + bu2 + c au2 + bu3 + c 1 + 2 + 3 2av1 + b 2av2 + b 2av3 + b av2 + bv1 + c av2 + bv2 + c av2 + bv3 + c + 2av1 + b + 2av2 + b . 2av3 + b Tiep theo, ta xét m®t so áp dụng liên quan đen hàm phân thác. Bài toán 2.13. Xét các so nguyên dương x, y, z có x + y + z = 12. Cháng minh rang Lài giai. 2x2 + x + 21 2y2 + y + 17 2z2 + z 13 ≥ 29207 . 4641 Theo Định lý 2.5, ta có 2u2 + u 4v + 1 ≥ 2v2 + v 4v + 1 + u − v, ∀u, v ∈ (0, +∞), u = / v. (2.11) Chon 3 so v ∈ (0, +∞) sao cho 4v + 1 ∈ {21; 17; 13} và có tőng bang 12. Ta thu được v1 = 5, v2 = 4, v3 = 3. Tiep theo, the vào (2.11), ta được 2x2 + x 21 ≥ 2y2 + y 17 ≥ 2z2 + z 13 ≥ 55 21 + x − 5, ∀x ∈ (0, +∞); 35 17 + y − 4, ∀y ∈ (0, +∞); 21 13 + z − 3, ∀z ∈ (0, +∞). C®ng các ve tương áng ta thu được đieu can cháng minh. ≥ ≤
  • 41. 38 Viết đề tài giá sinh viên – ZALO:0973.287.149-TEAMLUANVAN.COM − − − − − Bài toán 2.14. Xét các so nguyên âm x, y, z có x + y + z = −10. Tìm giá trị lớn nhat của Lài giai. 2x x2 A = 12 2y y2 + 8 2z z2 + . 6 Theo Định lý 2.6, ta có 2u u2 2 − 2v ≤ 2v v2 2 − 2v + u − v, ∀u, v ∈ (−∞, 0), u v. (2.12) Chon 3 so v ∈ (−∞, 0) sao cho 2 − 2v ∈ {12; 8; 6} và có tőng bang -10. Ta thu được v1 = −5, v2 = −3, v3 = −2. Tiep theo, the vào (2.12), ta được 2x − x2 35 12 ≤ − 12 + x + 5, ∀x ∈ (−∞, 0); 2y − y2 15 8 ≤ − 8 + y + 3, ∀y ∈ (−∞, 0); 2z − z2 4 6 ≤ − 3 + z + 2, ∀z ∈ (−∞, 0). 49 49 C®ng các ve tương áng ta thu được A ≤ − 8 . Lại có A = − 8 khi x = −5, y = −3, z = −2. 49 V y giá trị lớn nhat của A bang − 8 đạt được khi x = −5, y = −3, z = −2. Bài toán 2.15. Xét các so nguyên dương x, y, z có x + y + z = 10. Tìm giá trị nhỏ nhat của bieu thác A = 35(x2 + x) + 55(y2 + y) + 77(z2 + z). Lài giai. B®i so chung nhỏ nhat của 35, 55, 77 là 385. Ta có A = 385 Theo Định lý 2.5, ta có x2 + x + 11 y2 + y 7 z2 + z + . 5 u2 + u 2v + 1 ≥ v2 + v 2v + 1 + u − v, ∀u, v ∈ (0, +∞), u v. (2.13)
  • 42. 39 Viết đề tài giá sinh viên – ZALO:0973.287.149-TEAMLUANVAN.COM − − − Chon ba so v ∈ (0, +∞) sao cho 2v + 1 ∈ {11; 7; 5} và có tőng bang 10. Ta thu được v1 = 5, v2 = 3, v3 = 2. Tiep theo, the vào (2.13), ta được x2 + x 11 ≥ y2 + y 7 ≥ z2 + z 5 ≥ 30 11 + x − 5, ∀x ∈ (0, +∞), 12 7 + y − 3, ∀y ∈ (0, +∞), 6 5 + z − 2, ∀z ∈ (0, +∞). C®ng các ve tương áng ta thu được A = 2172 khi x = 5, y = 3, z = 2 A 385 ≥ 2172 385 hay A ≥ 2172, lại có V y giá trị nhỏ nhat của A là 2172 đạt được khi x = 5, y = 3, z = 2. Bài toán 2.16. Xét các so nguyên dương x, y, z có x + y + z = 10. Tìm giá trị lớn nhat của bieu thác x2 + x 30 M = + 2x + 1 y2 + y 12 + 2y + 1 z2 + z 6 . 2z + 1 Lài giai. Theo Định lý 2.5, ta có u2 + u 2v + 1 ≥ v2 + v 2v + 1 + u − v, ∀u, v ∈ (0, +∞), u v. (2.14) Chon ba so u ∈ (0, +∞) sao cho u2 + u ∈ {30; 12; 6} và có tőng bang 10. Ta thu được u1 = 5, u2 = 3, u3 = 2. Tiep theo, the vào (2.14), ta được 30 2x + 1 ≥ 12 2y + 1 ≥ 6 2z + 1 ≥ x2 + x 2x + 1 + 5 − x, ∀x ∈ (0, +∞), y2 + y 2y + 1 + 3 − y, ∀y ∈ (0, +∞), z2 + z 2z + 1 + 2 − z, ∀z ∈ (0, +∞). C®ng các ve tương áng ta thu được 30 12 6 x2 + x y2 + y z2 + z 2x + 1 + 2y + 1 + 2z + 1 ≥ . + + . 2x + 1 2y + 1 2z + 1 Chuyen ve trái của bat đȁng thác này sang bên phải ta thu được M ≤ 0. Ket lu n: Giá trị lớn nhat của M bang 0 khi x = 5, y = 3, z = 2.
  • 43. 40 Viết đề tài giá sinh viên – ZALO:0973.287.149-TEAMLUANVAN.COM 2.4 Bat đang thfíc sinh b i hàm phân tuyen tính trên m t khoảng Cho hàm phân thác hǎu tỉ với h so nguyên ax + b f(x) = , c > 0, ad − bc /= 0 . cx + d Trong mục này ta xét bài toán xác định các khoảng (α, β), (α < β) sao cho áng với moi hàm phân thác hǎu tỉ với h so nguyên ở trên ta đeu có bat đȁng thác au + b cu + d ad − bc ≥ (cv + d)2 av + b cv + d ad − bc (cv + d)2 + u − v, ∀u, v ∈ (α, β), u v và xét các áp dụng liên quan. d Nh n xét rang đe cx + d /= 0 với moi x ∈ (α, β) thì − c ∈ / (α, β). Đieu d d này tương đương với ho c − c ≤ α ho c − c ≥ β. Tà đó suy ra M nh đe 2.2. Cho hàm phân thác hǎu tỉ với h so nguyên ax + b f(x) = , c > 0, ad − bc /= 0 . cx + d Đieu ki n can đe có bat đȁng thác au + b cu + d ad − bc ≥ (cv + d)2 d av + b cv + d ad − bc (cv + d)2 d + u − v, ∀u, v ∈ (α, β), u v (2.15) là ho c − c ≤ α ho c − c ≥ β. Tiep theo, ta phát bieu và cháng minh đieu ki n đủ đe có (2.15). Định lj 2.7. Cho hàm phân thác hǎu tỉ với h so nguyên ax + b f(x) = , c > 0, ad − bc /= 0 . cx + d
  • 44. 41 Viết đề tài giá sinh viên – ZALO:0973.287.149-TEAMLUANVAN.COM − − — − — − cv + d (cv + d)2 cv + d (cv + d)2 Khi đó Neu ad bc > 0 thì đieu ki n can và đủ đe có bat đȁng thác (2.15) là d β ≤ − c . Neu ad bc < 0 thì đieu ki n can và đủ đe có bat đȁng thác (2.15) là d α ≥ − c . ChGng minh. Trường hợp 1. Xét ad − bc > 0. Khi đó, au + b (2.15) ⇔ ≥ av + b + (u − v) h ad − bc i , ∀u, v ∈ (α, β) c(u v)2 (ad bc) ⇔ − (cu + d)(cv + d)2 ≥ 0, ∀u, v ∈ (α, β). d d Khi đó, với u > − c thì (2.15) không thỏa mãn, với u < − c thì (2.15) thỏa d mãn, mà ta can (2.15) thỏa mãn ∀u, v ∈ (α, β) nên suy ra β ≤ − c . Trường hợp 2. Xét ad − bc < 0. Khi đó au + b (2.15) ⇔ ≤ av + b + (u − v) ad − bc , ∀u, v ∈ (α, β) c(u v)2 (ad bc) ⇔ − (cu + d)(cv + d)2 ≤ 0, ∀u, v ∈ (α, β). d Khi đó, với u > − c thì (2.15) thỏa mãn, với u < −d c thì (2.15) không thỏa d mãn, mà ta can (2.15) thỏa mãn ∀u, v ∈ (α, β) nên suy ra α ≥ − c . V y ta có đieu can cháng minh. Tương tự, ta cũng có định lý: Định lj 2.8. Cho hàm phân thác hǎu tỉ với h so nguyên ax + b f(x) = , c > 0, ad − bc /= 0 . cx + d Xét bat đȁng thác au + b cu + d ad − bc ≤ (cv + d)2 av + b cv + d ad − bc (cv + d)2 + u − v, ∀u, v ∈ (α, β), u v. (2.16) cu + d cu + d
  • 45. 42 Viết đề tài giá sinh viên – ZALO:0973.287.149-TEAMLUANVAN.COM − − x + 1 y + 1 z + 1 Khi đó Neu ad bc > 0 thì đieu ki n can và đủ đe có bat đȁng thác (2.16) là d α ≥ − c . Neu ad bc < 0 thì đieu ki n can và đủ đe có bat đȁng thác (2.16) là d β ≤ − c . Tà Định lý 2.7, Định lý 2.8 ta phát bieu ket quả đe sả dụng trong các bài toán áp dụng ở phan sau. H quả 2.3. Giả sả f(x) = ax + b cx + d là phân thác hǎu tỉ h so nguyên thỏa d mãn: c > 0, ad − bc < 0, α ≥ − c ho c c > 0, ad − bc > 0, β ≤ d — c , a.b, c, d ∈ Z. Khi đó, với moi b® so u1, u2, u3 ∈ (α, β) và v1, v2, v3 ∈ (α, β) sao cho u1 + u2 + u3 = v1 + v2 + v3, ta đeu có auk + b 3 avk + b 3 Σ cuk + d ≥ Σ cvk + d . k=1 ad − bc (cvk + d)2 ax + b k=1 ad − bc (cvk + d)2 H quả 2.4. Giả sả f(x) = cx + d là phân thác hǎu tỉ h so nguyên thỏa d mãn: c > 0, ad − bc > 0, α ≥ − c ho c c > 0, ad − bc < 0, β ≤ d — c , a.b, c, d ∈ Z. Khi đó, với moi b® so u1, u2, u3 ∈ (α, β) và v1, v2, v3 ∈ (α, β) sao cho u1 + u2 + u3 = v1 + v2 + v3, ta đeu có auk + b 3 avk + b 3 Σ cuk + d ≤ Σ cvk + d . k=1 ad − bc (cvk + d)2 k=1 ad − bc (cvk + d)2 Bài toán 2.17. Xét các so nguyên dương x, y, z có x + y + z = 10. Tìm giá trị lớn nhat của A = 36 −x + 2 + 16 −y + 2 + 9 −z + 2 .
  • 46. 43 Viết đề tài giá sinh viên – ZALO:0973.287.149-TEAMLUANVAN.COM u + 1 x + 1 y + 1 z + 1 u + 1 x + 1 y + 1 z + 1 Lài giai. Theo h quả 2.3, ta có −u + 2 u + 1 3 ≥ − (v + 1)2 −v + 2 v + 1 −3 (v + 1)2 + u − v, ∀u, v ∈ (0, +∞), u /= v ⇔ (v + 1)2 −u + 2 ≤ (−v + 2)(v + 1) − 3(u − v), u, v ∈ (0, +∞), u /= v. (2.17) Chon ba so v ∈ (0, +∞) sao cho (v + 1)2 ∈ {36; 16; 9} và có tőng bang 10. Ta thu được v1 = 5, v2 = 3, v3 = 2. Tiep theo, the vào (2.17), ta được 36 −x + 2 ≤ −18 − 3(x − 5), ∀x ∈ (0, +∞), 16 −y + 2 ≤ −4 − 3(y − 3), ∀y ∈ (0, +∞), 9 −z + 2 ≤ −3(z − 2), ∀z ∈ (0, +∞). C®ng các ve tương áng ta thu được A ≤ −22. Lại có A = −22 khi x = 5, y = 3, z = 2. V y giá trị lớn nhat của A bang -22, đạt được khi x = 5, y = 3, z = 2. Bài toán 2.18. Xét các so nguyên âm x, y, z có x + y + z = −9. Cháng minh bat đȁng thác 9(2x − 1) + 4(2y − 1) + 2z − 1 ≤ 46. Lài giai. Theo h quả 2.4, ta có 2u − 1 u + 1 3 ≤ (v + 1)2 2v − 1 v + 1 3 (v + 1)2 + u − v, ∀u, v ∈ (−∞; 0), u v ⇔ (v + 1)2 2u − 1 ≤ (2v − 1)(v + 1) + 3(u − v), u, v ∈ (−∞; 0), u v. (2.18)
  • 47. 44 Viết đề tài giá sinh viên – ZALO:0973.287.149-TEAMLUANVAN.COM x + 1 y + 1 z + 1 u + 1 Chon ba so v ∈ (−∞; 0) sao cho (v + 1)2 ∈ {9; 4; 1} và có tőng bang -9. Ta thu được v1 = −4, v2 = −3, v3 = −2. Tiep theo, the vào (2.18), ta được 9 2x − 1 ≤ 27 + 3(x + 4), ∀x ∈ (−∞; 0), 4 2y − 1 ≤ 14 + 3(y + 3), ∀y ∈ (−∞; 0), 2z − 2 ≤ 5 + 3(z + 2), ∀z ∈ (−∞; 0). C®ng các ve tương áng ta thu được đpcm. Bài toán 2.19. Xét các so nguyên dương x, y, z có x + y + z = 10. Tìm giá trị lớn nhat của bieu thác P = 36 x − 1 + 16 y − 1 + 9 z − 1 . x + 1 y + 1 z + 1 Lài giai. Theo h quả 2.4, ta có u − 1 u + 1 2 ≤ (v + 1)2 v − 1 v + 1 2 (v + 1)2 + u − v, ∀u, v ∈ (0, +∞), u /= v ⇔ (v + 1)2 u − 1 ≤ v2 − 1 + 2(u − v), ∀u, v ∈ (0, +∞), u /= v. (2.19) Chon ba so v ∈ (0, +∞) sao cho (v + 1)2 ∈ {36; 16; 9} và có tőng bang 10. Ta thu được v1 = 5, v2 = 3, v3 = 2. Tiep theo, the vào (2.19), ta được 36 x − 1 x + 1 16 y − 1 y + 1 ≤ 24 + 2(x − 5), ∀x ∈ (0, +∞), ≤ 8 + 2(y − 3), ∀y ∈ (0, +∞), 9 z − 1 z + 1 ≤ 3 + 2(z − 2), ∀z ∈ (0, +∞). C®ng các ve tương áng ta thu được P ≤ 35, ∀x, y, z ∈ (0, +∞). V y max P = 35, đạt được khi x = 5, y = 3, z = 2.
  • 48. 45 Viết đề tài giá sinh viên – ZALO:0973.287.149-TEAMLUANVAN.COM Σ Σ Σ Σ 2.5 Phân thfíc chính quy và m t so tính chat Định nghĩa 2.2 (xem [4]). Hàm f(x) xác định trên t p các so thực dương, không đong nhat với 0 được goi là phân thác chính quy neu n f(x) = akxαk , k=1 n Ví dn 2.1. De dàng kiem cháng các hàm so sau đây là phân thác chính quy f (x) = 7 + 3x + x2 + 1 1 x2 1 + x3 , f (x) = 4x2 + 3x3 + 1 2 x5 3 + x4 . Định nghĩa 2.3 (xem [4]). Hàm f(x) xác định trên t p các so thực dương, không đong nhat với 0 được goi là phân thác chính quy hǎu tỉ neu n f(x) = akxαk , k=1 n Tà định nghĩa, ta de dàng suy ra các tính chat sau: Tính chat 2.1. Neu f (x) là hàm phân thác chính quy thì f (x) > 0 áng với moi x > 0. Tính chat 2.2. Neu f(x) và g(x) là các hàm phân thác chính quy thì với moi c p so dương α, β, hàm so h(x) = αf (x) + βg(x) cũng là hàm phân thác chính quy. Tính chat 2.3. Neu f (x) và g(x) là các hàm phân thác chính quy thì hàm so h(x) = f(g(x)) cũng là hàm phân thác chính quy. Tính chat 2.4. Neu f (x) là hàm phân thác chính quy thì hàm so h(x) = [f(x)]m với m nguyên dương cũng là hàm phân thác chính quy. akαk = 0, ∀k = 1, n. k=1 akαk = 0, ∀k = 1, n. k=1 trong đó ak > 0; αk ∈ Z; trong đó ak ∈ N; αk ∈ Z;
  • 49. 46 Viết đề tài giá sinh viên – ZALO:0973.287.149-TEAMLUANVAN.COM Σ Σ f(x1, x2, . . . , xm) = Σ akxαk1 xαk2 . . . xαkm , f(x1, x2, . . . , xm) = akxαk1 xαk2 . . . xαkm , Định nghĩa 2.4 (xem [4]). Hàm so f (x1, x2, . . . , xm) không đong nhat 0, xác định với xi dương, ∀i = 1, n được goi là hàm phân thác chính quy hǎu tỉ m bien neu 1 2 m k=1 trong đó ak ∈ N; αkj ∈ Z; ∀k = 1, n; ∀j = 1, m, a1α11 + a2α21 + · · · + anαn1 = 0, a1α12 + a2α22 + · · · + anαn2 = 0, . . . , a1α1m + a2α2m + · · · + anαnm = 0. Tà định nghĩa trên, ta có tính chat sau: Tính chat 2.5. Hàm so f (x1, x2, . . . , xm) là hàm phân thác chính quy hǎu tỉ khi và chỉ khi các hàm phân thác thành phan f (1, . . . , 1, xk, 1, . . . , 1) cũng là hàm phân thác chính quy hǎu tỉ. Ví dn 2.2. De dàng kiem tra được hàm so f(x, y) = 2x3 y8 + 3x4 y9 + 6 + 1 x3y7 y là hàm phân thác chính quy, và các hàm phân thác thành phan là: f1(x) = 2x3 + 3x4 + 6x−3 + 1; f2(y) = 2y8 + 3y9 + 6y−7 + y−1 . Định lj 2.9 (xem [4]). Với moi hàm phân thác chính quy nk 1 2 m k=1 Σ n ta đeu có f(x1, . . . , xm) ≥ ChGng minh. k=1 ak, ∀xi dương, i = 1, n. Áp dụng bat đȁng thác AM - GM ta có n akxαk1 xαk2 . . . xαkm f(x1, x2, . . . , xm) 1 2 m = k=1 a1 + a2 + · · · + an a1 + a2 + · · · + an n
  • 50. 47 Viết đề tài giá sinh viên – ZALO:0973.287.149-TEAMLUANVAN.COM Σ Σ Σ − Σ − (x) = x a Σ p n n Σ akαkj = 0, ak αk − p = ak p = q − p. p = 0. Σ Σ k=1 Σ n akαk1 n akαk2 n akαkn 1 a1+a2+···+an ≥ xk=1 Σ n k=1 2 . . . xk=1 = 1 Dau "=" xảy ra khi và chỉ khi x1 = x2 = · · · = xm = 1. H quả 2.5. Với moi hàm phân thác chính quy f(x1, x2, . . . , xm) với xi dương, ∀i = 1, n, ta đeu có min f(x1, x2, . . . , xm) = f(1, 1, . . . , 1). Nh n xét 2.2. Với moi hàm phân thác dạng g(x) = n k=1 akxαk , ak ≥ 0, đ t n n ak = p, akαk = q thì hàm so k=1 k=1 −q f(x) = x p g(x) là m®t hàm phân thác chính quy. ChGng minh. Ta có n n nên f(x) = q x p g q p k=1 kxαk = ak k=1 xαk− q Σ q Σ Σ q q Đieu đó cháng tỏ f(x) là phân thác chính quy. Tà đó ta thu được định lí quan trong sau đây: Định lj 2.10 (xem [4]). Moi hàm phân thác chính quy dạng đeu có tính chat n g(x) = akxαk , ak ≥ 0, k = 1, n k=1 q g(x) ≥ g(1)xp , ∀x > 0, k=1 k=1 m x 1 n do j = 1, m . k=1 akαk −
  • 51. 48 Viết đề tài giá sinh viên – ZALO:0973.287.149-TEAMLUANVAN.COM Σ Σ trong đó ChGng minh. n ak = p, k=1 n akαk = q. k=1 −q Theo nh n xét trên, ta có f(x) = x p g(x) là m®t phân thác chính quy nên theo H quả 2.5, ta suy ra f(x) ≥ f(1) mà f(1) = g(1) nên ta có đieu can cháng minh. Ví dn 2.3. Tìm giá trị nhỏ nhat của bieu thác: f(x, y) = 2x−3 y3 + x5 y−1 + xy−5 (x > 0, y > 0). Ta de dàng kiem tra được f(x, y) là phân thác chính quy hai bien x; y. Ta có f(1, 1) = 4 f(x, y) x−3y3 x5y−1 xy−5 −3 3 5 −1 1 −5 = + 4 2 ⇔f(x, y) ≥ 4. 4 + 2 ≥ x 2 y2 x4 y 4 x4 y 4 = 1 V y min f(x, y) = 4 khi và chỉ khi x = 1, y = 1.
  • 52. 49 Viết đề tài giá sinh viên – ZALO:0973.287.149-TEAMLUANVAN.COM Chương 3 M t so bài toán liên quan đen bat đang thfíc và cfic trị trên t p so nguyên 3.1 Bat đang thfíc trên t p so nguyên Trong mục này sě trình bày m®t so bài toán ve bat đȁng thác trong t p so nguyên. Đe cháng minh bat đȁng thác trên t p so nguyên ta van áp dụng các bat đȁng thác quen thu®c như: bat đȁng thác AM - GM..., tuy nhiên không phải lúc nào cũng áp dụng trực tiep được mà can ky thu t tách, ghép m®t cách khéo léo. đây ta sả dụng tính chat của so nguyên đe tách, ghép chȁng hạn như: so nguyên dương n có the tách được thành tőng n so 1, ho c tích ab (với a, b nguyên dương) có the tách thành tőng của b so a ho c tőng của a so b. . . .Các bài toán dưới đây sě làm rõ thêm ý tưởng trên. Bài toán 3.1 (xem[3]). Với a, b, c là các so tự nhiên dương. Cháng minh rang 2 2 2 1 a + b + c a + b + c ≥ aa bb cc a+b+c .
  • 53. 50 Viết đề tài giá sinh viên – ZALO:0973.287.149-TEAMLUANVAN.COM Σ a2 + b2 + c2 a + b + c 1 n Y n Lài giai. Đ t Ta có n = a + b + c; a, b, c ∈ N∗, x1 = x2 = · · · = xa = a, xa+1 = xa+2 = · · · = xa+b = b, xa+b+1 = xa+b+2 = · · · = xa+b+c = c. a2 = a.a = x1 + x2 + · · · + xa, b2 = b.b = xa+1 + xa+2 + · · · + xa+b, c2 = c.c = xa+b+1 + xa+b+2 + · · · + xa+b+c, a a+b n aa = Y xi, bb = Y xi, cc = Y xi. Suy ra i=1 n i=a n i=a+b !1 Ta có đieu can cháng minh. i=1 i=1 Bài toán 3.2 (xem[3]). Với a, b, c là các so tự nhiên dương. Cháng minh rang ab + bc + ca a b c Lài giai. Đ t a + b + c ≥ ba+b+c ca+b+c aa+b+c . n = a + b + c; a, b, c ∈ N∗, x1 = x2 = · · · = xa = b, xa+1 = xa+2 = · · · = xa+b = c, xa+b+1 = xa+b+2 = · · · = xa+b+c = a. 1 a+b+c . aa .bb .cc = xi = xi ≥
  • 54. 51 Viết đề tài giá sinh viên – ZALO:0973.287.149-TEAMLUANVAN.COM Σ Y ! Ta thu được ab = x1 + x2 + · · · + xb, bc = xb+1 + xb+2 + · · · + xb+c, ca = xb+c+1 + xb+c+2 + · · · + xb+c+a, ba = x1x2 . . . xa, cb = xa+1xa+2 . . . xa+b, ac = xa+b+1xa+b+2 . . . xa+b+c, n ab + bc + ca = 1 (ab + bc + ac) = 1 Σ x , a + b + c n n i i=1 n !1 a ba+b+c b ca+b+c c aa+b+c = 1 ba cb ac n = n xi . i=1 Áp dụng bat đȁng thác AM-GM ta có n 1 n i=1 xi ≥ 1 n n xi i=1 với n là sô nguyên dương, xi là so thực dương, ∀i = 1, n. Tà đó suy ra đieu can cháng minh. Bài toán 3.3 (xem[3]). Với a, b, c là các so tự nhiên dương. Cháng minh rang a b a b Lài giai. Đ t (1 + a)a+b (1 + b)a+b ≥ 1 + aa+b ba+b . n = a + b; a, b ∈ N∗, x1 = x2 = · · · = xa = a, xa+1 = xa+2 = · · · = xa+b = b. Y
  • 55. 52 Viết đề tài giá sinh viên – ZALO:0973.287.149-TEAMLUANVAN.COM n ! n ! Y Y ! (1 + xi) xi i=1 i=1 Ta thu được 1 Y n 1 Y n Áp dụng Bat đȁng thác 1.4.1 ta có n i=1 (1 + xi) 1 n ≥ 1 + 1 n n xi i=1 với n là sô nguyên dương, xi là so thực dương, ∀i = 1, n. Tà đó suy ra đieu can cháng minh. Bài toán 3.4 (xem[3]). Với a, b, c là các so tự nhiên dương. Cháng minh rang Lài giai. Đ t (a + b + c)2 (a4 + b4 + c4 ) ≥ (a2 + b2 + c2 )3 . n = a + b + c; a, b, c ∈ N∗, x1 = x2 = · · · = xa = a, xa+1 = xa+2 = · · · = xa+b = b, xa+b+1 = xa+b+2 = · · · = xa+b+c = c. ! a b (1 + a)a+b .(1 + b)a+b = , a b aa+b .ba+b = .
  • 56. 53 Viết đề tài giá sinh viên – ZALO:0973.287.149-TEAMLUANVAN.COM b = x a+b c = x a+b+c Σ Σ x Σ x Σ ! ! 3 Σ ! 3 ! i=1 Khi đó ta được a2 = x1 + x2 + · · · + xa, b2 = xa+1 + xa+2 + · · · + xa+b, c2 = xa+b+1 + xa+b+2 + · · · + xa+b+c, a4 = x3 + x3 + · · · + x3 , 1 4 3 a+1 2 3 a+2 a + · · · + x3 , 4 3 a+b+1 3 a+b+2 + · · · + x3 , Σ n (a2 + b2 + c2 )3 = n i=1 3 xi . Áp dụng Bat đȁng thác 1.4.4 ta được 1 n n i ≥ i=1 3 1 n3 xi i=1 với n là sô nguyên dương, xi là so thực dương, ∀i = 1, n. Hay n n2 i ≥ i=1 n i=1 3 xi . Đây là đieu can cháng minh. Bài toán 3.5 (xem[3]). Với a, b, c là các so tự nhiên dương. Cháng minh rang (a + b + c)2 (ab3 + bc3 + ca3 ) ≥ (ab + bc + ca)3 . + x + x x n (a + b + c)2 (a4 + b4 + c4 ) = n2 3 i , ! !
  • 57. 54 Viết đề tài giá sinh viên – ZALO:0973.287.149-TEAMLUANVAN.COM a+1 a+b ca = x a+b+c Σ Σ Σ x Σ x Σ ! ! 3 Σ ! 3 ! Lài giai. Đ t Khi đó ta được n = a + b + c; a, b, c ∈ N∗, x1 = x2 = · · · = xa = b, xa+1 = xa+2 = · · · = xa+b = c, xa+b+1 = xa+b+2 = · · · = xa+b+c = a. ab = x1 + x2 + · · · + xa, bc = xa+1 + xa+2 + · · · + xa+b, ca = xa+b+1 + xa+b+2 + · · · + xa+b+c, ab3 = x3 + x3 + · · · + x3 , 1 bc3 = x3 2 3 a+2 a + · · · + x3 , 3 3 a+b+1 3 a+b+2 + · · · + x3 , (a + b + c)2 (ab3 + bc3 + ca3 ) = n2 n 3 i i=1 (ab + bc + ca)3 = n i=1 3 xi . Áp dụng Bat đȁng thác 1.4.4 ta được 1 n n i ≥ i=1 3 1 n3 xi i=1 với n là sô nguyên dương, xi là so thực dương, ∀i = 1, n. Hay n n2 i ≥ i=1 n i=1 3 xi . Đây là đieu can cháng minh. Bài toán 3.6 (xem[3]). Với a, b, c là các so tự nhiên dương. Cháng minh + x + x x n ! , !
  • 58. 55 Viết đề tài giá sinh viên – ZALO:0973.287.149-TEAMLUANVAN.COM a i √ Σ a+b q i √ aΣ +b+c q i Σ n 1 + x2 ≤ ,n2 + xi i i=1 rang a √ 1 + a2 + b √ 1 + b2 + c √ 1 + c2 ≥ q (a + b + c)2 + (a2 + b2 + c2)2. Lài giai. Đ t n = a + b + c; a, b, c ∈ N∗, x1 = x2 = · · · = xa = a, xa+1 = xa+2 = · · · = xa+b = b, xa+b+1 = xa+b+2 = · · · = xa+b+c = c. Khi đó ta được a √ 1 + a2 = Σ i=1 q 1 + x2 , b 1 + b2 = 1 + x2 , c 1 + c2 = i=a 1 + x2 , i=a+b √ √ √ Σ i=1 q a 1 + a2 + b 1 + b2 + c n 1 + c2 = n 1 + x2 , a2 + b2 + c2 = xi. i=1 Áp dụng Bat đȁng thác 1.4.2 ta có Σ q ‚ . Σ n !2 với n là sô nguyên dương, xi là so thực dương, ∀i = 1, n. Hay a √ 1 + a2 + b √ 1 + b2 + c √ 1 + c2 ≥ q (a + b + c)2 + (a2 + b2 + c2)2. Đây là đieu can cháng minh. i=1 i
  • 59. 56 Viết đề tài giá sinh viên – ZALO:0973.287.149-TEAMLUANVAN.COM √ q 1 + x , √ Σ a+b q i n 1 + x2 ≥ ,n2 + xi i i=1 Bài toán 3.7 (xem[3]). Với a, b là các so tự nhiên dương. Cháng minh rang a √ 1 + b2 + b √ 1 + a2 ≥ q (a + b)2 + 4a2b2. Lài giai. Đ t Khi đó ta được n = a + b; a, b ∈ N∗, x1 = x2 = · · · = xa = b, xa+1 = xa+2 = · · · = xa+b = a. a 1 + b2 = 2 i i=1 b 1 + a2 = 1 + x2 , i=a √ √ Σ i=1 q a 1 + a2 + b 1 + b2 = n Σ a 1 + x2 , a+b !2 Σ n !2 4a2 b2 = (ab + ba)2 = Theo Bat đȁng thác 1.4.2, ta có i=1 xi + xi i=a = xi . i=1 Σ q ‚ . Σ n !2 với n là sô nguyên dương, xi là so thực dương, ∀i = 1, n. Hay a √ 1 + b2 + b √ 1 + a2 ≥ q (a + b)2 + 4a2b2. Đây là đieu can cháng minh. Bài toán 3.8 (xem[3]). Với a, b là các so tự nhiên dương. Cháng minh rang a2 b2 c2 (a + b + c)(a2 + b2 + c2 ) 1 + a + 1 + b + 1 + c ≤ (a + b + c) + (a2 + b2 + c2) . i=1 i a Σ Σ
  • 60. 57 Viết đề tài giá sinh viên – ZALO:0973.287.149-TEAMLUANVAN.COM a Σ Σ Σ 1 + a 1 + b a + b + 2ab i=a Lài giai. Đ t Khi đó ta được n = a + b + c; a, b, c ∈ N∗, x1 = x2 = · · · = xa = a, xa+1 = xa+2 = · · · = xa+b = b, xa+b+1 = xa+b+2 = · · · = xa+b+c = c. a2 = Σ xi , 1 + a 2 i=1 a+b 1 + xi b = Σ xi , c2 a+b+c = xi , 2 2 2 i=1 a b c + + = Σ xi , 1 + a 1 + b 1 + c n n 1 + xi a2 + b2 + c2 = xi. i=1 Áp dụng Bat đȁng thác 1.4.3 ta có Σ n Σ xi n xi i=1 i=1 1 + xi n n + xi i=1 với n là sô nguyên dương, xi là so thực dương, ∀i = 1, n. Hay a2 b2 c2 (a + b + c)(a2 + b2 + c2 ) 1 + a + 1 + b + 1 + c ≤ Đây là đieu can cháng minh. (a + b + c) + (a2 + b2 + c2) . Bài toán 3.9 (xem[3]). Với a, b là các so tự nhiên dương. Cháng minh rang ab 1 + 1 ≤ 2ab(a + b) . n 1 + b 1 + xi 1 + c i=a+b 1 + xi ≤
  • 61. 58 Viết đề tài giá sinh viên – ZALO:0973.287.149-TEAMLUANVAN.COM b Σ Σ Σ Σ Lài giai. Đ t Khi đó ta được n = a + b; a, b ∈ N∗, x1 = x2 = · · · = xb = a, xb+1 = xb+2 = · · · = xa+b = b. ab Σ xi = , 1 + a ba = i=1 a+b 1 + xi xi , 1 + b 1 + xi i=b+1 n ab ba + = Σ xi , 1 + a 1 + b i=1 b 1 + xi a+b n 2ab = ab + ba = xi + i=1 i= Σ b+1 xi = xi. i=1 Áp dụng Bat đȁng thác 1.4.3, ta có Σ n Σ xi n xi i=1 i=1 1 + xi n n + xi i=1 với n là sô nguyên dương, xi là so thực dương, ∀i = 1, n. Hay ab 1 + 1 ≤ 2ab(a + b) . 1 + a Đây là đieu can cháng minh. 1 + b a + b + 2ab 3.2 Cfic trị trên t p so nguyên Trong mục này ta trình bày các bài toán ve cực trị trên t p so nguyên. Đe tìm giá trị lớn nhat, giá trị nhỏ nhat của m®t bieu thác trên t p so nguyên n ≤
  • 62. 59 Viết đề tài giá sinh viên – ZALO:0973.287.149-TEAMLUANVAN.COM − 2 # − ta can ket hợp giǎa vi c ước lượng giá trị bieu thác và các tính chat so hoc của so nguyên như: tính chia het, đong dư, tính sap thá tự. . . .Đôi khi còn phải ket hợp đánh giá bieu thác với giải phương trình nghi m nguyên đe tìm được giá trị của bien khi bieu thác đạt cực trị. Các bài toán dưới đây sě làm rõ hơn ý tưởng trên. Bài toán 3.10. Với m, n là các so nguyên dương sao cho tőng m so dương chȁn khác nhau và n so dương lẻ khác nhau bang 2369. Tìm giá trị lớn nhat của P = 3m + 2n. Lài giai. Tőng của m so dương chȁn phân bi t nhỏ nhat là m(m + 1) 2 + 4 + · · · + 2m = 2 2 Tőng của n so dương lẻ phân bi t nhỏ nhat là = m2 + m. Do đó 1 + 3 + · · · + 2n − 1 = n2 . hay 2369 ≥ n2 + m2 + m = 1 2 1 m + + n2 2 4 Ta có 1 2 m + + n2 2 9477 . 4 P = 3m + 2n = 3 m + 1 + 2n − 3 ≤ ‚ . ,(32 + 22) " m + 2 2 1 + n2 3 2 2 ≤ r 13. 9477 − 3 = 174. 4 2 Xét h phương trình 3m + 2n = 174 m2 + m + n2 = 2369. ( ≤
  • 63. 60 Viết đề tài giá sinh viên – ZALO:0973.287.149-TEAMLUANVAN.COM m Ta có 3m + 2n = 174 , suy ra n = 87 − 3 2 . Do đó m = 2t; n = 87 − 3t. Suy ra (2t)2 + 2t + (87 − 3t)2 = 2369 hay t = 20. V y h trên có nghi m m = 40, n = 27. Tóm lại P ≤ 174 với moi m, n thỏa mãn yêu cau đe bài, hơn nǎa áng với m = 40, n = 27 thì P = 174. V y giá trị lớn nhat của P là 174. Bài toán 3.11 (xem [2]). Cho k là so nguyên dương lớn hơn 2. Tìm giá trị lớn nhat của bieu thác f(x, y, z) = xy + 2xz + 3yz trên mien D = {(x, y, z) : x, y, z nguyên dương và x + y + z = k}. Lài giai. Vì t p hợp D có hǎu hạn phan tả (x, y, z) nên P sě có giá trị lớn nhat trên D. Giả sả max (x,y,z)∈D f(x, y, z) = x0y0 + 2x0z0 + 3y0z0. (3.1) Xét x0 ≥ 2. Khi đó có the xảy ra hai trường hợp 1. Neu z0 ≥ y0. De thay (x0 − 1, y0 + 1, z0) ∈ D và f(x0 − 1, y0 + 1, z0) = x0y0 + 2x0z0 + 3y0z0 + (z0 − y0) + (x0 − 1). Do x0 > 1 và z0 ≥ y0 nên tà trên suy ra f(x0 − 1, y0 + 1, z0) > f(x0, y0, z0). Đieu này mâu thuan với giả thiet (3.1). V y không xảy ra trường hợp z0 ≥ y0. 2. Neu z0 < y0. De thay (x0 − 1, y0, z0 + 1) ∈ D và f(x0 − 1, y0, z0 + 1) = x0y0 + 2x0z0 + 3y0z0 + 2(y0 − z0) + 2(x0 − 1). Do x0 ≥ 2 và z0 < y0 nên tà trên suy ra f(x0 − 1, y0 + 1, z0) > f(x0, y0, z0).
  • 64. 61 Viết đề tài giá sinh viên – ZALO:0973.287.149-TEAMLUANVAN.COM Đieu này mâu thuan với giả thiet (3.1). Suy ra không xảy ra trường hợp z0 < y0. V y không the xảy ra giả thiet x0 ≥ 2. Tà đây suy ra x0 = 1. Khi x0 = 1, bài toán trở thành: Tìm giá trị lớn nhat của bieu thác f1(y, z) = y + 2z + 3yz trên mien D1 = {(y, z) : y, z nguyên dương và y + z = k − 1}, ở đây k là so nguyên dương lớn hơn 2. Do t p hợp D1 có hǎu hạn phan tả (y, z) nên f1(y, z) sě có giá trị lớn nhat trên D1. Giả sả Ta sě cháng minh bang phản cháng. max (y,z)∈D1 f1(y, z) = y1 + 2z1 + 3y1z1. z1 ≥ y1 (3.2) Th t v y, giả sả z1 < y1. De thay (y1 − 1, z1 + 1) ∈ D1 và f1(y1 − 1, z1 + 1) = y1 + 2z1 + 3y1z1 + 3(y1 − z1 − 1) + 1. Do z1 < y1 và z1, y1 là so nguyên nên z1 + 1 ≤ y1. Suy ra f(y1 − 1, z1 + 1) > f(y1, z1). Vô lý. V y giả thiet z1 < y1 sai , (3.2) đúng nghĩa là z1 ≤ y1 Ta lại cháng minh bang phản cháng. Th t v y, giả sả z1 − y1 ≤ 1 (3.3) z1 − y1 > 1 (3.4) De thay (y1 + 1, z1 − 1) ∈ D1 và f1(y1 + 1, z1 − 1) = y1 + 2z1 + 3y1z1 + 3(z1 − y1 − 1).
  • 65. 62 Viết đề tài giá sinh viên – ZALO:0973.287.149-TEAMLUANVAN.COM − 1 1 − Do z1 − y1 > 1 và z1, y1 là so nguyên nên z1 − y1 − 1 ≥ 1, suy ra f1(y1 − 1, z1 + 1) > f1(y1, z1). Đieu này mâu thuan với giả thiet. Cháng tỏ đieu giả sả (3.4) là sai. V y (3.3) đúng. Tà (3.2) và (3.3) ta có 0 ≤ z1 − y1 ≤ 1. Như v y có hai khả năng xảy ra 1. Neu z1 − y1 = 0 hay z1 = y1 , suy ra z1 = y1 = k − 1 . 2 Vì the đe z1, y1 nguyên thì k − 1 chia het cho 2 hay k lẻ. Lúc này max (x,y,z)∈D f(x, y, z) = f 1, k − 1 , 2 k − 1 2 = 3 k − 1 2 (k 1)2 + 3 4 3k2 + 1 = . 4 2. Neu z1 − y1 = 1 hay z1 = y1 + 1 , tà z1 + y1 = k − 1, suy ra y = k − 2 , z = k . 2 2 Vì the đe z1, y1 nguyên thì k chia het cho 2 hay k chȁn . Lúc này max (x,y,z)inD f(x, y, z) = f 1, k − 2 , k 2 2 3k2 4 = . 4 Ket lu n max (x,y,z)∈D f(x, y, z) = 3k2 + 1 4 3k2 − 4 4 khi k lẻ khi k chȁn. Bài toán 3.12 (xem [2] ). Cho k là so nguyên dương lớn hơn 2. Tìm giá trị lớn nhat của hàm so f(x, y, z) = xyz trên mien D = {(x, y, z) : x, y, z nguyên dương và x + y + z = k}. Lài giai. Không giảm tính tőng quát, giả sả x ≥ y ≥ z.
  • 66. 63 Viết đề tài giá sinh viên – ZALO:0973.287.149-TEAMLUANVAN.COM Vì D là t p hǎu hạn các phan tả (x, y, z) nên f(x, y, z) sě đạt giá trị lớn nhat trên D. Giả sả max (x,y,z)∈D f(x, y, z) = f(x0, y0, z0) = x0y0z0 vớix0 ≥ y0 ≥ z0. (3.5) Giả sả x0 − z0 > 1. Khi đó chỉ có the xảy ra ba trường hợp 1. x0 = y0 > z0 + 1. Ta có x0 + y0 + z0 = k hay x0 + (y0 − 1) + (z0 + 1) = k. M t khác do x0 = y0 > z0 +1 và z0 > 0 nên x0, (y0 −1), (z0 +1) cũng nguyên dương, tác là (x0, y0 − 1, z0 + 1) ∈ D. Lại có f(x0, y0 − 1, z0 + 1) = x0(y0 − 1)(z0 + 1) = x0y0z0 + x0(y0 − z0 − 1). Do x0 = y0 > z0 + 1 nên tà trên suy ra f(x0, y0 − 1, z0 + 1) > x0y0z0 = f(x0, y0, z0). Đieu này mâu thuan với (3.5).V y trường hợp 1 không the xảy ra. 2. Neu x0 > y0 > z0 . Xét b® ba nguyên dương (x0 − 1, y0, z0 + 1). Do x0 > y0 > z0 và x0, y0, z0 nguyên dương nên x0 ≥ y0 + 1 và y0 ≥ z0 + 1, suy ra x0 − 1 > 0. Rõ ràng (x0 − 1) + y0 + (z0 + 1) = k nên (x0 − 1, y0, z0 + 1) ∈ D. M t khác f(x0 − 1, y0, z0 + 1) = y0(x0 − 1)(z0 + 1) = x0y0z0 + y0(x0 − z0 − 1) > x0y0z0. Suy ra f(x0 − 1, y0, z0 + 1) > f(x0, y0, z0). Đieu này mâu thuan với (3.5). Suy ra trường hợp 2 không the xảy ra. 3. x0 − 1 > y0 > z0. L p lu n tương tự như trên ta có (x0 − 1) + (y0 + 1) + z0 = k nên (x0 − 1, y0 + 1, z0) ∈ D và f(x0 − 1, y0 + 1, z0) = x0y0z0 + z0(x0 − y0 − 1) > f(x0, y0, z0).
  • 67. 64 Viết đề tài giá sinh viên – ZALO:0973.287.149-TEAMLUANVAN.COM − − Đieu này mâu thuan với (3.5). Suy ra trường hợp 3 không the xảy ra. Đieu đó cháng tỏ giả thiet x0 − z0 > 1 là sai. V y ta phải có x0 − z0 ≤ 1. Do v y chỉ có the xảy ra hai trường hợp a. Neu x0 − z0 = 0. Ket hợp với x ≥ y ≥ z và x0 + y0 + z0 = k, ta có k x0 = y0 = z0 = 3 . Đieu này xảy ra khi và chỉ khi k chia het cho 3. b. Neu x0 − z0 = 1. Lúc này lại có hai khả năng b1. x0 = y0 + 1 = z0 + 1. Suy ra k + 2 x = , y = z = k − 1 . 0 3 0 0 3 Trường hợp này xảy khi k ≡ 1 (mod 3). b2. x0 = y0 = z0 + 1. Suy ra x0 = y0 k + 1 = , z0 3 = k − 2 . 3 Trường hợp này xảy khi k ≡ 2 (mod 3). Ket lu n max (x,y,z)∈D f(x, y, z) = k3 27 khi k ≡ 0 (mod 3) (k + 2)(k 1)2 27 khi k ≡ 1 (mod 3) (k 2)(k + 1)2 27 khi k ≡ 2 (mod 3). Bài toán 3.13 (xem [2]). Tìm giá trị nhỏ nhat của hàm so f(x, y, z, t) = x2 + y2 + 2z2 + t2 trên mien D = {(x, y, z, t) : x, y, z, t ∈ N; x2 − y2 + t2 = 21; x2 + 3y3 + 4z2 = 101}. Lài giai. Lay (x, y, z, t) tùy ý thu®c D, ta có x2 − y2 + t2 = 21, x2 + 3y3 + 4z2 = 101.
  • 68. 65 Viết đề tài giá sinh viên – ZALO:0973.287.149-TEAMLUANVAN.COM C®ng tàng ve hai phương trình trên ta được 2x2 + 2y2 + 4z2 + t2 = 122. Suy ra 2(x2 + y2 + 2z2 + t2 ) = 122 + t2 . Tà đó suy ra f(x, y, z, t) ≥ 61 với moi (x, y, z, t) ∈ D. Đȁng thác xảy ra khi t = 0. Xét h phương trình H này tương đương với t = 0 x2 − y2 + t2 = 21 x2 + 3y3 + 4z2 = 101 x, y, z ∈ N. t = 0 x2 − y2 = 21 x2 + 3y3 + 4z2 = 101 x, y, z ∈ N. Vì x2 − y2 = 21 và x, y nguyên dương nên x + y = 21 x − y = 1 ho c x + y = 7 x − y = 3. Suy ra x = 11, y = 10 ho c x = 5, y = 2. Tà x2 + 3y3 + 4z2 = 101, ta có x = 5, y = 2, z = 4. Như v y (5, 2, 4, 0) ∈ D, f(5, 2, 4, 0) = 61 . V y max (x,y,x)∈D f(x, y, z) = 61. Bài toán 3.14 (xem [2]). Tìm giá trị nhỏ nhat của hàm so f(x) = |5x2 + 11xy − 5y2 | ( (
  • 69. 66 Viết đề tài giá sinh viên – ZALO:0973.287.149-TEAMLUANVAN.COM 2 2 . . 2 2 .5x2 + 11xy − 5y2. /= 1 . 2 2 trên mien D = {(x, y) : x, y ∈ N; y là so lẻ }. Lài giai. Lay (x, y) tùy ý thu®c D, ta có y là so tự nhiên lẻ nên 5y2 là so lẻ. Neu x là so chȁn thì 5x2 + 11xy là so chȁn, suy ra 5x2 + 11xy − 5y2 là so lẻ. Neu x là so lẻ thì 5x2 , 11xy là so lẻ , suy ra 5x2 + 11xy − 5y2 là so lẻ. V y trong moi trường hợp, ta đeu có 5x2 + 11xy − 5y2 là so lẻ. Ta sě cháng minh |5x2 + 11xy − 5y2 | = 1. Th t v y, giả sả Suy ra |5x + 11xy − 5y | = 1. 20 .5x2 + 11xy − 5y2. = 20 hay ⇔| (10x + 11y) − 221y | = 20, (10x + 11y)2 − 221y2 = 20 ho c (10x + 11y)2 − 221y2 = −20. Do 221 = 13.17, bình phương m®t so nguyên tùy ý đong dư với 1, 3, 4, -1, -3, -4 theo modulo 13, 20 đong dư 7 theo modulo 13, -20 đong dư -7 theo modulo 13 nên không ton tại x, y thỏa mãn hai đȁng thác trên. V y L p lu n tương tự ta cũng có 5x2 + 11xy − 5y2 = / 3. Ket hợp với |5x2 + 11xy − 5y2 | là so lẻ, nên suy ra |5x + 11xy − 5y | ≥ 5. Tác là ta có f(x, y) ≥ 5 với moi (x, y) thu®c D, lại có (0, 1) ∈ D, f(0, 1) = 5. V y giá trị nhỏ nhat của hàm f(x, y) trên mien D là 5 đạt được khi (x, y) = (0, 1).
  • 70. 67 Viết đề tài giá sinh viên – ZALO:0973.287.149-TEAMLUANVAN.COM − − Bài toán 3.15 (xem [2]). Tìm giá trị lớn nhat và nhỏ nhat của hàm so f(x, y, z) = x 2z + y t trên mien D = {(x, y, z) : x, y, z, t ∈ Z; 1 ≤ x ≤ y ≤ z ≤ t ≤ 100}. Lài giai. 1. Lay (x, y, z, t) tùy ý thu®c D. Do 1 ≤ x ≤ y ≤ z ≤ t ≤ t nên x y ≤ 1, 2z t ≤ 2. Suy ra f(x, y, z, t) ≤ 3 với moi (x, y, z, t) thu®c D. Rõ ràng (1, 1, 1, 1) ∈ D và f(1, 1, 1, 1) = 3. V y max (x,y,z,t)∈D f(x, y, z, t) = 3. 2. Lay (x, y, z, t) tùy ý thu®c D. Do 1 ≤ x ≤ y ≤ z ≤ t ≤ 100 nên x z 1 y hay f(x, y, z, t) = + 2 ≥ y t 1 + 2 y 100 y f(x, y, z, t) ≥ 1 y + . y 50 Xét hàm so g(x) = thay a. Neu y ≤ 7 thì y + 50 với 1 ≤ y ≤ 100 và y là so nguyên. Ta nh n Suy ra g(y) g(7) = (7 − y)(50 − 7y) 350y ≥ 0. g(y) ≥ g(7), ∀y = 1, 2, 3, 4, 5, 6, 7. b. Neu y ≥ 8 thì g(y) g(8) = (8 − y)(50 − 8y) 350y ≥ 0.
  • 71. 68 Viết đề tài giá sinh viên – ZALO:0973.287.149-TEAMLUANVAN.COM Suy ra g(y) ≥ g(8), ∀y = 9, 10, . . . , 100. Tà đó suy ra g(y) ≥ min g(7), g(8), ∀y = 1, 2, . . . .., 100 hay Tà đó, ta có g(y) ≥ min {g(7), g(8)} = min { 99 99 , 350 114 400 } = 99 . 350 và f(1, 7, 7, 100) = f(x, y, z, t) ≥ 99 . Do đó 350 350 , ∀(x, y, z, t) ∈ D 90 min (x,y,z,t)∈D f(x, y, z, t) = . 350
  • 72. 69 Viết đề tài giá sinh viên – ZALO:0973.287.149-TEAMLUANVAN.COM Ket lu n Lu n văn “Bat đȁng thác và bài toán cực trị trong lớp các đa thác và phân thác h so nguyên ” đã giải quyet được nhǎng van đe sau: 1. Lu n văn đã trình bày chi tiet m®t so dạng toán ve đa thác và phân thác với h so nguyên. 2. Trình bày các dạng toán ve ước lượng mien giá trị của đa thác và phân thác đại so và m®t so bài toán cực trị tương áng. 3. Cuoi cùng, lu n văn trình bày m®t so bài toán ve bat đȁng thác và cực trị trên t p so nguyên trong các đe toán thi hoc sinh giỏi trong nước, Olympic khu vực và quoc te.
  • 73. Viết đề tài giá sinh viên – ZALO:0973.287.149-TEAMLUANVAN.COM Tài li u tham khảo [A] Tieng Vi t [1] Tran Xuân Đáng (2010), Đa thúc với các h so nguyên và đong dư thúc, K yeu ”Chuyên đe boi dưỡng hè 2010”, ĐHKHTN, Hà N®i, 2010. [2] Phan Huy Khải (2008), Giá tr lớn nhat và nhó nhat của hàm so, NXB Giáo dục. [3] Nguyen Vũ Lương (2006), Các bài giảng đay đủ ve bat đȁng thúc Cauchy, NXB ĐHQGHN. [4] Nguyen Văn M u (2003), Đa thúc đại so và phân thúc hũu ty, NXB Giáo dục. [5] Nguyen Văn M u, Lê Ngoc Lăng, Phạm the Long, Nguyen Minh Tuan (2006), Các đe thi olympic Toán sinh viên toàn quoc, NXB Giáo dục. [6] Nguyen Văn M u, Nguyen Văn Ngoc (2009), Đa thúc đoi xúng và áp dựng, NXB Giáo dục. [B] Tieng Anh [7] Victor Prasolov (2001), Polynomial in serie Algorithms and Computation in Mathematics, Vol.11, Springer-Verlag, Berlin-Heidelberg, 2010. [8] Borcea, J., R. Pereira, M. Putinar (2009), Hausdorff geometry of complex polynomials, positive charge distributions and normal operators, June 29, 2008-July 6, 2008.